Vous êtes sur la page 1sur 2054

AZIMAT NUCLEAR MEDICINE

BY
DR ZOOL HILMI
2012

SIMPLE PHYSIC

Radionuclide

Decay mode

Half time

Energy (keV)

Production
method

Thallium-201

Electron capture

73.1 hr

69-83(94%) ,
135(3%) 167(10%)

Cyclotron

Gallium-67

Electron capture

78.3 hr

93(37%),185(20%),3
00(17%),395(5%)

Cyclotron

Indium-111

Electron capture

2.8 d

171(90%),245(94%)

Cyclotron

Iodine-123

Electron capture

13.2 hr

159

Cyclotron

Iodine-131

Beta minus

8d

364

Reactor

Technetium-99m

Isomeric
transition

6 hr

140

Generator

Cobalt-57

Electron capture

272 d

122

Cyclotron

Fluorine-18

Positron emitting

110 min

0.653 MeV

Cyclotron

Gallium-68

Positron emitting

68 min

1.9 MeV

Generator

Carbon-11

Positron emitting

20 min

0.96 MeV

Cyclotron

Nitrogen-13

Positron emitting

10 min

1.19 MeV

Cyclotron

Oxygen-15

Positron emitting

2 min

1.70 MeV

Cyclotron

Rubidium-82

Positron emitting

1.3 min

3.15 MeV

Generator

Coherent/Rayleigh/
classical scatter

-Low energy photon excites atom but then


passes through without energy transfer

Pair production

-High energy photon interact with nucleus


& converted to electron and photon
-Positron and electron annihilation with
two 511 keV 180 degree

Photodisintegration
/xray
interaction with
matter

Photo electric

Compton scatter

-High energy photon absorbed by nucleus


resulting disintegration of nucleus
-Energy threshold 15 MeV

-Photon interact with inner shell electron


-Photon energy totally absorbed by inner
shell electron
-Electron is ejected (photoelectron)
-Outer shell electron fill the inner shellcharacteristic radiation or Auger electron
-Energy of emitted photoelectron =
difference of incident photon energy and
electron binding energy
-Low energy dominant
-PE 3 /
-Photon interact with outer shell electron
-Loosely bound electron
-High energy photon more straight scatter
-Compton electron density/ E

-Consist 2 neutrons and 2 protons = Helium


-Atomic no > 82
-In decay, atomic no decreases by 2 and mass
no decreases by 4
-Travel from 1 10 cm in air
-Travel < 0.1 cm in tissue
-High risk if ingested or injected

Alpha

Electron

Decay
Beta

Positron

Electron
capture
Isomeric
transition
Gamma

Internal
conversion

-Neutron rich
-n/p high
-n= p + + v

-High proton
-n/p low
-n= n + + v

-High proton
-n/p low
-p + e = n + v

-Tc-99m = + Tc99

Q = EXPOSURE
AMOUNT OF ELECTRICAL CHARGE
PRODUCED
BY
IONIZING
ELECTROMEGNETIC RADIATION PER
MASS OF AIR
UNIT = C/Kg = Roentgen
1 R = 2.58 x 10 C/Kg

D = ABSORB DOSE
DIRECTLY IONIZATION CHARGE
PARTICLE DEPOSIT ENERGY BY
(EXCITATION/IONIZING) PER UNIT
MASS
UNIT = J/Kg
1 Gray = 1 J/ Kg
100 rad = 1 Gray
Rad = RADIATION ABSORB DOSE

SOURCE

SKIN DOSE
RADIATION EXPOSURE INCIDENT TO
PATIENT DUE TO RADIOLOGICAL
EXAMINATION
CHEST X RAY = 0.1 0.2 mGy
SKULL
= 1.5 mGy
ABDOMEN = 3 mGy
LUMBAR
= 10 mGy

H = EQUIVALENT DOSE
ICRP 1990
MEASURE OF RADIATION SPECIFIC
BIOLOGIC DAMAGE IN HUMAN
UNIT = Sievert
100 rem = 1 Sv = J/Kg
WR = RADIATION
FACTOR
XRAY///e
PROTON > 2 MeV
NEUTRON
LPHA

WEIGHTING
=1
=5
= 5 20
= 20

E = EFFECTIVE DOSE
ICRP 1990
MEASURE OF RADIATION AND
ORGAN SYSTEM SPECIFIC DAMAGE
IN HUMAN
UNIT = Sievert
100 rem = 1 Sv = J/Kg

K = KERMA
ENERGY CARRIED BY PHOTON (OR
OTHER
INDIRECT
IONIZING
RADIATION) TRANSFER ENERGY TO
CHARGE
PARTICLE
(
PE/COMPTON/PAIR PRODUCTION

UNIT = J/Kg OR Gray

WT = TISSUE WEIGHTING FACTOR


GONAD = 0.2
BONE
MARROW/LUNG/STOMACH/COLON
= 0.12
BLADDER/BREAST/LIVER/THYROID
= 0.05
SKIN/BONE = 0.01

PARATHYROID SCINTIGRAPHY

Pearls & Pitfalls

Parathyroid
imaging

The most common indication for


parathyroid imaging is to localize the
hyperfunctioning gland (adenoma) either
in the thyroid bed or in the ectopic
location (lower neck or mediastinum)
Parathyroid adenomas are usually single
Parathyroid imaging is usually performed
by using Tc-99m sestamibi with
sequential images after 2 hours, whereas
parathyroid
adenomas
usually
hyperconcentrate Tc-99m sestamibi and
persist over time

PARATHYROID SCINTIGRAPHY
ANATOMY AND EMBRYOLOGY

ANATOMY

4 parathyroid glands
Measuring 6mm x 3mm
Weighing 35 40 gram
Rarely, may be only 2 glands or as many as 8
glands

EMBRYOLOGY

Inferior parathyroid glands arise from 3rd


branchial pouch and migrate caudally with the
thymus
Superior glands arise from 4th branchial pouch
and migrate with the thyroid

Type of cells in parathyroid


1. Chief cell produces parathyroid hormone
(PTH) and it has little mitochondria or
cytoplasma
1. Oxyphil cell high number of mitochondria.

Pathophysiology
Hyperpathyroidism :
1. Primary hyperpathyroidism
2. Secondary hyperpathyroidism
3. Tertiary hyperpathyroidism

Hyperparathyroidism
Chief cells (single or multiple glands)synthesis
more and release more PTH
PTH synthesize, stored and secreted by
parathyroid glands
PTH responsible for calcium and phosphorus
homeostasis by its action on bone, small
intestine and kidneys

Scan of Tc-99m MIBI

Tc-99m MIBI
Heart
GI
Kidney
Bladder

Normal uptake for Tc-99m MIBI


Salivary gland
Parotid gland
Renal
Bladder
Heart
Intestine
Thyroid
Parathyroid

Concerning imaging of parathyroid


glands
A. Approximately 50% of cases of primary
hyperparathyroidism are due to a single functioning
adenoma
B. The sensitivity of parathyroid scintigraphy in the
detection of adenomas within the gland approaches
75%
C. In secondary hyperparathyroidism approximately 80%
of hyperplastic glands will be visualized
D. The majority are parathyroid adenomas are
functioning
E. False positive scans may occur in the presence of
coexisting thyroid disease

Concerning imaging of parathyroid


glands
A.
B.
C.
D.
E.

F
T
F
T
T

Tc-99m sestamibi uptake may be found


in
A. Parathyroid hyperplasia
B. Parathyroid adenoma
C. Thyroid adenoma

Tc-99m sestamibi uptake may be found


in
A. T
B. T
C. T

In performing Tl-201/Tc-99m
pertechnitate parathyroid studies, it is
best perform the pertechnitate study
first

In performing Tl-201/Tc-99m
pertechnitate parathyroid studies, it is
best perform the pertechnitate study
first
FALSE

What is the typical finding of


parathyroid adenomas on parathyroid
imaging with Tc-99m sestamibi?

What is the typical finding of


parathyroid adenomas on parathyroid
imaging with Tc-99m sestamibi?
Parathyroid adenomas show delayed washout
compared with normal thyroid tissue

Hyperparathyroidism
1. Tc-99m sestamibi is taken up by thyroid and
parathyroid tissue, but wash out more rapidly
from the thyroid.
2. Parathyroid adenoma in the region of the left
lower lobe of the thyroid.
3. > 90% predictive value for preoperative
localization of parathyroid adenoma, lower test
accuracy for hyperplasia and small tumors.
4. Thyroid follicular adenoma.

Parathyroid adenoma
1. Dual isotope imaging with subtraction, in the past using
Tl-201 and Tc-99m pertechnitate and recently using I-123
and Tc-99m sestamibi
2. I-123 by mouth. After a delay of 2 to 3 hours, an anterior I123 thyroid scan is obtained. Without moving the patient,
an image is obtained after IV injection of Tc-99m MIBI. The
I-123 image is computer subtracted from the Tc-99m MIBI
image.
3. The I-123 thyroid scan appears normal, although the left
lobe extends more inferiorly. The MIBI image shows an
asymmetrical bulbous configuration in the region of the
right lower pole of the thyroid. Subtraction demonstrates
focal radiotracer compatible with parathyroid adenoma at
the lower pole of the right thyroid.
4. Parathyroid, thyroid adenoma, thyroid carcinoma and
metastatic carcinoma.

Mediastinal parathyroid adenoma


1. Parathyroid scan with Tc-99m sestamibi or Tc99m tetrofosmin.
2. Focal persistent uptake in the mediastinum,
normal salivary, liver, cardiac uptake. Axillary
uptake resolves with arm elevated, thus is
caused by skin folds.
3. Various benign and malignant neoplasm.
4. Mediastinal parathyroid adenoma.

ADRENAL SCINTIGRAPHY

Pearls & Pitfalls


NP-59 is an adrenal cortical
imaging agent with imaging
performed at 4-5 days.
Adrenocortical
Unilateral adrenal uptake is
imaging
usually an adenoma and
bilateral uptake is usually due
to adrenal hyperplasia

ADRENAL SCINTIGRAPHY
Common indication
(Cushings syndrome)

is

hypercortisolism

Less common :
a) Hyperaldosterolism (Conns syndrome)
b) Adrenal virilizing tumors

ADRENAL

CORTEX

ZONA
GLOMERULOSA

ZONA
FASCICULATA

ALDOSTERONE

CORTISOL

CONNS

CUSHINGS

MEDULLA

ZONA
RETICULARIS

ANDROGENS

EPINEPHRINE

Drugs that interfere with NP 59


uptake
Drug

Mechanism

Spironolactone

Aldosterone receptor blocker

Ketoconazole

Suppression of cortical biosynthesis

Diuretic/oral contraception

Stimulate renin/angiotensin

Glucocorticoids

Suppression of ACTH

Cholesterol lowering agents

Decrease cholesterol

Hypercholesterolemia

Decrease LDL receptor activity

HYPERALDOSTERONISM NP-59
PATTERN
SYMMETRICAL
BILATERALLY EARLY
IMAGING (BEFORE DAY 5)

-Bilateral autonomous
hyperplasia
-Secondary
aldosteronism

UNILATERAL EARLY
IMAGING (BEFORE
DAY 5)

SYMMETRICAL LATE
IMAGING (ON OR
AFTER DAY 5)

-Conns tumor
-aldosterone
secreting tumor

Normal adrenal

CUSHINGS SYNDROME NP-59


PATTERN
SYMMETRIC

BILATERAL HYPERPLASIA

ASYMMETRIC

BILATERAL HYPERPLASIA
(SOME ASYMMETRY IS
COMMON)

BILATERAL
VISUALIZATION

ADRENAL
SCINTIGRAM

UNILATERAL
VISUALIZATION

ADENOMA

BILATERAL NON
VISUALIZATION

CARCINOID

DRUG THERAPY

BILATERAL HYPERPLASIA
ASSOSIATED WITH
UNILATERAL ADENOMA

ADRENAL REMNANT
AFTER
ADRENALECTOMY

Concerning scintigraphy of the adrenal


A. Asymmetrical uptake of NP 59 may be seen in two
thirds in normal subject
B. Bilateral symmetrical uptake of NP 59 in the presence
of glucocorticoid excess is usually due to adrenal
hyperplasia
C. Unilateral increased uptake following NP 59 is most
commonly due to functioning adenoma
D. Bilateral non visualization of the adrenals in the
presence of endogenous glucocorticoid excess is most
likely to be due to adrenal carcinoma
E. Unilateral non visualization exclude an adrenal
carcinoma

Concerning scintigraphy of the adrenal


TTTTF

In the scintigraphy assessment of


adrenal hypertension
A. Pretreatment with dexamethasone in patients with
primary hyperaldosteronism significantly improves
the diagnostic accuracy of NP 59 scanning
B. Following dexamethasone suppression, a Conns
adenoma will usually be seen before day five
C. NP 59 scintigraphy can distinguish ACTH dependent
from ACTH independent Cushings syndrome
D. False negative results may be seen during MIBG
scanning in patients with pheochromocytoma on
steroid therapy
E. Activity in the normal gland is usually suppressed in
the presence of a unilateral pheochromocytoma

In the scintigraphy assessment of


adrenal hypertension
TTTFF

Adrenocortical scintigraphy
1. A, Unilateral NP-59 uptake consistent with left
adrenal adenoma, concordant with the CT. B,
Bilateral adrenal hyperplasia, discordant with CT.
2. Uptake in the right adrenal bed region indicating
adrenal remnant. Discordant with the negative
CT.
3. Transport and receptors system for serum
cholesterol account for adrenal uptake.
Cholesterol is required for the production of
adrenal hormones.
4. Hyperaldosteronism (adrenal adenoma or
hyperplasia) and hyperandrogenism.

ADRENOMEDULLARY
SCINTIGRAPHY

Pearls & Pitfalls


MIBG is a medullary adrenal
imaging agent which effectively
localizes in pheochromocytoma
Adrenomedullary
and neuroblastoma. It may also
imaging
localize in carcinoid, medullary
thyroid
carcinoma
and
paraganglioma.

Heart
Liver
Kidney
Bladder

MIBG scan
Heart
Liver
Kidney
Bladder

Thyroid blockade
Potassium perchlorate for emergency and
allergic to iodine= 400 mg
Compound

Daily dose

Capsules
Potassium iodate

170 mg

Potassium iodide

130 mg

Solution
Lugol 1%

1 drop/kg max 40 (20 drops bd)

Oncology Committee of the EANM

In-111 pentetreotide (octreoscan)


normal uptake at kidney, spleen, liver ,
thyroid and intestinal activity

Somatostatin analog limitations

False negative
Small &
Few somatostatin receptors

Multiple endocrine neoplasia (MEN)


type 1
Dominantly inherited
Mutation of MEN 1 gene on chromosomal region
11q13
Include :
Parathyroid = hyperplasia or adenoma (90%)
Pancreatic islet = adenoma, carcinoma or
hyperplasia (80%)
Anterior pituitary = adenoma (65%)
Adrenal cortex = hyperplasia or adenoma (40%)
Carcinoid tumors and lipomata

Multiple endocrine neoplasia (MEN)


type 2

Parafollicular cells of thyroid = Medullary


carcinoma (> 95%)
Adrenal adrenal = pheochromocytoma (50%)
Parathyroid = hyperplasia or adenoma (<25%)

The following tissues may show


increased uptake of MIBG
A.
B.
C.
D.
E.

Non-secreting paraganglioma
Small (oat) cell bronchogenic carcinoma
Squamous carcinoma of the bladder
Medullary carcinoma of the thyroid
The heart

The following tissues may show


increased uptake of MIBG
TTFTT

The following are true concerning


MIBG scanning
A. Focal bone marrow activity in the presence of a
known neuroblastoma may be normal
B. The sensitivity of MIBG scanning in the detection of
neuroblastoma is approximately 90%
C. MIBG scanning is more accurate than CT or MRI in the
detection of primary adrenal pheochromocytoma
D. Increased adrenal uptake of MIBG in the MEN
syndrome
confirm
the
presence
of
pheochromocytoma
E. MIBG scanning accurately reflects the extent of bony
involvement in metastatic neuroblastoma

The following are true concerning


MIBG scanning
FTFFF

A. I-131 NP 59
B. In-111-Octreoscan
C. I-131 MIBG
1.
2.
3.
4.
5.

Adrenal cortical adenoma


Pituitary adenoma
Medullary thyroid carcinoma
Pheochromocytoma
Carcinoid tumor

A. I-131 NP 59
B. In-111-Octreoscan
C. I-131 MIBG
1.
2.
3.
4.
5.

Adrenal cortical adenoma = A


Pituitary adenoma = B
Medullary thyroid carcinoma =B
Pheochromocytoma = C
Carcinoid tumor = B

Neuroblastoma
1. Bone scan shows uptake symmetrically in the distal
femurs, cranial and facial bones. I-131 MIBG shows a large
area of midline abdominal uptake. Inspection of the bone
scan in the same area suggests a soft tissue left perirenal
density, best seen in the anterior view. In addition, diffuse
marrow/bone uptake is seen on the MIBG study.
2. The prominent mid line uptake on the MIBG is consistent
with neuroblastoma. Subtle bone uptake is seen in the
region. The symmetrical bone uptake in the distal femurs,
cranial and facial bone is very suggestive of tumor. The
MIBG confirms metastatic disease with extensive tumor in
the marrow/ bone from skull to feet.
3. First, primary neuroblastoma. Osteosarcoma metastatic to
the lung is another. Metastases of various tumors
occasionally are seen on bone scans, lung, colon and
breast.
4. Combination of Tc-99m bone scan and I-131 MIBG.

Octreoscan
1. In-111 octreoscan (octreotide), I-131 MIBG.
2. Prominent liver, spleen and kidney uptake is
seen with In-111 octreoscan.
3. Focal abnormal uptake in the right temporal
bone that correlates with CT.
4. Paraganglioma considering the patients
history, also meningioma or NET metastasis.

Pheochromocytoma
1. I-131 MIBG
2. Localization occurs through the norepinephrine
reuptake
mechanism.
It
localizes
in
catecholamine storage vesicles in presynaptic
adrenergic nerve ending and cells of the adrenal
medulla.
3. Sensitivity: 90%, specificity, 95% for detection of
pheochromocytoma.
4. Neuroblastoma (90%), carcinoid (50%) and
medullary thyroid carcinoma (25%).

NET
1. Peptide analogue of somatostatin and
octreotide. Binds to tumors with somatostatin
receptors.
2. NET
3. A, multiple metastases to both lobe of the liver.
Two large foci and one small focus of uptake
consistent with paraaortic tumor adenopathy.
Possible small tumor in right hilum. B,
prominent irregular uptake in the anterior
mediastinum and focal uptake in the lower lung
posteriorly.
4. Kidney and spleen.

Infection and inflammation

Pearls & Pitfalls

Infection and
inflammation

Gallium-67 citrate and In-111


leukocyte scans are usually
photon poor and the images
are coarse or grainy.
Tc-99m leukocyte scan have
more counts and the images
appear less grainy and
smoother.
Gallium-67 photon energies
are 90, 190, 290 and 390 keV.
In-111 photon energies are 173
keV and 247 keV.

Pearls & Pitfalls

Infection and
inflammation

Gallium-67 activity is normally


seen in the skeleton, lacrimal
glands, nasopharynx and liver.
Liver activity is usually greater
than spleen. Colon activity is
normal on delayed images.
Indium-111 leukocyte activity is
normally seen in the bone
marrow, liver and spleen with
spleen more intense than liver.
Patchy lung activity may be due
to leukocyte damaged during
labeling.

Pearls & Pitfalls

Infection and
inflammation

Colonic activity is normal on


Ga-67 and Tc-99m leukocyte
scans but not on In-111
leukocyte scans.
Renal activity may be seen
normally on Ga-67 images
during the 1st 24 hours and on
Tc-99m leukocyte images but
not on In-111 leukocyte scans.

Pearls & Pitfalls

Infection and
inflammation

Focal areas of uptake on Ga-67 scan


are nonspecific and can represent
either tumor or inflammation.
Indium-111 leukocytes have been
reported to localized in some
neoplasms
although
this
is
uncommon occurrence.
On Ga-67 imaging, sarcoidosis and
lymphoma may have similar
appearance. Both may show
mediastinal and lymph node
involvement.
Sarcoidosis
is
suggested by the presence of the
lambda sign and the panda sign.
Abdominal involvement is more
common in lymphoma.

Pearls & Pitfalls

Infection and
inflammation

Diffuse lung uptake on Ga-67


scan is often due to PCP in
patients with AIDS.
Focal uptake in the abdomen on
leukocyte scan may be due to an
abscess or inflammatory bowel
disease (Crohns disease). Activity
in the colon can be seen in
ulcerative colitis or CMV.
Ga-67 is preferred to labeled
leukocyte in the setting of
suspected spinal osteomyelitis.

Infection & inflammation

Gallium 67
citrate

Radiolabeled
leukocytes

Indium-111
oxine
leukocyte

F18 FDG
Tc-99m
HMPAO

Tc-99m
Fenulesomab

Tc-99m
Sulesomab

Radiopharmaceutical

Gallium-67 citrate

Indium-111 oxine
leukocytes

Tc-99m HMPAO
labeled leukocytes

Production of
radionuclide

Cyclotron

Cyclotron

Generator

Decay mode

Electron capture

Electron capture

Isomeric transition

Half life

78hours

67hours

6hours

Photo peak (KeV)

93, 185, 288, 394

173, 247

140

Dose

5 mCi

0.3 0.5 mCi

5 10 mCi

Imaging time

48 hours

24 hours

1 - 4 hours

Normal
biodistribution

Liver > indium, spleen,


marrow, bone,
gastrointestinal

Liver, spleen > Ga 67,


marrow, lung

Liver, spleen, marrow,


GI, GU and lung

Radiopharmaceutical

Gallium-67

Indium-111

Thalium-201

Half life

78 hours

67 hours

72 hours

Photo peak (KeV)

93, 185, 288, 394

173, 247

69-83, 135, 167

Normal Gallium-67 citrate Scan


liver/spleen/GI
marrow

Liver
Spleen
Marrow
Bone
Gastrointestinal

*spleen intense than


liver/ marrow

Liver
Spleen
Marrow
Lung

liver/spleen/ bladder
marrow

(lungs)

Gallium-67 citrate imaging protocol


Patient preparation

No recent barium contrast study

Radiopharmaceutical

-Gallium-67 citrate
-5 mCi
-IV

Instrumentation

Photopeak :20% window, 93, 285 and 300


KeV
Collimator: medium energy

Imaging procedure

24 hr images (optional):site of suspected


infection if early intervention considered.
48 hrs images : whole body imaging
Delayed 72 96 hr images : differentiate
intraabdominal infection or normal bowel
clearance

Indium-111 oxine leukocyte protocol


Patient preparation

Draw 50 ml of blood, radiolabel in vitro

Radiopharmaceutical

In-111 oxine labeled leukocyte


500 Ci

Instrumentation

Photopeak :20% window, 173 and 247


KeV
Collimator: medium energy

Imaging procedure

4 hr images (optional):site of suspected


acute abscess.
24 hrs images : whole body imaging

Tc-99m HMPAO leukocyte protocol


Patient preparation

Wound dressing should be changed prior


imaging
Draw 50 ml blood and radiolabeled in
vitro

Radiopharmaceutical

Tc-99m HMPAO labeled leukocyte in vitro


10 mCi
Given IV

Instrumentation

Imaging procedure

Photopeak :20% window, 140 KeV


Collimator: low energy GP
1 - 2 hr images (mandatory):intra
abdominal imaging
4 hrs images : peripheral skeletal imaging:
osteomyelitis of feet
Whole body imaging

Indications
Gallium-67

In-111 oxine leukocytes

Tc-99m HMPAO leukocytes

Severe leucopenia

Intra abdominal infection

Pediatric patients

Sarcoidosis

Cardiovascular infection

Inflammatory bowel disease

Idiopathic pulmonary fibrosis

Diabetic mid foot and hind


foot osteomyelitis

Osteomyelitis of extremities
including feet in non diabetic

Pulmonary drug reactions


(amiodarone, bleomycin)

Inflammatory bowel disease

Diabetic forefoot osteomyelitis

Pneumocytic carinii

Hip and knee prosthesis

FUO

Orthopedic hardware, prior


fracture or infection

Immunosuppressed patient
with lung infections
Suspected low grade chronic
infections (fungal/protozoa)

Osteomyelitis in different clinical


situations
Normal x ray

Three phase Bone scan

Neonates

Three phase bone scan and if


negative for Tc-99m HMPAO

Suspected osteomyelitis in non Bone scan + leukocyte scan


marrow skeleton (distal
extremities)
Suspected osteomyelitis in
Marrow scan + leukocyte scan
bone marrow containing
skeleton (knees and hips)
Vertebral osteomyelitis
Ga - 67

Osteomyelitis
Type of study

Sensitivity
%

Specificity
%

Three phase bone scan


(normal x ray)

94

95

Three phase bone scan


(underlying disease)

95

33

Gallium 67

81

69

Indium-111 oxine
leukocytes

88

85

Tc-99m HMPAO leukocytes

87

81

MRI

95

87

Infections
Regions

Scans

Diabetic foot

Distal foot Tc-99m HMPAO leukocytes


Mid/hind foot In-111 leukocytes

Vertebral osteomyelitis

Ga-67/bone scan
F18-FDG
Tc-99m infecton

Infected joint prosthesis

In-111 leukocytes/ Tc-99m SC accuracy 90%

Intra abdominal infections

In-111 leukocytes sensitivity 90% @ 24 Hr imaging


Tc-99m HMPAO leukocytes @ 2 Hr imaging
Tc-99m Fanolesomab for acute appendicitis

Inflammatory bowel disease

Tc-99m HMPAO : Crohns small bowel involvement


Ulcerative colitis = colonic involvement without small bowel

Cardiovascular

Prosthetic graft In-111 leukocytes d/t no blood pool

Renal

Ga-67 * increased uptake may due to renal/hepatic failure


and iron overload

Intra cerebral

Tl-201 : increased uptake = malignant, no uptake = infection

FUO

Ga-67
Post op fever=In-111 leukocytes

Malignant external otitis

Ga-67

Pulmonary Ga-67 uptake

Diffuse

Intense

Mild

PCP

CMV
MAI
Treated PCP
Severe PCP
Interstitial pneumonitis

Focal

Nodal

Bacterial
Pneumonia

MAI
TB
Lymphoma

PCP= pneumocystic carinii pneumonia


CMV= cytomegalovirus
MAI= mycobacterium avium intracellular

Gallium-67

Thallium-201

- ve

+ ve

Kaposis
sarcoma

+ ve

- ve

Infection

+ ve

+ ve

Lymphoma

Kaposis sarcoma

Abnormal CXR Pulmonary fibrosis


& Normal
Treated
Gallium-67
sarcoidosis
Inactive TB

PCP
CMV
Normal CXR &
Abnormal
Gallium-67

Sarcoidosis
Pulmonary drug
toxicity
Lymphocytis
interstitial
pneumonitis

Gallium scan
100% sensitivity

Burkitts
lymphoma
Pyogenic acute
osteomyelitis
Pyogenic acute
arthritis

Bleomycin
Amiodarone
Gallium-67 lung
Busulfan
uptake
Nitrofurantoin
associated with
Cyclophosphamide
drugs
Methotrexate

Ga-67 is taken up by the lungs due to


drug toxicity. Which drugs are the
culprits?

Ga-67 is taken up by the lungs due to


drug toxicity. Which drugs are the
culprits?
Bleomycin is the most common. Uptake can
be seen with cytoxin, nitrofurontoin and
amiodarone.

What is the role of Ga-67 in


sarcoidosis?

What is the role of Ga-67 in


sarcoidosis?
Ga-67 lung uptake is a sensitive test for the
diagnosis of active alveolitis of sarcoidosis. Ga67 may be markedly increased in the setting of
a normal CXR in early disease and may be
negative in the setting of an abnormal CXR in
inactive disease.

Which leukocytes are labeled with In111 oxine and Tc-99m HMPAO?

Which leukocytes are labeled with In111 oxine and Tc-99m HMPAO?
In-111 binds to neutrophils, lymphocytes,
monocytes, erythrocytes and platelets.
Tc-99m HMPAO binds to neutrophils.

Which of the following statements is


true regarding In-111 oxine
leukocytes?
A. It is diagnostically useful for evaluating
inflammatory lung disease.
B. It has a high sensitivity for detecting
osteomyelitis in the spine.
C. It should be used when the peripheral
leukocyte count is less than 3000/mm.
D. It is the radiopharmaceutical of choice for
intraabdominal infection.

Which of the following statements is


true regarding In-111 oxine
leukocytes?
FFTT

What is the optimal imaging time for


In-111 leukocytes and Tc-99m HMPAO
leukocytes?

What is the optimal imaging time for


In-111 leukocytes and Tc-99m HMPAO
leukocytes?
In-111 labeled leukocytes are routinely imaged at 24
hours. Imaging at 4 6 hours is less sensitive for
detection of infection. The one exception is
inflammatory bowel disease in which imaging should
be done at 4 hours because intraluminal shedding of
inflammed cells may result in inaccurate localization at
24 hours.
Tc-99m HMPAO leukocytes should be imaged at 1 -2
hours for intraabdominal infection because of biliary
and renal clearance seen by 2 hours. Extra abdominal
infection can be imaged later, usually 4 hours allowing
more time for background clearance.

Regarding osteomyelitis, which of


these statements are true?
A. The three phase bone scan is sensitive test
for diagnosis.
B. The three phase positive scan is specific for
osteomyelitis.
C. A negative flow phase study almost always
rules it out.
D. In patients with prostheses, a bone marrow
scan can be useful to rule out a false positive
In-111 leukocyte study.

Regarding osteomyelitis, which of


these statements are true?
TTTF

Which of the following statement are


true?
A. A negative three phase bone scan exclude
osteomyelitis with high degree of certainty.
B. The specificity of bone scan is poor in patients
with underlying bone disease such as fractures,
orthopedic hardware and neuropathic joints.
C. In-111 oxine and Tc-99m HMPAO leukocytes
have poor specificity for the diagnosis of
osteomyelitis in a patient with a hip prosthesis.
D. Because the three phase bone scan may be
positive in a patient with a Charcots joint, a
radiolabeled leukocyte study should be
performed.

Which of the following statement are


true?
A. TTTF

In the imaging of the lung infection


A. Gallium uptake is highly sensitive for PCP
B. Localized uptake of Gallium is common in PCP
C. Uptake of Indium-111 labeled white cells in the
lungs is relatively non specific for infection
D. Unmatched defects of perfusion are the most
common abnormality seen on V/Q scan in
patients with bacterial infection
E. In patients with TB, only sites of active infection
are Gallium avid

In the imaging of the lung infection


TFTFT

Concerning In-111 white cell scans


A. The white cells must be autologous
B. Lung activity is pathological between 1 and 4
hours after injection
C. If a focus of increased uptake is seen on the
early images, the 24 hour images do not need to
be performed
D. In-111 white cell are more useful in chronic PUO
E. If the patient is on appropriate antibiotics, the
white cell study is unlikely to be positive

Concerning In-111 white cell scans


FTFFF

Concerning Tc-99m HMPAO and In-111


labeled white cell scans
A. Diffuse bowel activity on a Tc-99m HMPAO study at 24
hrs indicates severe colitis
B. On a Tc-99m HMPAO labeled white scan, the typical
appearances of IBD is intense activity at 1 hour which
fades with time
C. Rectal disease can be distinguished from bladder
activity on a Tc-99m HMPAO
D. Activity is seen in the bowel in IBD because the
labeled WBCs fix in the inflammatory bowel wall
E. An abscess that communicates with the bowel can be
distinguish from one that does not

Concerning Tc-99m HMPAO and In-111


labeled white cell scans
FFTFT

Concerning the imaging of the


immunosuppressed patient
A. In-111 labeled WBCs are sensitive as Gallium
at detecting PCP
B. Tc-99m DTPA clearance is more useful than
Gallium scanning in the following up of PCP
C. Gallium uptake in the colon is highly
suspicious for acute colitis
D. Kaposis lesions take up Tl-201
E. Active MAI infection in HIV positive patients
has specific appearance on Gallium scan

Concerning the imaging of the


immunosuppressed patient
FTFTF

The source of lectoferrin


A.
B.
C.
D.
E.

Lymphocytes
Leucocytes
Bacteria
Fungus
Tissue macrophage

The source of lectoferrin


FTFFF

The source of siderophores


A.
B.
C.
D.
E.

Lymphocytes
Leucocytes
Bacteria
Fungus
Tissue macrophage

The source of siderophores


FFTFF

Lactoferrin is physiologically produced


by certain organs that localize gallium
A.
B.
C.
D.
E.

Lacrimal glands
Salivary glands
Lactating breast
Bone
Liver

Lactoferrin is physiologically produced


by certain organs that localize gallium
TTTFF

Sarcoidosis
1. Patient A: lambda sign (hilar and paratracheal
nodal uptake). patient B: diffuse pulmonary
uptake.
2. Classic panda sign.
3. Either A or B. The panda sign can be seen at
any stage of the disease.
4. Sarcoidosis. Ga-67 scan is used to confirm the
clinical diagnosis and differentiate active
alveolitis from inactive fibrosis.

Intraabdominal abscess
1. Very increased uptake in the right lower quadrant
strongly suggests an intraabdominal abscess. Tumor
cannot be excluded.
2. Increased vascular permeability, bacterial uptake and
binding to leukocytes play a role, however, binding to
lactoferrin of degranulated neutrophils at the site of
infection is probably the primary mechanism.
3. Photopeak at 91 to 93, 185, 300 and 394 keV. The
lower three photopeaks are used for imaging. A
medium energy collimator should be used.
4. The recommended adult dose of Ga-67 is 5 mCi, half
life is 78 hours, imaging time is at 48 hours is routine.
If abscess suspected, imaging at 6 to 24 hours may
provide an early diagnosis.

Tc-99m HMPAO and osteomyelitis


1. A negative bone scan rules out osteomyelitis with a
high degree of certainty.
2. A bone marrow can improve the specificity of a
leukocyte study if there is displaced normal marrow.
This is most helpful in the hips and knees.
3. In-111 leukocytes can make same diagnosis. The
superior imaging resolution of Tc-99m HMPAO often
better differentiates soft tissue and bone infection.
Ga-67 is less specific because increased uptake occurs
with bone remodeling from any cause.
4. Patient A: soft tissue uptake. No bone uptake rules
out osteomyelitis, consistent cellulitis. Patient B: soft
tissue and bone localization consistent with
osteomyelitis.

Tc-99m HMPAO leukocytes free Tc99m pertechnetate


1. Activity in the salivary glands, thyroid, stomach,
bowel and bladder.
2. Free Tc-99m pertechnetate.
3. Gastrointestinal bleeding, swallowed leukocytes
from oropharyngeal, esophageal or lung
inflammation/ infection, accessory spleen,
uninfected postoperative surgical wounds,
intestinal stomas.
4. Ga-67 = 48 hours, In-111 oxine leukocytes = 24
hours, Tc-99m HMPAO = 1 to 4 hours.

Intraabdominal abscess
1. Tc-99m HMPAO binds only to neutrophils. In-111 oxine
binds to mixed leukocytes.
2. Normal bone marrow distribution of the radiolabeled
leukocytes.
3. Abdominal imaging: Tc-99m HMPAO images are acquired
at 1 to 2 hours because hepatobiliary, intestinal and renal
clearance
occurs
subsequently,
complicating
interpretation. Extremity imaging: 2 to 6 hours. In-111
leukocytes: 24 hours. Four hour imaging for inflammatory
bowel disease because sloughing of intestinal mucosa
leukocytes may occur and 24 hour may be misleading.
4. Infection in the right lower quadrant overlying the
sacroiliac joint in the anterior view and lateral right of the
spine. The patient had a perforated appendix.

Pericarditis Ga-67
1. Abnormal uptake in myocardium, pericardium or both
consistent with pericarditis or myocarditis. The
patient developed a pericardial fraction rub the day
after the Ga-67 scan. Incidental note of normal liver
uptake and transverse and left colon clearance.
2. Ga-67 shows uptake not only with inflammation and
infection, but also in tumors that can sometimes be
the cause of persistent fever e.g.. lymphoma.
3. Large bowel = 4.5 rads.
4. Most pulmonary inflammatory and infectious
diseases. Uptake is nonspecific, although the pattern
of uptake and the clinical setting, e.g. AIDS may be
helpful in determine the differential diagnosis.

Right ileum osteomyelitis


1. In-111 oxine or Tc-99m HMPAO leukocytes,
Ga-67 could also be used. The intense spleen
uptake is consistent with a radiolabeled
leukocyte study. In-111 oxine leukocytes
were used. The image resolution is poor
compared with Tc-99m leukocytes.
2. Spleen, 15 20 rads.
3. Abnormal focal uptake in right groin and right
ileum, indicating osteomyelitis.
4. 173, 247 keV. Physical half life is 77 hours.

Osteomyelitis of spine
1. Bone scan shows increased uptake of the T11
vertebrae. In-111 oxine leukocyte shows
decreased uptake in the same region. Ga-67
shows increased uptake that matches the bone
scan in relative intensity.
2. Osteomyelitis fracture, infarction, metastasis,
orthopedic hardware, surgical defect, Pagets
disease, radiation therapy.
3. Osteomyelitis in this clinical setting. Many of the
diseases listed can be excluded by history and
radiographs.
4. As high as 40%.

FDG PET paraspinal infection


1. Intense uptake involving the retroperitoneum
anterior to the recent lumbar fusion surgery.
2. Consistent with postoperative infection.
3. Yes. Leukocytes and macrophages utilize
glucose.
4. All positron emitters have 511 keV
photopeak. F-18, C-11, N-13 , O15.

Knee athroplasty TRO infection


1. Increased blood flow in the right knee.
2. Increased uptake in the proximal tibia on
both.
3. Serves as a template for normal marrow
distribution.
4. Negative for infected prosthesis.

Skeletal imaging

Pearls & Pitfalls

Skeletal imaging

Common indications for bone


scans include evaluation of
primary osseous or metastatic
neoplasm, avascular necrosis,
trauma, infection and less
commonly, arthritis or RSD.
When
bone
scans
are
performed
with
Tc-99m
diphosphonate agents, normal
skeletal activity should be
reasonably symmetric left and
right sites.

Pearls & Pitfalls

Skeletal imaging

Look at the kidney activity for


potential abnormalities.
A
large amount of activity of
diffuse soft tissue activity
remaining at 4 hours is
frequently due to renal
insufficiency.
Activity in the lower cervical
region is frequently due to
benign
causes
such
as
degenerative changes.

Pearls & Pitfalls

Skeletal imaging

A lesion that is hot all three


phases of a bone scan can be
osteomyelitis bit also may be
an
acute
fracture,
hypervascular
tumor,
neuropathic joint or RSD.
A flare phenomenon is most
likely seen within 1 to 3
months therapy completion.

Pearls & Pitfalls

Skeletal imaging

Pagets disease is commonly


seen as intense activity in the
skull, femur, vertebral body or
half of the pelvis. It is usually
polyostotic but may be
monostotic and may cause
bowing of a femur.
Good visualization
of the
bones and not the kidneys may
indicate a superscan due to
diffuse
metastases
or
hyperparathyroidism.

Pearls & Pitfalls

Skeletal imaging

Focal hot lesions in multiple


adjacent ribs are essentially
always due to fracture.
Long lesions running along the
length of a rib are not usually
fractures.
Multiple
sequential
cold
vertebral bodies are almost due
to radiation therapy.
Tumors that commonly cause
cold (photopenic) metastatic
lesions including kidney, lung,
thyroid and breast tumors.

Pearls & Pitfalls

Skeletal imaging

Stress fractures usually occur


in the pelvis and below the
knees, they can be seen as
focal or fusiform primary
cortical activity.
Bilaterally increased activity
along the cortex of the tibias
may be due to shin splints,
hypertrophic osteoarthropathy
or periosteal reaction.

Pearls & Pitfalls

Skeletal imaging

A loose hip prosthesis is


suggested by activity at the tip
and near the lesser trochanter.
An infected prosthesis usually
has activity all along the length of
the shaft.
Postoperative activity around a
cemented
prosthesis
can
normally persist for 6 months to
1 year and activity around a
noncemented prosthesis can
normally persist for 2 to 3 years.

Pearls & Pitfalls

Skeletal imaging

Diffuse liver activity on a bone


scan is probably due to hepatic
necrosis
or
radiopharmaceutical problem.
Focal liver activity is often due
to metastasis from colon,
breast, ovary or lung.
Diffused splenic activity due to
splenic infarction.
Diffused renal parenchymal
activity is probably due to
chemotherapy.

Pearls & Pitfalls

Skeletal imaging

Increased activity in two


adjacent vertebral bodies in
the absence of compression
fractures suggests diskitis
especially in a child.
Osteomyelitis, acute fractures,
vascular tumors such as
Ewings sarcoma and RSD are
hot on angiographic, blood
pool and delayed bone scan
images.

Pearls & Pitfalls

Skeletal imaging

Cellulitis is hot on the 1st two


phases but fades on delayed
images.
Shin splints have normal
angiographic and blood pool
images but are hot on delayed
views in the posteromedial
aspect of the tibias.
Osteomyelitis usually does not
cross joints.
Increased activity seen on both
sides of a joint is more likely due
to septic arthritis.

Bone pain for bone scan


Metastatic tumors
Benign bone tumor : osteoid osteoma
Trauma
Avascular necrosis
Infection
Osteomalacia
Pagets

Reduced hydrolyzed
technetium

Tc-99m
pertechnitate
Free pertechnetate
preparation,
there are 3 form
of technetium Chelated technetium

Liver
uptake in
bone scan

Reduced hydrolyzed
technetium
Excess amount of
aluminum in Tc-99m
MDP preparation
Excess amount of
stannous ion in Tc99m MDP
preparation

Excess amount of
aluminum in Tc-99m SC
preparation

Lungs uptake
Thyroid

Tc-99m pertechnetate

Gastric mucosa

Metastatic bone disease

Bone tumors
Benign

Malignant

Osteoid osteoma

Osteosarcoma

Giant cell tumor

Ewings sarcoma

Fibrous dysplasia

Chondrosarcoma

Bilateral
increased
uptake of renal
in bone scan

Urinary tract
obstruction
Acute tubular
necrosis
Chemotherapy
Radiation nephritis
Thalassemia
Hypercalcemia
Nephrocalcinosis
Nephrotoxic
antibiotic

Bilateral
decreased
uptake of
renal in
bone scan
superscan

Tumors

Non tumor

Prostate

Renal failure

Lungs
carcinoma
Breast
carcinoma
Bladder
carcinoma

Nephrectomy

Lymphoma

Hyperparathyroidism
Osteomalacia
Fibrous dysplasia
Pagets disease

Renal osteodystrophy

Breast carcinoma

Tumors that
metastasize to
bone

Lungs carcinoma
Prostate carcinoma
Lymphoma
Thyroid carcinoma
Renal carcinoma
Neuroblastoma

Multiple myeloma

Tumors falsely
normal in bone
scan

Severe anaplastic
tumors

Lytic lesions

Metal artifact

Differential
diagnosis of
cold defect in
bone scan

Radiation changes
Barium in bowel
Early avascular necrosis
Multiple myeloma
Benign tumor
Cysts
Osseous metastasis
Tumor marrow
involvement

Soft tissue calcification

Dystrophic
calcification

Calcification of
dying or dead
tissue

Metastatic
calcification

Normal tissue
with
hypercalcemia

Heterotrophic
Bone
formation

-infection
-osteomyelitis
-cellulitis
-thrombophlebitis
-DVT
-osteosarcoma
-osteochondroma

Calcinosis
cutis

-calcinosis
cutis universalis
-calcinosis
cutis circumcripta
-calciphylaxis
-rabdhomyolysis

Soft tissue accumulation of


diphosphonate
Infarct
Amyloid
Muscle: myositis ossificans
Fibroids
Tumors: meningioma/neuroblastoma
Systemic sclerosis

Histocytosis X
Disease in which there are focal accumulation
of macrophages in various organs including
bones

Triad:
1. Letterer-Siwe disease
2. Hand-Schuller-Cristian disease
3. Eosinophilic granuloma of bone

Fibrous dysplasia

Bone dysplasia's
a/w skeletal
tracer uptake

Osteogenic imperfecta
Albers-Schenberg
Disease (Marble Bones)
Englemanns disease
Malorheosthosis
Ribbings disease

Fibrous dysplasia

Osteogenic imperfecta

Osteopetrosis
Albers-Schenberg Disease (Marble Bones)

Englemanns disease

Ribbings disease
(multiple diaphyseal sclerosis)

Melorheostosis

Metabolic bone
disease

Pagets disease
(osteitis deformans)
Osteoporosis
osteomalacia &
Rickets
Hyperparathyroidism
Renal osteodystrophy
Hypertrophic
osteoathroplathy

Metabolic bone disease


Disease
Cause of +ve
bone scan

Renal
osteodystrophy

Primary
hyperparathyroidism

Osteomalacia

Hyperparathyroid

Hyperparathyroid

Hyperparathyroid

Metabolic features:

Differentiating
features

Metabolic features:
No tracer in bladder

Metabolic features:
Uncommon: brown
tumor & ectopic
calcification

pseudofractures

Aluminum
induced
osteomalacia

Osteoporosis

Fracture

Long bone
uptake

Intense linear
uptake at site of
vertebral
fracture

High
background
activity

May be
low/patchy
uptake at axial
skeletal

Focal skeletal phase uptake


At the tip of the hardware

Increased flow and


intense uptake around
the prosthesis stem

Hip joint prosthesis


Loosening

Infection

Blood pool image is Blood pool image is


normal
increased vascularity
Discrete focus of Markedly increased
tracer
increased tracer
uptake
in
uptake at the tip of delayed image
femoral prosthesis in
delayed image

Stress fracture

Fatigue fracture

Insufficient fracture

Cause by repeated
abnormal fracture
on normal bone

Resulting from
normal stress on
abnormal bone

Reflex sympathetic dystrophy


syndrome
Poorly understood
Often forgotten in clinical practice
Symptoms: pain/tender, swelling and dystrophic skin
changes
Other terms:
Causalgia
Acute atrophy of bone
Sudecks atrophy
Post traumatic osteoporosis
Shoulder hand syndrome

Legg-Calve-Perthes

FORM OF OSTEOCHONDRITIS DISEASES


CAUSED BY AN INFARCT IN THE CAPITAL
FEMORAL EPIPHYSIS

4. Bone scan
TTFTT

7. Indication for bone scan


TTTTT

8. Bone scan
TFFTT

15. Abnormal bone scan uptake in


kidneys
TTTTT

20. Cause of superscan


FFFTT

Malignancy
A. In multiple myeloma the MDP bone scan is usually
abnormal
B. Increased MDP bone scan uptake into metastatic
deposits 2 months after chemotherapy indicates
failure of response and progression of disease
C. A solitary rib hot spot in a cancer patient usually
represent benign disease
D. A rib hot spot which is present over a year is likely to
represent a simple healing fracture
E. A solitary area of abnormal uptake is a patient with a
known primary cancer with normal plain film
appearances is likely to be benign

Malignancy
TFFFT

Bone scanning in malignancy


A. A malignant superscan requires more time to
acquire the image
B. It is possible to distinguish a malignant
superscan from one of metabolic bone disease
C. A superscan characteristically shows increased
bone and renal uptake of MDP
D. Uptake of MDP into malignant bone disease
relies solely on osteoblastic activity
E. It is not possible to distinguish benign from
malignant vertebral collapse

Bone scanning in malignancy


FTFFT

Bone scan in Pagets disease


A. A bone scan may be normal in Pagets disease
B. A bone scan is a method of choice to detect
sarcomatous change in Pagets disease
C. Pagets very rarely only involves a single bone
D. The appearance of new lesions in a patient with
established Pagets disease is a common
recognized finding
E. A bone scan is the method of choice in assessing
response to therapy in Pagets disease

Bone scan in Pagets disease


TFFFF

Bone scan in malignant disease


A. Patients with a known primary cancer and single
skeletal hot spot have a roughly equal chance that it is
benign or malignant
B. Patients with a known primary cancer and a single
vertebral hot spot are unlikely to have malignant
disease
C. A bone scan is a useful routine staging procedure of
prostate carcinoma
D. A bone scan is a useful routine staging procedure for
all patients with carcinoma of the cervix
E. A bone scan is a useful routine staging procedure for
patients with stage III and IV breast carcinoma

Bone scan in malignant disease


TFTFT

Bone scan in trauma


A. Tibial stress fractures are indistinguishable from
shin splint on bone scan
B. Compartmental syndromes are indistinguishable
from shin splint on bone scan
C. Reflex sympathetic dystrophy syndrome may
show reduced uptake of MDP
D. 50% of non pathological rib fractures heal by
one year
E. In talar coalition increased activity is usually at
the site of coalition

Bone scan in trauma


FFTFF

Bone scan in avascular necrosis


A. Septic arthritis of the hip in children may cause absent
uptake of MDP into the femoral head
B. Osteomyelitis and bone infarction are easily
distinguishable in sickle cell disease
C. Decreased MDP accumulation lasts for about 6 weeks
following bone infarction in sickle cell disease
D. In chronic Perthes disease bone scan has a sensitivity
and specificity of over 90%
E. Bone scan a few weeks following femora; neck
fracture that is associated with higher uptake in the
femoral neck indicates a favorable prognosis

Bone scan in avascular necrosis


TFFFT

Bone scan in infection


A. Labeled WBC scanning is the method of choice in
diagnosing osteomyelitis in HIV patients
B. Tc-99m HMPAO labeled WBC are preferable to In-111
labeled in detection of acute osteomyelitis of
extremities
C. Tc-99m HMPAO labeled are the method of choice in
diagnosing infection in children
D. It is not possible to carry out WBC scanning in
severely neutropenic patients
E. WBC scanning is the method of choice in
differentiating acute infarct from osteomyelitis in
sickle cell disease

Bone scan in infection


FTTFF

Increased uptake of MDP is seen in


A.
B.
C.
D.
E.

Bone island
Osteopoikilosis
Fibrous dysplasia
Melorheostosis
Eosinophilic granuloma

Increased uptake of MDP is seen in


FFTTT

Primary bone tumors


A. Vertebral hemangiomata may cause reduced or
increased uptake of MDP into a vertebral body
B. It is possible to differentiate benign from malignant
bone tumors from the degree of uptake of MDP
C. Bone scan is a good method for assessing the extent
of involvement of a bone in osteogenic sarcoma
D. Bone scan is a reliable method of detecting soft tissue
metastasis e.g.. lung in osteogenic sarcoma
E. Bone scan is a useful preliminary staging procedure in
Ewing's sarcoma

Primary bone tumors


TFFFT

Metabolic bone disease


A. The tie sign is a recognized feature
B. Reduced uptake of MDP into the diaphyses of
long bones is a recognized feature
C. Osteomalacia causes a reduction in uptake of
MDP as there is reduction of bone mass
D. Aluminum osteomalacia in hemodialysis
patients causes a metabolic superscan in
advanced disease
E. Treatment with biphosphonates may cause a
reduction in MDP uptake into the skeleton

Metabolic bone disease


TFFFT

Infection
A. Both infection and loosening of a hip prosthesis will show
both increased vascularity and uptake on delayed images
B. Increased activity in relation to a knee prosthesis one year
after surgery nearly always signifies infection or loosening
C. Cellulitis in the limb may cause increased uptake into bone
on a bone scan
D. Uptake of Gallium into a bone at the site of increased uptake
on a bone scan nearly always signifies infection
E. Bone scan are less sensitive in detecting osteomyelitis in
neonates because of their small size

Infection
FFTFF

Photon deficient bone scan lesions


may be found in
A. Metastatic disease
B. Legg-Calve-Perthes disease
C. Infarct of sickle cell disease

Photon deficient bone scan lesions


may be found in
TTT

Pulmonary, gastric, and enhanced


kidney activity on a bone scan is often
seen in patients with
A.
B.
C.
D.
E.

Myelosclerosis
Histocytosis
Metastases from osteogenic sarcoma
Hyperparathyroidism
Hyperthyroidism

Pulmonary, gastric, and enhanced


kidney activity on a bone scan is often
seen in patients with
FFFTF

Spleenic uptake in bone scan is most


often seen in
A.
B.
C.
D.
E.

Cirrhosis with portal hypertension


Hypersplenism
Sickle cell disease
Lymphoma
Splenic abscess

Spleenic uptake in bone scan is most


often seen in
FFTFF

The following neoplasm does not


frequently metastasize to bone
A.
B.
C.
D.
E.

Ovary
Prostate
Lung
Lymphoma
Breast

The following neoplasm does not


frequently metastasize to bone
TFFFF

Which of the following commonly


cause photopenic lesions on a bone
scan
A.
B.
C.
D.
E.

Malignant melanoma
Ewings sarcoma
Multiple myeloma
Hypernephroma
Colon carcinoma

Which of the following commonly


cause photopenic lesions on a bone
scan
FFTTF

What are the potential impurities in


Tc-99m labeled diphosphonate
compounds, based on their
biodistribution.

What are the potential impurities in


Tc-99m labeled diphosphonate
compounds, based on their
biodistribution.
Activity in the oropharynx, thyroid gland and
stomach suggestive of free pertechnetate.
Activity in the liver suggests of colloidal
impurity.
Rarely, activity is seen in the gut, the result of
excretion of activity through the biliary
system.

What is the distribution of metastatic


deposit from epithelial primary
malignancies in the skeleton?

What is the distribution of metastatic


deposit from epithelial primary
malignancies in the skeleton?
80% at axial skeleton (spine, ribs, pelvis and
sternum)
10% at skull
10% at long bones

What factors distinguish a superscan


resulting from metastatic disease or
from metabolic disease?

What factors distinguish a superscan


resulting from metastatic disease or
from metabolic disease?
Metastatic disease: uptake seen at axial
skeleton and proximal parts of the femur and
humeri.
Metabolic disease: entire skeleton is typically
effected with increased uptake seen at
extremities as well as in the axial skeleton.

What is the mechanism of the flare


phenomenon?

What is the mechanism of the flare


phenomenon?
In some patients treated with chemotherapy
for metastatic disease, regression of the
tumor burden is associated with increased
osteoblastic activity, presumably caused by
skeletal healing in response to chemotherapy.
This can appear on bone scan as a paradoxical
increased worsening tracer uptake which up
to 6 months after therapy.

What factors contribute to prolonged


fracture positivity on bone scan?

What factors contribute to prolonged


fracture positivity on bone scan?
Displaced or comminuted fractures and
fractures involving joints tend to have
prolonged tracer uptake. Elderly patients have
delayed healing.

What is the scintigraphy pattern of


osteomyelitis in children

What is the scintigraphy pattern of


osteomyelitis in children
Hot focus in metaphyseal region

What is scintigraphy finding in RSD

What is scintigraphy finding in RSD


Increased blood flow and diffusely increased
periarticular activity in delayed images

What is the explanation for the


abnormal bone scan in RSD

Loss of sympathetic stimulation


Results in vasodilatation
Increased blood flow and bone turnover
Synovitis contributing diffusely increased
periarticular activity

What is shin splint

What is shin splint


Periostitis along posteromedial angle of tibia
Tear of tibialis posterior muscle or soleus
muscle tendon complex

What is scintigraphic appearance in


shin splint

What is scintigraphic appearance in


shin splint
Longitudinal wavy irregular uptake in the
periosteal aspect of the posteromedial tibia

What is the three phase bone scan


finding in the acute phase of AVN

What is the three phase bone scan


finding in the acute phase of AVN
Diminish flow and cold defect in delayed view

What is the finding in the chronic


phase of AVN

What is the finding in the chronic


phase of AVN
Increased or normal flow with increased
uptake

How is the bone marrow scan used in


AVN

How is the bone marrow scan used in


AVN

How is the bone marrow scan used in


AVN
A cold defect in the region of increased uptake
in bone scan indicates the bone scan
abnormality is due to infarct

What are the scintigraphic finding of


cellulitis

What are the scintigraphic finding of


cellulitis
Increased flow and blood pool images in the
first and second phase
Delayed phase normal uptake

What are the scintigraphic finding of


osteomyelitis

What are the scintigraphic finding of


osteomyelitis
Increased flow and blood pool images in the
first and second phase
Increased uptake in delayed phase

Name four primary tumors causing


photopenic metastases

Name four primary tumors causing


photopenic metastases

Renal
Thyroid
Multiple myeloma
Lung

Describe the scintigraphic pattern in


Pagets

Describe the scintigraphic pattern in


Pagets
Expansile or thickening of the bone with sharp
demarcation of normal area

How does Pagets affect the bone

How does Pagets affect the bone


Increased bone resorption accompanied by
bone formation

What is the scintigraphic finding of


radiation osteitis in bone scan

What is the scintigraphic finding of


radiation osteitis in bone scan
Increased activity in the first few weeks,
followed by defect

What is the scintigraphic pattern of


early fracture in bone scan

What is the scintigraphic pattern of


early fracture in bone scan
Cold defect in the first few hours

What are the reasons of cold defect in


first few hours of fracture in bone scan

What are the reasons of cold defect in


first few hours of fracture in bone scan
Rupture of vascular supply from fracture and
too early for reactive bone formation

How soon will fracture become


positive and what is the time course in
bone scan

How soon will fracture become


positive and what is the time course in
bone scan
Cold defect in the first few hours
Increasing activity up to few weeks or months
Then tapering off until resolved

Stress fractures
1. Increased activity in a linear pattern along the
posterior and medial aspect of both mid tibias.
2. Shin splints
3. B , focal ovoid activity posteromedial right tibia
at the junction of the proximal 2/3 and distal
1/3. C, focal fusiform activity posteromedially in
the right proximal tibia and linear activity along
the posteromedial left tibia proximally and more
prominently distally.
4. B, stress fracture. C, stress fracture and shin
splints

Pagets disease
1. Abnormal increased uptake in the entire left femur,
which appears bowed and widened and the distal
third of the left tibia, which tapers proximally.
2. A sharp leading edge, referred to as flame shaped
or :blade of grass, may be demonstrated on the lytic
phase on radiograph and bone scan.
3. Pagets
disease,
fibrous
dysplasia,
chronic
osteomyelitis, primary bone tumors.
4. High output congestive heart failure may occur. Once
believed to be the result of arteriovenous shunting
within the bone lesion, now hyperemia and increased
blood flow through the lesion and not shunting are
likely the causes.

Rib fractures
1. Patient A; focal increased uptake in multiple ribs
posterolaterally
and
the
costovertebral
junctions. Patient B; increased vertical linear
uptake in the sternum from the manubrium to
the xiphoid.
2. The uptake in adjacent (patient A) and the
vertical uptake in the sternum both have a
geometric and characteristic pattern.
3. Trauma or surgery
4. Patient A; multiple rib fracture. Patient B;
median sternnotomy for coronary artery bypass
grafting (CABG)

Osteomyelitis
1. First phase; arterial blood flow to the bone.
Second phase; blood pool. Third phase; delayed
phase or bone uptake at 3 hours after injection.
All three phases are typically focally increased
with osteomyelitis. With cellulitis only two
phases are positive.
2. Increased flow, blood pool and delayed uptake
to the left first digit distal phalanx.
3. Consistent with osteomyelitis of the digit.
4. Sensitivity and specificity 95% if the radiograph
is normal or has only suggestive changes of
osteomyelitis.

Abnormal breast uptake


1. Nonuniform abnormal soft tissue uptake
exists in the soft tissue overlying the chest,
likely the right breast.
2. Breast ca, aseptic mastitis or radiation
therapy.
3. Breast examination, mammography and
possible biopsy.
4. To determine whether breast cancer bone
metastases are present.

Extra renal pelvis


1. Solitary right kidney with prominent renal
pelvis. Incidental uptake at antecubital
injection site
2. Ureteropelvic junction obstruction or
obstruction secondary to other processes,
extrarenal pelvis.
3. The renal pelvis has drained and the kidney is
now seen inferomedial to its prior location
4. Image B was taken with the patient erect,
leading to gravity drainage of an extra renal
pelvis. The kidney is mobile (ptotic)

Lymphedema
1. The soft tissues of the left arm enlarged and
show abnormal increased soft tissue activity,
the left anterior ribs are uniformly more
intense than the right.
2. Venous or lymphatic obstruction, soft tissue
neoplasm, soft tissue injury.
3. Breast cancer.
4. Lymphedema secondary to axillary lymph
node dissection and left mastectomy

Renal position
1. The right kidney is not in the renal fossa.
Nonuniform activity is noted in the right
sacroiliac region, which extends beyond the
expected superior margin of the bone.
2. Congenital renal anomaly, pelvic kidney.
3. Anomalies of number
(supernumerary
kidney), position (malrotation) and fusion
(horseshoe)
4. Yes. Ureteropelvic junction obstruction,
vesicoureteral reflux, decrease function and
increase risk of trauma.

Metastatic prostate cancer


1. Abnormal focal uptake throughout the axial
and
appendicular
skeleton
strongly
suggestive of metastatic disease.
2. Greater than 20 ng/ml. the prevalence of
bone scan evidence od metastases is < 1%.
3. Multiple myeloma, thyroid ca,, renal cell ca
and lymphoma
4. CT and MRI have poor sensitivity for
detection of prostate cancer soft tissue/nodal
metastases.

Fibrous dysplasia
1. A , shows increased uptake in the entire
mandible. B, shows intense increased uptake
in the mandible and maxilla which appear
deformed and overgrown.
2. Check the rest of the bone scan for other
sites
3. Fibrous dysplasia, cherubism.
4. Fibrous dysplasia.

Sacral insufficiency fracture


1. Patient A has increased blood pooling and
delayed uptake bilaterally in the region of the
sacroiliac joints and across the sacrum (H
pattern). The bone scan for patient B.
2. Both of these are diagnostic of sacral
insufficiency fractures.
3. Proximal femur, wrist and proximal humerus
4. Osteoporosis

Heterotopic ossification
1. Intense activity is seen overlying the right
acetabulum with a separate area of uptake
overlying the proximal right femur.
2. Urinary contamination; fracture with exuberant
callus; heterotopic ossifican or myositis
ossificans; soft tissue injury (contusion).
3. If urinary contamination is suspected; remove
clothing and overlying bed sheets; wash the
patients skin in the area of suspected
contamination.
4. Heterotopic ossification.

Superscan
1. Increased tracer in the large majority of the
visualized
bones,
with
nonuniform
involvement in both femurs, both humeri and
skull.
2. Soft tissue and GI tract.
3. The kidneys are not visualized, but faint
activity is seen in the urinary bladder. Little
soft tissue activity is seen.
4. Superscan.

hyperparathyroidism
1. Abnormal diffuse uptake in the lungs and stomach.
Poor visualization of small kidneys and bladder,
increased uptake in the shoulders, hips, knees and
ankle.
2. Hyperparathyroidism, metastatic calcification caused
by hypercalcemia, renal failure or metabolic bone
disease.
3. This particular pattern of metastatic calcification is
characteristic of long standing hyperparathyroidism.
4. Free Tc-99m pertechnetate has gastric, thyroid and
salivary gland uptake. The latter two are not seen in
this patient, who also shows large uptake.

Reflex sympathetic dystrophy


1. Three phase study demonstrate abnormal increased
blood flow and blood pool of the distal right upper
extremity. The delayed bone phase shows increased
activity in the bones in the same distribution with a
striking increased in periarticular activity causing the
joints to stand out.
2. RSD. Disuse of a limb of new onset e.g.. recent stroke
or immobilization by orthopedic cast or splint.
3. Shoulder hand syndrome, a frequently encountered
form of RSD
4. Neurogenic origin with loss of sympathetic autonomic
tone is the generally accepted explanation, although
not firmly establish.

Myocardial uptake
1. Horseshoe pattern of uptake in the anterior
chest that does not correspond to normal
bony anatomy and therefore is most likely
abnormal soft tissue uptake.
2. Cardiac uptake either the myocardium or
pericardium.
3. Tc-99m pyrophosphate
4. Idiopathic or secondary cardiomyopathy e.g..
due to cardiotoxic drugs, myocarditis or
pericarditis.

Avascular necrosis of femoral heads


1. Nonvisualization of the left ilium, leg length
discrepancy as a result of high riding left hip
and increased uptake both femoral heads,
left worse than right.
2. The ilium has been surgically resected.
Femoral heads; avascular necrosis, fracture,
osteotomies,
slipped capital femoral
epiphyses in the appropriate group.
3. Trauma, steroid, sickle cell disease, chronic
renal disease, alcoholism.
4. Humeral head and tarsal navicular.

Cardiac

Pearls & Pitfalls

Myocardial
perfusion imaging

The common indication for


MPI is to determine whether
there is normal perfusion,
ischemia or infarction.
Tl-201 photon energy (69 81
keV)
Myocardial uptake of Tl-201
chloride
and
Tc-99m
sestamibi/tetrofosmin
is
proportional to regional blood
flow and requires cell viability.

Pearls & Pitfalls

Myocardial
perfusion imaging

Thallium is actively taken up by


Na/K pump in the cells.
Tc-99m sestamibi/tetrofosmin
passively diffuse across the
membrane and localize in
cytoplasmic mitochondria.
Thallium redistributes over
time, so post stress images
must be obtained immediately
after injection.

Pearls & Pitfalls

Myocardial
perfusion imaging

Tc-99m sestamibi/tetrofosmin
do not redistribute and best
images are made 30 to 90
minutes after injection to allow
clearance
of
interfering
background activity from the
liver but before the activity
reaches the transverse colon.
Infarcts
and
hibernating
myocardium produce perfusion
defects on the rest images.

Pearls & Pitfalls

Myocardial
perfusion imaging

Stress images are needed to


elucidate ischemia with lesser
degree of stenosis (50 90%).
Stenosis less than 50% in diameter
are not detected at rest and are
variably diagnosed with exercise
imaging.
Stress may be physical or
pharmacologic.
Physical
stress
should
be
discontinued
with
patient
exhaustion, claudication, severe
angina or hypotension, arrhythmia
or severe ECG changes.

Pearls & Pitfalls

Myocardial
perfusion imaging

Dipyridamole and adenosine


are used instead of physical
stress to dilate normal
coronary arteries.
Vessels in ischemic areas are
already maximally dilated and
do not dilate further.
The adverse effect of the
dipyridamole are reversed with
100
to
200
mg
IV
aminophylline.

Pearls & Pitfalls

Myocardial
perfusion imaging

Dipyridamole and adenosine


should not be used in patients
with asthma.
Dobutamine is inotropic and
chronotropic and is used for
pharmacologic stress.
Adverse effects may be reversed
by using short acting beta
blocker.
The most common cause of false
negative is submaximal stress.
The most common cause of false
positive is artifact.

Pearls & Pitfalls

Myocardial
perfusion imaging

The myocardium normally thins


at the apex, membranous
septum and base of inferior wall.
A high degree of lung activity on
exercise Thallium images (>50%
of peak myocardial activity) is
related to poor prognosis, more
extensive disease and LV
dysfunction on exercise.
On Tc-99m sestamibi scans, lung
activity is uncommon.

Pearls & Pitfalls

Myocardial
perfusion imaging

Transient LV dilatation with


exercise may be related to
subendocardial ischemia and less
often to real LV dilatation. It is
sign of poor prognosis.
With adequate exercise, the
sensitivity and specificity of MPI
are about 85%.
An occluded coronary artery with
adequate collaterals or balanced
three vessels disease can
produce an apparently normal
scan

Pearls & Pitfalls

Myocardial
perfusion imaging

Real defects on MPI should be


seen on at least 2 views (e.g..,
short and vertical or horizontal
long axes)
LBBB may produce myocardial
perfusion
findings
indistinguishable from stress
induced
reversible
septal
ischemia.
A defect seen at the base of the
heart is frequently artifactual
unless it extends to the apex.

Pearls & Pitfalls

Myocardial
perfusion imaging

Not all fixed defects are infarcts.


They may also be soft tissue
attenuation artifacts, hibernating
or
repetitive
stunned
myocardium.
Hybernating myocardium is a
chronically hypoperfused area
that has reduced cellular
metabolism.
This
area
has
decreased
contractility
and
revascularization
is
usually
needed.

Pearls & Pitfalls

Myocardial
perfusion imaging

Stunned myocardium is due to


an acute occlusion with
relatively rapid perfusion.
These area have normal or
near normal perfusion and
decreased contractility and
revascularization is not usually
needed.
LV walls that diverged towards
the
apex
should
raise
suspicious of a LV aneurysm.

Pearls & Pitfalls

Myocardial
perfusion imaging

Diaphragmatic attenuation looks


like an inferior wall defect but it
goes away on prone images.
Relatively intense GI activity
adjacent to the inferior wall can
cause an apparent inferior
myocardial defect on SPECT
studies as a
result of a
reconstruction artifact.
Breast attenuation usually causes
an apparent defect in the
anterior or lateral LV wall.

Pearls & Pitfalls

Myocardial
perfusion imaging

Bulls eye plot underestimate


apical defect and overestimate
basal defects.
The normal LVEF is 50 - 65%.

Cardiac Muscle Layers

parasympathetic

sympathetic

SA NODES

heart rate

heart rate ( receptor )

ATRIUM

contractility

contractility and conduction


velocity ( receptor )

AV NODES AND CONDUCTION


SYSTEM

conduction velocity and


block conduction

conduction velocity (
receptor )

VENTRICLE

contractility and conduction


velocity ( receptor )

CORONARY ARTERY

Constriction ( receptor )
Dilatation ( receptor )

Coronary artery

Right coronary artery (RCA)

Left main artery (LMA)

Left circumflex (LCX)

AV node
branch

Acute marginal
Branch (AM)

Posterior
descending
Artery (PDA)

Marginal
branches
Obtuse M1
& M2

Left anterior
Descending (LAD)

Diagonal
branches
D1 & D2

Septal
branches

CORONARY CIRCULATION
CORONARY ARTERY

Left anterior descending ( LAD )

VASCULAR DISTRIBUTIONS

Septum
Anterior wall
Apex

Left circumflex ( LCX )

Lateral wall
Posterior wall
Posterior inferior wall
Apex

Right coronary ( RCA )

Inferior wall
Posterior inferior wall
Right ventricular wall

Left main coronary

Anterior wall
Septum
Posterolateral wall

Planes used for SPECT MPI

CONDUCTION SYSTEM
SA ( sinoatrium node )

Pacemaker of heart
Posterior wall of right atrium
Depolarize spontaneously 70-80 time/ min
Contraction of atrium
(interatrium septum)

(interventricular septum)

AV ( atrioventricular node )

( AV bundle )

PURKINJE FIBRES

INNERVATION
parasympathetic

sympathetic

SA NODES

heart rate

heart rate ( receptor )

ATRIUM

contractility

contractility and conduction


velocity ( receptor )

AV NODES AND CONDUCTION


SYSTEM

conduction velocity and


block conduction

conduction velocity (
receptor )

VENTRICLE

contractility and conduction


velocity ( receptor )

CORONARY ARTERY

Constriction ( receptor )
Dilatation ( receptor )

STROKE VOLUME
volume of blood pumped by left ventricle in 1
minute
SV ( STROKE VOLUME ) x HR ( HEART RATE )

VENTRICLE CONTRACTILITY
EF = EDV ESV X 100
EDV

Stroke volume = EDV ESV


EF = ejection fraction ( % )
EDV = End diastolic volume ( ml )
ESV = End systolic volume ( ml )

ECG STAT
DR ZOOL HILMI

ECG paper 25 mm/s


Large square = 5mm = 0.2 s
Each large square = 5 small square = 1 mm =
0.04 s
1 mV = 1 cm vertically
0.1 mV = 1 mm = 1 small square

6 chest leads = V1 to V6
6 limb leads = I, II, III, aVR, aVL, aVF
-

II, III, aVF = inferior


V1 to V4 = anterior
I, aVL, V5 to V6 = lateral
V1 and aVR = right atrium and cavity of left ventricle

INTERPRETATION OF ECG

RATE
RHYTHM
AXIS
P WAVE
PR INTERVAL
QRS COMPLEX
Q WAVE

ST SEGMENT
T WAVE
QT INTERVAL
U WAVE

RATE
Tachycardia > 100 beats /min
Bradycardia < 60 beats/min
Heart rate = 300/ no of large square btw 2 R
Irregular rhythm HR = no of intervals btw QRS
complexes in 10 seconds X 6 (lead II = 25 cm
=10 seconds = 50 large square)

IMPORTANT
PR = 3 5 SMALL SQUARE / 0.12 0.2 SECONDS
QRS 3 SMALL SQUARE / 0.12 SECONDS
BBB > 3 SMALL SQUARE
QT < 11 SMALL SQUARE OR < 0.44 SECONDS OR
< 0.5 RR INTERVAL
CORRECTED QT = QT/RR

RHYTHM
Lead II
Sinus rhythm = originates in the sinus node and
conduct to the ventricle

CARDINAL FEATURES OF SINUS RHYTHM


P wave is upright in leads I and II
Each P wave is usually followed by QRS complex
Heart rate between 60 to 99

CARDIAC AXIS

LEAD I
LEAD II
LEAD III

NORMAL
AXIS

RIGHT AXIS
DEVIATION

LEFT AXIS
DEVIATION

+
+
+/-

+/+

+
-

CARDIAC AXIS
LEAD I

AVF

NORMAL

LEFT AXIS

+ & (LEAD II -)

RIGHT AXIS

INDETERMINATE

CARDIAC AXIS CAUSES


LEFT AXIS

RIGHT AXIS

LVH

RVH

INFERIOR MI

LATERAL MI

LEFT ANTERIOR
FASCICULAR BLOCK

LEFT POSTERIOR
FASCICULAR BLOCK

NORMAL FINDINGS ECG

Tall R waves
Prominent U waves
Exaggerated sinus arrhythmia
Sinus bradycardia
Wandering atrial pacemaker
Wenckebach phenomenon
Junctional rhythm
1st degree heart block
ST segment elevation(high take off, benign early
repolarization)

P WAVE
Atrial depolarization
Characteristic of the P wave:
positive in leads I and II
Best seen in lead II and V1
Commonly biphasic in lead V1
< 3 small square (0.12s)in duration
< 2.5 small square (0.25mV) in amplitude

NORMAL ECG

P WAVE
Bifid P, in lead II , notch with peak to peak interval
of > 1 mm = mitral stenosis

P WAVE
Large negative
deflection = left atrial
enlargement

PR INTERVAL
Electrical impulse conducted through the AV
node, the bundle of His and bundle branches
and the Purkinje fibers

Beginning of the P wave to the 1st deflections


of QRS complex, whether it is Q or R wave
3 to 5 small square (0.12 0.20 s)

QRS COMPLEX
Ventricular depolarization
Not > 2 small square (0.10 s)
BBB > (0.12 s) wide QRS
Q wave : any initial negative deflection
R wave : any positive deflection
S wave : any negative deflection after R wave
Non pathological Q wave seen at: leads I, III,
aVL, V5 and V6

ST SEGMENT
QRS complex terminates at the J point or ST
junction
ST segment lies between the J point and the
beginning of the T wave
Period between the end of ventricular
depolarization and the beginning of the
repolarization

ST SEGMENT
Leads V1 to V3, rapidly S wave merges directly
with the T wave, making the J point indistinct
and the ST segment difficult to identify

This produces elevation of the ST segment,


and this is known as high take off

HIGH TAKE OFF

EARLY REPOLARISATION

T WAVE
Ventricular repolarization
Normal T is ASYMMETRICAL
SYMMETRICAL, Inverted T = myocardial
ischemia
Tall T wave = MI and hyperkalemia
T wave < 2/3 amplitude of R
T wave amplitude < 10 mm

QT INTERVAL
Total
time
for
depolarization
and
repolarization of the ventricle
Beginning of the QRS complex to the end of
the T wave
0.35 0.45 s

U WAVE
Small deflection that follows the T wave
Most prominent in leads V2 to V4
Prominent U = athletes/ hypokalemia/
hypercalcemia

PROMINENT U WAVE

CONDITIONS AFFECTING
THE LEFT SIDE OF THE
HEART

CONDITIONING AFFECTING LEFT SIDE


OF THE HEART
1. Left
atrial
hypertrophy

2. LVH

dilatation

and

LEFT ATRIAL ABNORMALITY


Atrial hypertrophy or dilatation or both
P wave in lead V1 is often biphasic
Early right atrial forces giving rise to an initial
positive deflection
These are followed by left atrial forces
travelling posteriorly, producing a later
negative deflection

LEFT ATRIAL ABNORMALITY

Biphasic P wave in V1

LEFT ATRIAL ABNORMALITY


A large deflection (>1 small square) suggests a
left atrial abnormality
Prolongation of P wave duration to greater
than 0.12 s is often found in association with
the left atrial abnormality
Normal P wave may be bifid

LEFT ATRIAL ABNORMALITY


Any condition causing left ventricular
hypertrophy may produce left atrial
enlargement as a secondary phenomenon
Causes of Left atrial enlargement :
Hypertension
Aortic stenosis
Mitral incompetence
Hypertrophic cardiomyopathy

LEFT ATRIAL ABNORMALITY


P mitrale in lead II
Commonly seen in
mitral stenosis

LEFT VENTRICULAR HYPERTHROPHY


Difficult to diagnose < 40 with ECG
Causes:
1. Hypertension
2. Aortic stenosis
3. Co-arctation of aorta

LEFT VENTRICULAR HYPERTHROPHY

LEFT VENTRICULAR HYPERTHROPHY

LEFT VENTRICULAR HYPERTHROPHY

LEFT VENTRICULAR HYPERTHROPHY

TAHCYCARDIA

HR > 100 bpm

TAHCYCARDIA
Divided into 2:
1. Supraventricular
tachycardia
2. Ventricular
tachycardia

SUPRAVENTRICULAR TACHYCARDIA
(SVT)
From atria or sinoatrial node
1. Sinus tachycardia
2. Atrial fibrillation
3. Atrial flutter
From atrioventricular node
1. Atrioventricular re-entrant tachycardia
2. Atrioventricular nodal re-entrant tachycardia

SUPRAVENTRICULAR TACHYCARDIA
(SVT)
At the atrium
Above bundle of
His
Narrow complex
tachycardia

SUPRAVENTRICULAR TACHYCARDIA
(SVT)

SUPRAVENTRICULAR TACHYCARDIA
(SVT)

SINUS TACHYCARDIA

SUPRAVENTRICULAR TACHYCARDIA
(SVT)

SINUS TACHYCARDIA

SUPRAVENTRICULAR TACHYCARDIA
(SVT)

SINUS TACHYCARDIA

Normal P wave in
inferior lead II,III
and aVF
Atrial rate 100-200
bpm
Regular ventricular
rhythm
Ventricular rate 100200 bpm

ATRIAL FIBRILATION AND ATRIAL


FLUTTER

ATRIAL FIBRILATION

ATRIAL FIBRILATION

NO P WAVES IN AF

ATRIAL FIBRILATION
Mapping
R
waves
against a piece of paper
usually
confirm
diagnosis
May be paroxysmal,
persistent
or
permanent
Treatment : Digoxin

ATRIAL FIBRILATION

ATRIAL FIBRILATION

IRREGULARLY IRREGULAR
NO P WAVE

ATRIAL FLUTTER

ATRIAL FLUTTER
Re-entry circuit in the
right
atrium
with
secondary activation of
left atrium
Atrial contraction 300
bpm
Broad
and
appear
sawtooth best seen at
inferior leads and V1

ATRIAL FLUTTER

JUNCTIONAL TACHYCARDIA

1. Atrioventricular re-entrant tachycardia


2. Atrioventricular nodal re-entrant tachycardia

JUNCTIONAL TACHYCARDIA
Negative P waves in
inferior leads

JUNCTIONAL TACHYCARDIA
PR interval < 3 ss
P
wave
negative
deflection in lead II

ECTOPIC ATRIAL TAHYCARDIA

SUPRAVENTRICULAR TACHYCARDIA
(SVT)

JUNCTIONAL TACHYCARDIA
WOLF PARKINSON
WHITE SYNDROM
Atrioventricular reentrant tachycardia
Electrical signal reenters the atria over
an abnormal extra
pathway

JUNCTIONAL TACHYCARDIA
WOLF
PARKINSON
WHITE SYNDROM
This extra pathway
allows electrical signals
to pass between the
ventricles and atria
Signals on this extra
pathways causing the
atria and ventricles to
beat too fast

JUNCTIONAL TACHYCARDIA
WOLF PARKINSON
WHITE SYNDROM
Short PR interval < 3
small square
Slurred upstroke to
the QRS indicating
pre-excitation (delta
wave)
Broad QRS

VENTRICULAR TACHYCARDIA
At the ventricle
Below bifurcation
bundle of His
Broad
QRS
complex
tachycardia

VENTRICULAR TACHYCARDIA

VENTRICULAR TACHYCARDIA

Define as 3 or more ventricular extrasystoles in


succession at rate > 120 bpm
Causes :
1. IHD
2. Cardiomyopathy
3. Myocarditis
4. Drugs: quanidine

ATRIOVENTRICULAR CONDUCTION
BLOCK
Delayed 1st degree block
Intermittently blocked 2nd degree block
Completely blocked 3rd degree block
Causes:
1. MI
2. Degeneration of His Purkinje system
3. Lyme disease, diphtheria
4. Surgery
5. Congenital disorder

1st DEGREE BLOCK

A delay conduction of atrial impulse to ventricle


At level of atrioventricular node
Prolonged PR interval > 2.0 s
A QRS complex follows each P wave, and PR
interval remains constant

1st DEGREE BLOCK

Prolonged PR

2nd DEGREE BLOCK


MOBITZ TYPE I BLOCK (WENCKEBACH
PHENOMENON)
MOBITZ TYPE II BLOCK

2nd DEGREE BLOCK


At
the
level
of
atrioventricular node
Intermittent
failure
of
transmission of the atrial
impulse to the ventricles
Initial PR interval is normal
but progressively lengthens
with each successive beat
until AV transmission is
blocked completely and the
P wave is not followed by
QRS complex
PR interval then return to
normal and cycle repeats

MOBITZ TYPE I (WENCKEBACH)

Progressive prolongation of PR interval


within group

2nd DEGREE BLOCK

Less common but is more likely to produce


symptoms
Intermittent failure of conduction of P wave
PR interval is constant, though it may normal or
prolonged
Block is often at the level of the bundle brunches and
is therefore associated with wide QRS complexes

MOBITZ TYPE II

Constant PR interval in a group

3rd DEGREE BLOCK

Complete failure of conduction between atria


and ventricles
Complete independence of atria and
ventricular contractions
P waves bear no relation with QRS complexes
Usually proceed with faster rate

AV BLOCK SUMMURY
1st DEGREE AV BLOCK
All P waves conducted with prolonged PR
2nd DEGREE AV BLOCK
Some P waves not conducted
Mobitz type I : Progressive PR prolongation
Mobitz type II : Constant PR interval
3rd DEGREE AV BLOCK
No P waves conducted

BUNDLE BRANCH BLOCK

Right bundle branch block


Left bundle branch block

COMPLETE RBBB
QRS prolongation (
0.12s)
Slurred S wave in
lead V6 and/or rSR
pattern in lead V1
Overall positive QRS
complex in lead V1
(a must)

RBBB

COMPLETE LBBB
QRS prolongation (
0.12s)
Broad R waves in
lead I and V6 with
no Q waves
Broad S waves in the
septal leads

LBBB

HYPERKALAEMIA
Potassium
concentration rises
above
5.5-6.5
mmol/L
Classic
tall,
symmetrically
narrow and peaked T
waves

HYPOKALEMIA
Serum potassium <
2.7 mmol/L
Broad, flat T waves
ST depression
QT
interval
prolongation

ACUTE CORONARY SYNDROME

STEMI

STEMI

STEMI

STEMI

STEMI

Normal Q wave

STEMI

STEMI

Viable myocardium

STEMI
ECG

ONSET

RESOLUTION

HYPERACUTE T WAVE
CHANGES

< 5 MINUTES

HOURS

ST ELEVATION

< 20 MINUTES

HOURS TO 3 DAYS

TERMINAL PORTION QRS


CHANGES

HOURS

HOURS

Q WAVE FORMATION

9 HOURS TO 2 DAYS

PERMANENT

T WAVE INVERSION

VARIABLE

WEEKS

STEMI LOCALISATION

NSTEMI/UNSTABLE ANGINA

NSTEMI/UNSTABLE ANGINA

T INVERSION

NSTEMI/UNSTABLE ANGINA

ST SEGMENT DEPRESSION AND T INVERSION

NSTEMI/UNSTABLE ANGINA

SUMMARY
COMPLETE OCCLUSION
(STEMI)

PARTIAL OCCLUSION
(NSTEMI)

ST ELEVATION

ST DEPRESSION

Q WAVES

T INVERSION
NO Q WAVES

FULL THICKNESS MI

NON Q WAVE MI

Q WAVE MI

PARTIAL THICKNESS MI

TRANSMURAL MI

SUBENDOCARDIAL MI

Pyrophosphate

Glucarate

Antimyosin

Teboroxime

Myocardial perfusion
imaging (MPI)
Fatty acid

NOET

Infarct avid
Single photon
N13

Metabolic

PET
Thallium
Rb82

FDG

O15

Tetrofosmin
Sestamibi

Cardiac scan
Neuronal
Radionuclide
ventriculography

C11epinephrine

Planar

PET

Tc-99m labeled
C11hydroxyephdrine

MIBG
C11phenylephrine

Tc-99m DTPA

Tc-99m RBC

MUGA

First pass
Tc-99m serum
Human albumin

Tc-99m
pertechnitate

Chemical, physical &


pharmacokinetic

Thalium-201

Tc-99m Sestamibi
( Cardiolite )

Tc-99m Tetrofosmin
( Myoview )

Chemical

Element cation

Isonitrile cation

Diphosphine cation

Preparation

cyclotron

Generator/kit

Generator/kit

Mode of decay

Electron capture

Isomeric transition

Isomeric transition

Half life

73 H

6H

6H

Principle emissions

Mercury x rays 6983KeV

Gamma rays 140KeV

Gamma rays 140KeV

Mechanism of uptake

Active transport

Passive diffusion

Passive diffusion

Myocyte localization

Cytosol

Mitochondria

Mitochondria

Redistribution

Yes

No

No

Extraction fraction

85%

60%

50%

Percent cardiac uptake

3%

1.5%

1.2%

Body clearance

Renal

Hepatic

Hepatic

Highest radiation
absorbed dose

Kidney
( rads/3mCi )

Colon
( rads/30mCi )

Gallbladder
( rads/30mCi )

Imaging time
Stress
Rest

10 min
3-4 hrs

15-30 min
30-90 min

5-15 min
30 min

WHAT IS REDISTRIBUTION?

REDISTRIBUTION
Only occurs with Tl 201
Is misnomer
Tl 201 exits the myocardium more slowly from
ischemic segments
In delayed imaging, more rapid exit of Tl 201
from normal segments result in equilibrium
among normal and ischemic segment

WHAT IS STUNNED MYOCARDIUM?

STUNNED MYOCARDIUM
Myocardium with persistent contractile
dysfunction despite restoration of perfusion
after ischemia
Improves with time
Normal by perfusion imaging
Absence of ventricular contraction on wall
motion studies
Increased uptake by FDG

WHAT IS HIBERNATING
MYOCARDIUM?

HIBERNATING MYOCARDIUM
Chronically ischemic myocardium that is still
viable
Appears cold on immediate Tl 201 imaging
Absence of ventricular contraction on wall
motion studies
Increased uptake of FDG compared to
perfusion imaging
Improved perfusion on reinjection of Tl 201

Normal scan

Stress (top) and rest (bottom) MPI images from normal male subject

ISCHEMIC MYOCARDIUM IMAGES?

ISCHEMIC MYOCARDIUM
Reduced Tl 201 myocardial uptake on post
stress images
Improves uptake on the rest images

Apical ischemia

Stress (top) and rest (bottom) MPI images from a subject with apical
ischemia.

SCAR IMAGES ?

SCAR
Reduced Tl 201 myocardial uptake post stress
images
Reduced uptake on the rest images

Antero-apical infarct

Stress (top) and rest (bottom) MPI images from a patient with prior
extensive antero-apical MI and no residual ischemia.

MYOCARDIAL IMAGING
RADIOPHARMACEUTICAL

DOSE ( mCi )

IMAGING TECHNIQUE

TYPE OF STUDY

Tl Cl

2-3

Planar or SPECT

Perfusion

Tc 99m sestaMIBI

10-30

Planar or SPECT

Perfusion

Tc 99m teboroxime

15-30

Planar or SPECT

Perfusion

Tc 99m PYP

10-15

Planar

Infarct

In labeled antimyosin
antibody

2-3

Planar

Infarct

RbCl

60

PET

Perfusion

NH

15-20

PET

Perfusion

F-FDG

5-10

PET

Metabolism

Sensitivity & Specificity


Sensitivity

Specificity

Myocardial
Perfusion Scan

93%

88%

Stress
Echocardiography

81%

82%

Exercise Stress
Test

68%

77%

INDICATIONS FOR MYOCARDIAL


PERFUSION SCAN
Detection of CAD
Assessment of severity of myocardial ischemia or
infarction
Assessment of myocardial viability
Evaluation for candidate of coronary bypass or
angioplasty
Clinical indication of MI, chest pain and SOB
CPK,LDH, +ve troponin and myoglobin
Detection of viable/hibernating myocardial tissue
Evaluation of angioplasty/bypass surgery
Evaluation of prognosis

CONTRAINDICATIONS FOR
MYOCARDIAL PERFUSION

Chest pain
Ventricular arrhythmia at rest
Very high BP
Allergy to pharmacological stress drugs
Third degree heart block
Severe valvular disease ( aortic valve stenosis )

Selection of Stress Technique


Coronary flow disturbance vs diameter narrowing
Mean hyperemic flow
Mean resting flow
Dypiridamole/ Adenosine
X4
Diff. btw.
sternotic
& nonstenotic
counts.

Dobutamine
X3

Treadmill exercise
X2
Resting blood flow
X1

50

100

% diameter narrowing

STRESS PROTOCOLS
1. PHYSICAL EXERCISE
2. PHARMACOLOGICAL STRESS

STRESS TESTING INDICATIONS


Diagnosis of CAD
Evaluation of CAD, location and extent of ischemia
Determine the cause for change in symptom pattern in
patients with known CAD
Evaluate the effectiveness of medical therapy
Risk stratification post myocardial infarction
Pre operative evaluation for major non cardiac surgery
in patient with known CAD
Assessment after angioplasty or bypass
Guide to rehabilitation therapy

STRESS TESTING CONTRAINDICATIONS

Acute MI
Unstable angina
Severe tachyarrhythmias and bradyarrhythmias
Uncontrolled symptomatic heart failure
Critical aortic stenosis
Acute aortic dissection
Pulmonary embolism
Poorly controlled HPT

INDICATIONS FOR TERMINATING A


STRESS TEST
Patients request
Inability to continue due to fatigue, dyspnea, or
faintness
Moderate to severe chest pain
Dizziness, near syncope
Pallor
Ventricular tachycardia
Atrial fibrillation
Third degree heart block
ST segment depression > 3 mm
Increased SBP > 240 mmHg or DBP > 120 mmHg

Myoview counseling
Stop medications:
At the appointment date :
1. Aspirin
2. OHA/insulin
2 days prior :
1. Neulin (theophylline)
2. Beta blockers
3. Viagra
3 days prior :
1. Nitrates/trimetazidine

CARDIAC STRESS TEST : EXERCISE AND


PHARMACOLOGY
LEG EXERCISE
Treadmill
Bicycle ergometer

PHARMACOLOGIC STRESS
Dipyridamole
Adenosine
Dobutamine

STRESS PROTOCOLS

1.PHYSICAL EXERCISE
2. PHARMACOLOGICAL STRESS

PHYSICAL EXERCISE

Treadmill or bicycle
ECG 10-12 lead used
Run baseline ECG
Start exercise
Monitor HR, BP, ECG changes till target HR is
achieved > 85% of ( 220-AGE )
Radiotracer is injected when target HR is reached
Maintain exercise for 1 minute

STRESS PROTOCOLS
1. PHYSICAL EXERCISE

2.PHARMACOLOGICAL
STRESS

PHARMACOLOGICAL STRESS
1. ADENOSINE
2. DOBUTAMINE
3. DIPYRIDAMOLE

Dobutamine
adrenergic agonist
Synthetic catecholamine
1 affinity
Short plasma half life
Cause ionotrophic/chronotrophic effect due
to high O2 demand cause by artery stenosis
Infusion rate (40-50 g/kg/min)

Dobutamine
Can be used in asthma patients
Side effect:
Supraventricular and ventricular arrhythmia
Palpitation
SOB
GI symptom
Chest pain

Dobutamine
Contraindications:
ST depression
Ventricular tachycardia
Systolic BP > 220 mmHg

Adenosine

Dypiridamole

Dobutamine

Mechanism of
action

Direct

Indirect

Indirect

Half life

< 10 s

30 60 min

2 min

Onset of action

Seconds

2 min

1 2 min

Patients with side


effect requiring
medical
intervention

0.6%

16%

NA

ADVERSE EFFECTS: ADENOSINE AND


DIPYRIDAMOLE
ADVERSE EFFECT

ADENOSINE ( % )

DIPYRIDAMOLE ( % )

Flushing

37

Dyspnea

35

Chest pain

25

20

GI symptoms

15

Headache

14

12

Dizziness

12

A-V block

ST wave changes

Arrhythmia

Hypotension

Bronchospasm

0.1

0.15

MI

0.0001

0.05

Death

0.5

ARTIFACTS FOR MPI


Patient motion
Diaphragm in men and breast in female cause
attenuation artifacts
Medallion, necklaces, pacemaker or metallic
button
Left arm of the patient, lying on the side may
cause unwanted attenuation
Gallbladder with sestaMIBI may cause
problems

Breast attenuation effect

Stress (top) and rest (bottom) MPI images from a female subject
showing a reduction in counts anteriorly in both stress and rest
images.

Reporting MPI

PET Cardiac

Concerning the blood supply of the


normal heart
1. The right coronary artery supplies the AV
node
2. The anterior intraventricular artery is a
branch of RCA
3. Arise from the inferior aspect of the aortic
arch
4. The circumflex artery is a branch of the LCA
5. The great cardiac vein drains the territory
supplied by the RCA

Concerning the blood supply of the


normal heart
1. The right coronary artery supplies the AV node
2. The anterior intraventricular artery is a branch
of RCA
3. Arise from the inferior aspect of the aortic arch
4. The circumflex artery is a branch of the LCA
5. The great cardiac vein drains the territory
supplied by the RCA
TFFTF

FOLLOWING ARE INDICATIONS OF


GATED BLOOD POOL STUDY
1.
2.
3.
4.
5.

Measuring right ventricular ejection fraction


Quantifying shunts
Cardiotoxic drugs
Extent of infarct
Myocardial reserve

FOLLOWING ARE INDICATIONS OF


GATED BLOOD POOL STUDY
1. Measuring right ventricular ejection fraction
2. Quantifying shunts
3. Cardiotoxic drugs
4. Extent of infarct
5. Myocardial reserve
FFTFT

IMAGING OF INFARCT
1. Tc pyrophosphate is the infarct avid imaging
procedure of choice
2. Pyrophosphate uptake is taken more around the
infarct than in the infarct
3. Myosin monoclonal antibody is useful only in
imaged within 2-3 days after administration
4. Pyrophosphate is taken up by bone
5. Images are acquired after 48 hours in myosin
antibody

IMAGING OF INFARCT
1. Tc pyrophosphate is the infarct avid imaging
procedure of choice
2. Pyrophosphate uptake is taken more around the
infarct than in the infarct
3. Myosin monoclonal antibody is useful only in
imaged within 2-3 days after administration
4. Pyrophosphate is taken up by bone
5. Images are acquired after 48 hours in myosin
antibody
FTFTF

CONTRAINDICATION FOR
MYOCARDIAL PERFUSION IMAGING (
EXERSICE STRESS )
1.
2.
3.
4.
5.

Sinus bradycardia
Third degree heart block
Ventricular arrhythmias
Aortic stenosis
Unstable angina

CONTRAINDICATION FOR
MYOCARDIAL PERFUSION IMAGING (
EXERSICE STRESS )
1. Sinus bradycardia
2. Third degree heart block
3. Ventricular arrhythmias
4. Aortic stenosis
5. Unstable angina
FTTTT

MYOCARDIAL PERFUSION IMAGING


1. Bulls eye maps are used to predict the
prognosis of infarct
2. Stress images are acquired 30-60 mins after
injection
3. GTN is administered when acquiring rest
images
4. Rest images are acquired far later than stress
images
5. Injection is done at maximum stress

MYOCARDIAL PERFUSION IMAGING


1. Bulls eye maps are used to predict the prognosis
of infarct
2. Stress images are acquired 30-60 mins after
injection
3. GTN is administered when acquiring rest images
4. Rest images are acquired far later than stress
images
5. Injection is done at maximum stress
TTTTT

STRESS IN MYOCARDIAL PERFUSION


IMAGING
1. Dipyridamole is the stress agent of choice
2. Systolic pressure less than 100mm is
contraindication for adenosine
3. The aim of stress is to decrease the coronary
arterial flow
4. Adrenaline is the antidote to adenosine
complication
5. Asthma is a contraindication to adenosine

STRESS IN MYOCARDIAL PERFUSION


IMAGING
1. Dipyridamole is the stress agent of choice
2. Systolic pressure less than 100mm is
contraindication for adenosine
3. The aim of stress is to decrease the coronary
arterial flow
4. Adrenaline is the antidote to adenosine
complication
5. Asthma is a contraindication to adenosine
FTFFT

CARDIAC SCINTIGRAPHY
1. LAO best for left ventricle
2. RAO 30 best for right ventricle
3. For assessing RV ejection fraction, inject the
tight bolus
4. For shunt quantification, inject over 3
seconds
5. First pass takes 15 seconds

CARDIAC SCINTIGRAPHY
1. LAO best for left ventricle
2. RAO 30 best for right ventricle
3. For assessing RV ejection fraction, inject the
tight bolus
4. For shunt quantification, inject over 3
seconds
5. First pass takes 15 seconds
TTFFT

CARDIAC SCINTIGRAPHY
1. MIBI is more prone for liver uptake
2. Tetrofosmin is easier to prepare than MIBI
3. Stannous pyrophosphate and pertechnate
are mixed before administrating in vein
4. MUGA scan are better than LIST studies
5. Acquisition begins with R wave of each cycle

CARDIAC SCINTIGRAPHY
1. MIBI is more prone for liver uptake
2. Tetrofosmin is easier to prepare than MIBI
3. Stannous pyrophosphate and pertechnate
are mixed before administrating in vein
4. MUGA scan are better than LIST studies
5. Acquisition begins with R wave of each cycle
TTFTT

THALLIUM 201
1. Decays by electron capture to Tl 203
2. Mainly produce gamma photon of 135 and
167 keV
3. After IV administration 15% is localized in the
myocardium
4. All the thallium ions enter the myocytes by
the sodium potassium ATPase pump
5. Extraction of thallium is reduced in acidosis

THALLIUM 201
1. Decays by electron capture to Tl 203
2. Mainly produce gamma photon of 135 and 167
keV
3. After IV administration 15% is localized in the
myocardium
4. All the thallium ions enter the myocytes by the
sodium potassium ATPase pump
5. Extraction of thallium is reduced in acidosis
FFFFF

FEATURES OF HIBERNATING
MYOCARDIUM
1. Reversible
2. FDG PET is the standard for assessing
myocardial viability
3. Adverse cardiac events in these patients are
higher if treated with revascularization
4. Decreased uptake in FDG
5. Decreased perfusion in thallium scans

FEATURES OF HIBERNATING
MYOCARDIUM
1. Reversible
2. FDG PET is the standard for assessing
myocardial viability
3. Adverse cardiac events in these patients are
higher if treated with revascularization
4. Decreased uptake in FDG
5. Decreased perfusion in thallium scans
TTFFT

MYOCARDIAL PERFUSION IMAGING


1. 2 separate injection are required for thallium,
unlike for MIBI and tetrofosmin
2. ST segment depression more than 3mm is
indicator that dobutamine should be stopped
3. Rest images are performed first, followed by
stress images
4. At least 2 days are required for satisfactory
testing using MIBI
5. Infarcts will show decreased uptake during
stress and improved in rest images

MYOCARDIAL PERFUSION IMAGING


1. 2 separate injection are required for thallium, unlike
for MIBI and tetrofosmin
2. ST segment depression more than 3mm is indicator
that dobutamine should be stopped
3. Rest images are performed first, followed by stress
images
4. At least 2 days are required for satisfactory testing
using MIBI
5. Infarcts will show decreased uptake during stress and
improved in rest images
FTFFF

THALLIUM 201 IMAGING


1. Taken up by infarcts
2. Left ventricular function can be assessed
using first pass studies
3. Can be used to assess a left to right shunt
4. Normal to have a photon deficient area at
the apex
5. Left ventricular failure is associated with
increased lung uptake

THALLIUM 201 IMAGING


1. Taken up by infarcts
2. Left ventricular function can be assessed using
first pass studies
3. Can be used to assess a left to right shunt
4. Normal to have a photon deficient area at the
apex
5. Left ventricular failure is associated with
increased lung uptake
FTTTT

THALLIUM SCINTIGRAPHY
1. Involves a lower radiation dose than Tc99m
MIBI scanning
2. Reverse redistribution is commonly due to
artifact
3. Injection is performed at peak exercise
4. There is increased uptake in areas of
myocardial infarction
5. Is more sensitive than cardiac stress testing

THALLIUM SCINTIGRAPHY
1. Involves a lower radiation dose than Tc99m MIBI
scanning
2. Reverse redistribution is commonly due to
artifact
3. Injection is performed at peak exercise
4. There is increased uptake in areas of myocardial
infarction
5. Is more sensitive than cardiac stress testing
FTTFT

THALLIUM 201
1. Kidney is the critical organ
2. Distribution is proportional to perfusion
3. Physical half life is 24 hours, making it low
dose
4. Images are of high resolution
5. High signal to noise ratio

THALLIUM 201
1. Kidney is the critical organ
2. Distribution is proportional to perfusion
3. Physical half life is 24 hours, making it low
dose
4. Images are of high resolution
5. High signal to noise ratio
TTFFF

PET AGENTS USED IN HEART IMAGING


1.
2.
3.
4.
5.

Nitrogen 13 ammonia
Rubidium 82
Potassium 38
Oxygen 15 labeled water
Inhaled 15 CO2

PET AGENTS USED IN HEART IMAGING


1. Nitrogen 13 ammonia
2. Rubidium 82
3. Potassium 38
4. Oxygen 15 labeled water
5. Inhaled 15 CO2
TTTTT

UPTAKE OF THALLIUM IS SEEN IN


FOLLOWING STRUCTURES
1.
2.
3.
4.
5.

Kidney
Lungs
Salivary glands
Skeletal muscle
Liver

UPTAKE OF THALLIUM IS SEEN IN


FOLLOWING STRUCTURES
1. Kidney
2. Lungs
3. Salivary glands
4. Skeletal muscle
5. Liver
TTFTF

STRESS TESTING
1. Adenosine combined with exercise improves
detection of perfusion defects
2. Dobutamine avoided if patient has asthma
3. Adenosine should be avoided if patient has
bifascicular block or LBBB
4. Bradyarrhythmia is reduced by exercise
5. Dipyridamole has the highest sensitivity and
specificity among pharmacological agents

STRESS TESTING
1. Adenosine combined with exercise improves
detection of perfusion defects
2. Dobutamine avoided if patient has asthma
3. Adenosine should be avoided if patient has
bifascicular block or LBBB
4. Bradyarrhythmia is reduced by exercise
5. Dipyridamole has the highest sensitivity and
specificity among pharmacological agents
TFFTF

MIBI CARDIAC IMAGING


1. Rescan is done at 4 hours as redistribution
occurs
2. 5% of negative studies go on to get MI within
one year
3. Increased uptake is seen in myocardial
infarction
4. Gated studies useful to look at wall motion
5. Fatty meal should not be taken as it produces
hepatic uptake

MIBI CARDIAC IMAGING


1. Rescan is done at 4 hours as redistribution
occurs
2. 5% of negative studies go on to get MI within
one year
3. Increased uptake is seen in myocardial infarction
4. Gated studies useful to look at wall motion
5. Fatty meal should not be taken as it produces
hepatic uptake
FFTTF

INCREASED UPTAKE OF THALLIUM


1.
2.
3.
4.
5.

Graves disease
Thyroid ca
Bronchogenic ca in lung
Lymphoma of lung
Brain in encephalitis

INCREASED UPTAKE OF THALLIUM


1. Graves disease
2. Thyroid ca
3. Bronchogenic ca in lung
4. Lymphoma of lung
5. Brain in encephalitis
TTTTT

MYOCARDIAL PERFUSION
SCINTIGRAPHY
1. The physical properties of thallium 201 are such that
its use is preferable to that of Tc99m labeled
compounds in myocardial perfusion imaging
2. The effective dose from Tl 201 myocardial perfusion
scan is one of the highest in diagnostic imaging and
roughly equates to 3 barium enemas
3. The uptake of Tl 201 relies solely on regional
myocardial perfusion
4. Tl 201 is a Na analogue
5. Tl 201 remains irreversibly fixed to the myocardium
within a few minutes of injection

MYOCARDIAL PERFUSION
SCINTIGRAPHY
1. The physical properties of thallium 201 are such that
its use is preferable to that of Tc99m labeled
compounds in myocardial perfusion imaging
2. The effective dose from Tl 201 myocardial perfusion
scan is one of the highest in diagnostic imaging and
roughly equates to 3 barium enemas
3. The uptake of Tl 201 relies solely on regional
myocardial perfusion
4. Tl 201 is a Na analogue
5. Tl 201 remains irreversibly fixed to the myocardium
within a few minutes of injection
FTFFF

THE FOLLOWING MAY CAUSE


REDUCTION IN Tl 201 UPTAKE TO THE
MYOCARDIUM
1.
2.
3.
4.
5.

Myocardial infarction
Hibernating myocardium
Cardiomyopathy
Cardiac sarcoidosis
Dobutamine

THE FOLLOWING MAY CAUSE


REDUCTION IN Tl 201 UPTAKE TO THE
MYOCARDIUM
1. Myocardial infarction
2. Hibernating myocardium
3. Cardiomyopathy
4. Cardiac sarcoidosis
5. Dobutamine
TTTTT

PROGNOSIS IN CORONARY ARTERY


DISEASE
1. Increased uptake of Tl 201 into the lungs is a poor
prognostic indicator during myocardial perfusion imaging
2. Myocardial perfusion imaging may identify patients at
increased risk of cardiac events following major vascular
surgery
3. Myocardial perfusion imaging has greater prognostic
power than exercise ECG
4. Myocardial perfusion imaging has less prognostic power
than coronary arteriography
5. In a patient with a strongly positive ECG but a normal
myocardial perfusion scan, the perfusion scan is the most
appropriate for defining prognosis

PROGNOSIS IN CORONARY ARTERY


DISEASE
1. Increased uptake of Tl 201 into the lungs is a poor
prognostic indicator during myocardial perfusion imaging
2. Myocardial perfusion imaging may identify patients at
increased risk of cardiac events following major vascular
surgery
3. Myocardial perfusion imaging has greater prognostic
power than exercise ECG
4. Myocardial perfusion imaging has less prognostic power
than coronary arteriography
5. In a patient with a strongly positive ECG but a normal
myocardial perfusion scan, the perfusion scan is the most
appropriate for defining prognosis
TTTFT

CONCERNING PHARMACOLOGICAL
STRESSOR AGENTS
1. Dipyridamole reduces uptake of myocardial
perfusion agents to myocardial
2. Adenosine may be use as a cardiac stressor
3. Caffeine intake should be restricted before an
exercise myocardial perfusion
4. Aminophylline may reverse chest pain caused
by dipyridamole
5. Dipyridamole is a potent coronary
vasodilator

CONCERNING PHARMACOLOGICAL
STRESSOR AGENTS
1. Dipyridamole reduces uptake of myocardial
perfusion agents to myocardial
2. Adenosine may be use as a cardiac stressor
3. Caffeine intake should be restricted before an
exercise myocardial perfusion
4. Aminophylline may reverse chest pain caused by
dipyridamole
5. Dipyridamole is a potent coronary vasodilator
FTTTT

Tc 99m LABELLED MYOCARDIAL


PERFUSION AGENTS
1. Stress and redistribution images of Tc 99m SestaMIBI
accurately identify ischemic myocardium
2. Tc99m SestaMIBI is more suited to SPECT scanning
than Tl 201
3. A fatty meal is required following sestaMIBI injection
4. Tc 99m SestaMIBI stress and rest myocardial perfusion
scans cannot be performed on the same day due to
the long redistribution time of this agent in
myocardium
5. Tc 99m SestaMIBI imaging should be performed as
soon as possible following injection

Tc 99m LABELLED MYOCARDIAL


PERFUSION AGENTS
1. Stress and redistribution images of Tc 99m SestaMIBI
accurately identify ischemic myocardium
2. Tc99m SestaMIBI is more suited to SPECT scanning than Tl
201
3. A fatty meal is required following sestaMIBI injection
4. Tc 99m SestaMIBI stress and rest myocardial perfusion
scans cannot be performed on the same day due to the
long redistribution time of this agent in myocardium
5. Tc 99m SestaMIBI imaging should be performed as soon as
possible following injection
FTTFF

EQUILIBRIUM GATED BLOOD POOL


IMAGING
1. Tc 99m DTPA is preferable to labeled red blood
cells
in
gated
ventriculography
as
radiopharmaceutical preparation is easier
2. Red cells may be labeled with Tc 99m in vivo
3. Concurrent ECG recording is only necessary if
there is risk of arrhythmia
4. It is possible to assess regional wall motion
abnormalities of the left ventricle
5. Background substraction is not usually required
in measuring LV ejection fractions as activity
within the adjacent lung is negligible

EQUILIBRIUM GATED BLOOD POOL


IMAGING
1. Tc 99m DTPA is preferable to labeled red blood cells in
gated ventriculography as radiopharmaceutical
preparation is easier
2. Red cells may be labeled with Tc 99m in vivo
3. Concurrent ECG recording is only necessary if there is
risk of arrhythmia
4. It is possible to assess regional wall motion
abnormalities of the left ventricle
5. Background substraction is not usually required in
measuring LV ejection fractions as activity within the
adjacent lung is negligible
FTFTF

EQUILIBRIUM GATED BLOOD POOL


IMAGING
1. Patients with coronary artery disease typically show an
increase in LV ejection fraction of only 5-10% on exercise
2. Regional wall motion abnormalities are more specific for
coronary artery disease than changes in ejection fraction
with exercise
3. Patients receiving chemotherapy should be suspected of
having cardiotoxicity if the ejection fraction falls by more
than 10% on sequential scans
4. It is possible to differentiate alcoholic from viral
cardiomyopathy by gated blood pool imaging
5. It is possible to differentiate ischemic cardiomyopathy
from dilated idiopathic cardiomyopathy with gated blood
pool imaging

EQUILIBRIUM GATED BLOOD POOL


IMAGING
1.

Patients with coronary artery disease typically show an increase in


LV ejection fraction of only 5-10% on exercise
2. Regional wall motion abnormalities are more specific for coronary
artery disease than changes in ejection fraction with exercise
3. Patients receiving chemotherapy should be suspected of having
cardiotoxicity if the ejection fraction falls by more than 10% on
sequential scans
4. It is possible to differentiate alcoholic from viral cardiomyopathy
by gated blood pool imaging
5. It is possible to differentiate ischemic cardiomyopathy from
dilated idiopathic cardiomyopathy with gated blood pool imaging
FTTFT

RADIONUCLIDE VENTRICULOGRAPHY
1. Radionuclide ventriculography is a more accurate and
reproducible method for measuring ejection fraction than
echocardiography
2. Assumptions for the calculation of ejection fraction are
greater for echocardiography than they are for
radionuclide ventriculography
3. In severe left ventricular dysfunction the volume of
injected radiopharmaceutical is reduced to avoid overload
4. Exercise ventriculography is not possible because there is
too much motion of the patient
5. It is not possible to measure RV ejection fraction using
gated equilibrium technique

RADIONUCLIDE VENTRICULOGRAPHY
1. Radionuclide ventriculography is a more accurate and
reproducible method for measuring ejection fraction than
echocardiography
2. Assumptions for the calculation of ejection fraction are
greater for echocardiography than they are for
radionuclide ventriculography
3. In severe left ventricular dysfunction the volume of
injected radiopharmaceutical is reduced to avoid overload
4. Exercise ventriculography is not possible because there is
too much motion of the patient
5. It is not possible to measure RV ejection fraction using
gated equilibrium technique
TTFFF

FIRST PASS RADIONUCLIDE


ANGIOGRAPHY
1. It is possible to detect R to L cardiac shunts with this
technique
2. It is possible to detect L to R cardiac shunts with this
technique
3. As large a dilution of radiopharmaceutical as possible
should be used in this technique
4. It is an accurate technique to quantify cardiac
shunting which coexist with coaortation of the aorta
5. It is often not possible to accurately quantitate
cardiac L to R shunts in the presence of tricuspid
regurgitation

FIRST PASS RADIONUCLIDE


ANGIOGRAPHY
1. It is possible to detect R to L cardiac shunts with this
technique
2. It is possible to detect L to R cardiac shunts with this
technique
3. As large a dilution of radiopharmaceutical as possible
should be used in this technique
4. It is an accurate technique to quantify cardiac
shunting which coexist with coaortation of the aorta
5. It is often not possible to accurately quantitate
cardiac L to R shunts in the presence of tricuspid
regurgitation
TTFFT

MYOCARDIAL INFARCT SCANNING


1. A myocardial infarction may be seen on an MDP
bone scan
2. Tc 99m labeled pyrophosphate imaging may
detect myocardial infarction before cardiac
enzyme changes
3. Rib fracture may give false positive results with
pyrophosphate scanning
4. Positive antimyosin monoclonal antibody
scanning is specific for myocardial infarction
5. Rejection of heart transplants can be detected
with antimyosin scanning

MYOCARDIAL INFARCT SCANNING


1. A myocardial infarction may be seen on an MDP bone
scan
2. Tc 99m labeled pyrophosphate imaging may detect
myocardial infarction before cardiac enzyme changes
3. Rib fracture may give false positive results with
pyrophosphate scanning
4. Positive antimyosin monoclonal antibody scanning is
specific for myocardial infarction
5. Rejection of heart transplants can be detected with
antimyosin scanning
TFTFT

How does the percent extraction of


thallium-201 passing through the
myocardial capillary bed compared
with the extraction of c-99m sestamibi
and Tc-99m teboroxime?

How does the percent extraction of


thallium-201 passing through the
myocardial capillary bed compared
with the extraction of c-99m sestamibi
and Tc-99m tetrofosmin?
Tl-201 has a myocardial extraction fraction of
0.85 in normal subjects at a normal flow rates.
The myocardial extraction of Tc-99m sestamibi
and Tc-99m tetrofosmin is considerably lower
0.5 and 0.6 respectively.

What percentage of Tl-201, Tc-99m


sestamibi and Tc-99m tetrofosmin
localizes in the heart?

What percentage of Tl-201, Tc-99m


sestamibi and Tc-99m tetrofosmin
localizes in the heart?
Tl-201 : 3-4%
Tc-99m sestamibi : 1.5%
Tc-99m tetrofosmin : 1.2%

What are the advantages and


disadvantages of Tl-201 as perfusion
agent?

What are the advantages and


disadvantages of Tl-201 as perfusion
agent?
Advantages
Single injection because of
redistribution
Imaging within 10 15
minutes
Can be used to assess
viability

Disadvantages
High radiation
Poor imaging characteristics
with a low photopeak of 69
80 keV
High scatter fraction

What quality control to detect patient


motion?

What quality control to detect patient


motion?
Review raw data in cinematic display and
review of sonogram can confirm the extend of
the problem.

In what ways can the image


interpreter determine if fixed
decreased activity is due to artifact or
disease?

In what ways can the image


interpreter determine if fixed
decreased activity is due to artifact or
disease?
Review the raw data in the cinematic display
to look for soft attenuation.
Review the gated SPECT can help determine if
there is wall motion and thickening that
would indicate infarct or attenuation.
Attenuation correction can be helpful.
Prone can differentiate attenuation from
infarction in inferior wall.

What is the significant of lung uptake


on Tl-201 exercise study?

What is the significant of lung uptake


on Tl-201 exercise study?
Lung uptake on exercise stress images, but not
the delayed images is consistent with exercise
induced cardiac dysfunction.
This finding is usually associated with three
vessel disease.

What other scintigraphic finding


suggest three vessel disease?

What other scintigraphic finding


suggest three vessel disease?
Exercise induced ischemic dilatation.
The normal heart dilates during stress but
gated SPECT is acquired poststress when
normal hearts have returned to baseline size.

What is the mechanism of action of


dipyridamole?

What is the mechanism of action of


dipyridamole?
Dipyridamole inhibits the action of adenosine
deaminase.
By augmenting the effects of endogenous
adenosine, dipyridamole is a powerful
vasodilator.

What effect can a cup of coffee have


on dipyridamole or adenosine stress
test?

What effect can a cup of coffee have


on dipyridamole or adenosine stress
test?
Caffeine in coffee, tea, soft drinks or food such
as chocolate are chemically related to
dipyridamole and adenosine and can block the
effect of dipyridamole pharmacological stress
testing.

What percentage of stenosis at rest is


necessary in the coronary arteries for
resting blood flow to be effected?

What percentage of stenosis at rest is


necessary in the coronary arteries for
resting blood flow to be effected?
> 85 90%

Why is imaging delayed for 30 90


minutes after administration of Tc-99m
sestamibi or tetrofosmin?

Why is imaging delayed for 30 90


minutes after administration of Tc-99m
sestamibi or tetrofosmin?
Lung uptake is also significant.
The lung and liver clear with time and the
target to background ratio improves.

What pharmaceutical is administered


to allows the Tc-99m to bind to RBC?

What pharmaceutical is administered


to allows the Tc-99m to bind to RBC?
Stannous (tin) pyrophosphate and stannous
chloride has been used.

What part of the RBC does the Tc-99m


label bind?

What part of the RBC does the Tc-99m


label bind?
Beta chain of hemoglobin when the patient is
pretreated with stannous ion.

What do amplitude and phase images


portray?

What do amplitude and phase images


portray?
Amplitude and phase images are parametric
or derived images.
The amplitude images portrays the maximum
count difference at each pixel location during
the cardiac cycle. High EF areas have high
amplitude and background areas have low
amplitude.
The phase images portrays the timing of
cyclical activity with respect to a reference
standard.

What is the hallmark of a ventricular


apical aneurysm by phase analysis?

What is the hallmark of a ventricular


apical aneurysm by phase analysis?
Aneurysms demonstrate paradoxical motion.
Activity in the area of the aneurysm is typically
180 degrees out of phase with the rest of the
ventricle.

LAD ischemia
1. Severe decreased perfusion in the majority of
the anterior wall, apex and septum which
normalizes on the rest image indicating
extensive severe ischemia.
2. LAD territory.
3. Transient cavity dilatation.
4. VT, angina related ST abnormalities,
decreased systolic pressure and level of
exercise achieved.

Apical infarct
1. Fixed stress and rest severe apical perfusion
defect. Heart and cavity size appear normal.
2. Myocardial infarction, apical thinning and
attenuation.
3. Small apical lateral scar.
4. Technical factors, operator error and
interpretation error

Breast attenuation
1. Mild fixed anteroseptal hypoperfusion that
demonstrates uniform brightening on gated
SPECT, indicating normal myocardial wall
thickening on gated images.
2. Apparent decreased tracer in the upper
portions of the heart is most obvious on the
left anterior oblique and lateral frames.
3. Normal perfusion study with normal wall
thickening and breast attenuation.
4. Assessment of regional wall motion/ wall
thickening and LVEF

Inferolateral wall infarction


1. If the patient develops severe angina chest pain,
a decrease in blood pressure, frequent
premature ventricular contraction or ST
elevation suggestive of acute infarct. Also if
patient can walk no further on the treadmill
because of general fatigue, leg pain or dyspnea.
2. Severe stress and rest fixed defect in the
basolateral, inferior and inferolateral walls,
sparing the apex.
3. MI or possibly hibernation. Circumflex artery.
4. The exercise stress level.

Dipyridamole induced reversible


ischemia
1. A perfusion defect involving the entire
inferior wall extending to the apex shows
partial reversibility.
2. RCA
3. Inferior wall ischemia with incomplete
reversibility. The latter may represent scar or
hibernating myocardium.
4. The patient is at risk for further cardiac
event, either infarct or death.

Adenosine stress
1. A. adenosine is infused IV for 6 minutes (140
g/kg/min) after 3 min the tracer is injected and
adenosine is continued for 3 more minutes. B.
adenosine is cleared from circulation <10 sec. return
to baseline blood flow levels occurs in 2 to 3 minutes
after stopping the infusion. C. stop infusion
2. Sinus node disease, 2nd to 3rd degree block,
bronchospastic lung disease and adenosine allergy
3. Small to moderate severe fixed defect at the apex on
both stress and rest images consistent with infarct.
Mildly improved perfusion of the anterior and lateral
walls at rest compared with stress consistent with
mild anterolateral ischemia.
4. Whenever adequate exercise stress is not possible.

Viability
1. Extensive fixed defects involving the anterior
wall, apex, septum extending to the lateral
wall.
2. Myocardial infarction vs. hibernating
myocardium.
3. In hibernating myocardium, blood flow and
function e.g.. contractility are chronically
reduced.
4. Blood flow is normal with reduced function.

Bulls eye
1. A polar plot is constructed by layering short axis slices
one on top of the other with the apex forming the
center and the base of the heart being the outermost
portion.
2. Misregistration use of inappropriate reference
database.
3. Stress; hypoperfusion of the anterior, lateral and
inferior walls. Rest; normalized perfusion of the
anterior and lateral walls and incomplete
normalization of the inferior wall. Most consistent
with ischemia of the LCX and infarct of the right RCA.
4. Include location and extent, severity and reversibility
for each perfusion abnormality. If gated SPECT is
performed, include LVEF, wall motion with or without
wall thickening fractions.

Inadequate stress
1. Severe fixed defect involving the both entire
lateral wall
2. Dilated left ventricular cavity at both stress
and rest.
3. LCX artery
4. False negative studies for ischemia may result

LBBB
1. Dipyridamole or adenosine
2. Exercise or dobutamine. Methods of stress that
result in increased heart rate can be associated
with false positive findings of septal reversibility
in patient with LBBB
3. These agents do not result in an increased in
heart rate.
4. The mild decreased activity in the anterior wall
appears fixed and likely is caused by breast
attenuation in light of the reported normal wall
motion.

Dobutamine stress
1. Dobutamine stress; patients who are not
candidate for either exercise; asthma
2. Angina and inability to tolerate dobutamine
3. Mild fixed anteroseptal perfusion defect with
decreased wall thickening. Severe fixed
inferior defect with absent wall thickening.
Dilate LV. No reversibility. Myocardial
thickening and wall motion signify
functioning viable myocardium.

Patient motion artifact


1. A. the first study shows an abnormal
configuration of the anterior wall; the repeat
study is normal. B, The initial sinogram shows a
discontinuity or break, the second is normal.
2. To visually present the raw unprocessed
projection images to evaluate for patient
motion.
3. Review SPECT projection, image by image or in
cinematic display.
4. Artifact caused by patient motion.

Breast attenuation
1. Tc-99m sestamibi, tetrofosmin and not Tl-201
because the GB is seen.
2. Mild fixed defect in the anterior wall.
Projection images; decreased uptake in the
half of the heart at stress and rest
3. Breast attenuation, anterior wall infarction.
4. Assessment of regional wall motion, wall
thickening and EF

Bowel activity
1. Dipyridamole inhibits the action of adenosine
diaminase, increasing endogenous adenosine, a
potent coronary artery vasodilatation. Coffee, tea,
caffeine containing soft drinks or chocolate,
theophylline and aminophylline.
2. Dipyridamole is infused for 4 minutes. Tracer is given
3 minutes after completion of dipyridamole infusion.
Side effects can be reversed with aminophylline.
3. Mild to moderate fixed defect of the anterior wall.
Severe, mostly fixed defect involving the entire
inferior wall but small area of reversibility in the
inferioapical region. Dilated LV.
4. Obtained delayed SPECT to allow additional hepatic
clearance or movement of bowel activity, have the
patient drink water.

Brain scintigraphy

Pearls & Pitfalls


The commonest indications for brain imaging
are perfusion abnormalities (stroke),
dementia (Alzheimers or multi infarct),
epilepsy, brain death, and distinguishing
recurrent tumor from radiation necrosis.

Brain imaging

The radiopharmaceutical Tc-99m HAMPAO,


TC-99m ECD, N-13 PET.

The radiopharmaceutical Tc-99m HAMPAO,


TC-99m ECD are lipophilic, extracted on first
pass and reflect perfusion.
Their uptake is highest in the cortical and
subcortical grey matter.
On imaging, the central area of decreased
activity is primarily white matter and should
not be mistaken for dilated lateral ventricle

Pearls & Pitfalls

Brain imaging

The radiopharmaceuticals Tl-201


and FDG PET are metabolic agents
that show activity in viable
recurrent and persistent tumors but
not in areas of radiation necrosis.
Multi infarct dementia presents
with multiple asymmetric cortical
perfusion defects.

Multiple small perfusion defects


can also occur from cocaine abuse
or vasculitis

Pearls & Pitfalls

Brain imaging

Alzheimer's dementia classically


presents
with
symmetrically
decreased activity in the posterior
parietal-temporal
lobes
with
preserved activity in the calcarine
cortex and basal ganglia and can be
also seen in Parkinsons dementia.
About 30% of Alzheimer's patients
have symmetrically decreased
activity.
Herpes encephalitis can be seen as
increased activity in the temporal
lobe

Pearls & Pitfalls


Epileptic seizure foci show increased
perfusion (Tc-99m HMPAO or Tc-99m
ECD) and metabolism FDG PET during
seizure activity but decreased or normal
activity interictally

Brain imaging

A normal radionuclide angiographic


examination of the brain presents a
trident appearance of intracranial flow in
the anterior cerebral and right and left
middle cerebral territories.

In brain death, there is no obvious arterial


phase (the trident is absent) and only
scalp activity is seen, which is often
accompanied by a hot nose sign.

Pearls & Pitfalls


These studies can also be
performed by Tc-99m HMPAO
or Tc-99m ECD.
Brain imaging

A Diamox challenge study


evaluate cerebral vascular
reserve.
In areas of vascular disease,
regional perfusion worsens
after Diamox compared with
perfusion without Diamox

Pearls & Pitfalls


Common indications for CSF
imaging are for evaluation of a CSF
leak or for differentiating normalpressure hydrocephalus from other
causes of hydrocephalus.
CSF imaging

These study are done with


intrathecal administration of In-111
DTPA
Most CSF leaks occur in the ear,
paranasal sinuses or nose

Pearls & Pitfalls

CSF imaging

Substantial leaks can be


imaged by noting asymmetric
activity around the region of
the ears on the frontal view or
activity in the nose on the
lateral view.
Some leaks are detected only
by removing and counting
cotton pledgets that were
placed in the area of concern

Pearls & Pitfalls

CSF imaging

Cysternography images are


usually obtained anteriorly.
Six hours after injection, these
images normally show a
trident appearance of activity
produced by labeled CSF in the
anterior
interhemispheric,
right and left sylvian cisterns
Any abnormal entry into the
lateral ventricles is seen as
heart shape activity

Pearls & Pitfalls

CSF imaging

Early ventricular entry with


stasis, accompanied by lack of
activity over the superior
surface of the brain after 24 to
48
hours,
support
the
diagnosis of normal pressure
hydrocephalus.
The classical clinical triad of
normal hydrocephalus includes
ataxia,
incontinence
and
dementia

FDG
RADIOPHARMACEUTICALS

F-18 FDG

HOW THE STUDY IS


PERFORMED

IV FDG (10-20 mCi)


Imaged about 45 min later
Patient sit in a quiet, darkened room
EEG monitoring

PATIENT PREPARATION

Fasting except water


Withheld alcohol/caffeine/psychiatric medication
Differ 4-6 weeks after RT

FDG uptake dependent on blood flow & by active cells


FDG is taken up by higher grade tumors
Neoplasia

Most metastasis have increased uptake


Radiation necrosis have little or no uptake

UNDERSTANDING THE
REPORT

Alzheimers dementia= decreased uptake in bilateral posterior


temporoparietal, posterior cingulate gyrus and frontal lobe if more
advanced
Pricks disease = decreased uptake at frontotemporal
Dementia

Lewy bodies = temporoparietal and more occipital


Parkinsons dementia = temporoparietal and visual cortex
Vascular dementia = focal, asymmetric, wedge shape at cortical and
subcortical
AIDS dementia = multifocal cortical at frontal, temporal and parietal

Seizures

Ictal = seizure = hyperperfusion


Interictal = after seizure = hypoperfusion

Diaschisis

POTENTIAL PROBLEMS

phenomenon which abnormal in one segment will decreased uptake in


uninvolved region

False positive

Inflammation / radiation necrosis with gliosis

False negative

Steroid/ high glucose/ RT/ small lesion/ low histologic grade

Thyroid disease

Hypothyroid may effect FDG uptake at parietal and temporal region

Brain perfusion SPECT imaging


RADIOPHARMACEUTICALS

Tc-99m HMPAO / Tc-99m ECD / Xenon-133

HOW THE STUDY IS


PERFORMED

PATIENT PREPARATION

IV RPC (15 30 mCi)


Patient went in the dark room for 5 15 min
For seizure, EEG is compulsory
Fasting except water
Withheld alcohol/caffeine/psychiatric medication
Differ 4-6 weeks after RT
Brain perfusion highly sensitive to blood flow
Cerebrovascular disease

Hemorrhagic stroke cannot be differentiated from other infarct


Lacuna strokes may be too small to detect

Alzheimers dementia= decreased uptake in bilateral posterior


temporoparietal, posterior cingulate gyrus and frontal lobe if more
advanced

UNDERSTANDING THE REPORT

Pricks disease = decreased uptake at frontotemporal


Dementia

Lewy bodies = temporoparietal and more occipital


Parkinsons dementia = temporoparietal and visual cortex
Vascular dementia = focal, asymmetric, wedge shape at cortical and
subcortical
AIDS dementia = multifocal cortical at frontal, temporal and parietal

Seizures

Ictal = seizure = hyperperfusion


Interictal = after seizure = hypoperfusion

Diaschisis

POTENTIAL PROBLEMS

Age of infarct
Luxury perfusion

phenomenon which abnormal in one segment will decreased uptake in


uninvolved region
It is not possible to determine the age of infarct
Radiotracer may be deposited at an infarct due to increased flow, even
the brain cells are actually nonfunctioning. Its seen with Tc-99m HMPAO

Blood Brain barrier protocol

SPECT cerebral perfusion imaging protocol

Brain death scan


RADIOPHARMACEUTICALS
HOW THE STUDY IS
PERFORMED

PATIENT PREPARATION

Tc-99m HMPAO / Tc-99m ECD / Tc-99m pertechnitate / Tc-99m DTPA

- Tc-99m pertechnitate / DTPA bolus (20 mCi) for cerebral blood flow
- Tc-99m HMPAO / ECD (15 30 mCi), delayed images from 20 min to 2 hours post
injection

No specific preparation
Absent of cerebral blood flow

UNDERSTANDING THE
REPORT

POTENTIAL PROBLEMS

Hot nose sign

Barbiturates or
hypothermia

Barbiturate coma, taking phenobarbital or hypothermia

False negative

Scalp blood flow may be increased due to inflammation

Continuum of
brain death

Ineffective blood flow can persist despite true brain death

Diamox Brain stress scan


RADIOPHARMACEUTICALS
HOW THE STUDY IS
PERFORMED

PATIENT PREPARATION
UNDERSTANDING THE
REPORT

Tc-99m HMPAO / Tc-99m ECD


- 1 g Diamox injected
- Rpc (9 mCi) injected 20 min later
- The base line scan can be performed a day before or after
- Contraindication for sulfa drug allergy
- Diamox is to be avoided within 3 days of acute stroke
Carotid artery
stenosis

Decreased flow to the affected region on


Diamox scan compared to base line

Multiinfarct
dementia or
Alzheimers
disease

MID = defect are often pronounced


following Diamox
AD = normal

Side effect of Diamox = vertigo/ tinnitus/nausea/ postural


hypotension
POTENTIAL PROBLEMS
Completely balanced bilateral lesion may give false negative

Thallium-201 or Tc-99m SestaMIBI Brain Scan


RADIOPHARMACEUTICALS

Tc-99msestamibi = localizes in mitochondria by active diffusion across


an interrupted BBB
Thallium 201 = concentrated like potassium in viable tumor, less in
inflammatory and no in necrotic tissue

HOW THE STUDY IS


PERFORMED

- Tl-201 (2 4 mCi) IV , image at 10 30 min and delayed at 2 4


hours
- Tc-99m SestaMIBI (10 30 mCi) IV an image at 10 min to 4 hours

PATIENT PREPARATION

No special preparation
Normal brain

UNDERSTANDING THE
REPORT

High grade gliomas


and lymphoma

Little or no thallium uptake

Increased uptake hot

Infection or radiation No significant increase from background


necrosis
False positive

POTENTIAL PROBLEMS
False negative

There has been reports of thallium uptake with


abscess, certain infections, inflammatory
demyelinating disease and radiation necrosis
SestaMIBI less prone to false positive
If a lesion is necrotic, partially treated, near an
areas of normal uptake in the skull base or
scalp, or below the resolution of imaging

Radionuclide Cysternogram
RADIOPHARMACEUTICALS

HOW THE STUDY IS


PERFORMED

PATIENT PREPARATION

UNDERSTANDING THE
REPORT

Indium-111 DTPA
3 days half life
Study requires 24 72 hours
- LP is performed
- In-111 DTPA (9-18 MBq) into intrathecal
- Image at 1-4 hr, 24 hr, up to 48 72 hr
- Lumbar puncture should be informed to patient before
procedure
- LP and radiotracer administration under fluoroscopic
guidance
Normally radiotracer reaches basal cisterns by 1 hr,
frontal poles and Sylvian fissures by 2 6 hr
Convexity by 12 hr
Sagittal sinus by 24 hr
Lumbar puncture

Extravasation will manifest as persistent


activity at the lumbar puncture site and
slow or no progression to the cranium

Overlapping
patterns

Often there is overlap of finding

POTENTIAL PROBLEMS

Cysternography protocol

CSF leak
RADIOPHARMACEUTICALS

HOW THE STUDY IS


PERFORMED

PATIENT PREPARATION
UNDERSTANDING THE
REPORT

Indium-111 DTPA
3 days half life
- Before the exam, labeled cotton pledgets are placed into the nasal
cavity and/or ears by ENT
- In-111 DTPA is introduced into the thecal sac via LP (9 18 MBq)
- Patient lies supine or in Trendelenberg
- Image at 1 4 hr
- Patient may be asked to Valsalva to increased CSF pressure
- The pledgets usually withdrawn when leak is detected or at 4 24 hr
and weighed and counted for radioactivity
- Plasma sample is collected to calculate a pledget to plasma
radioactivity ratio because CSF is absorbed into the blood stream
and will appear in normal nasal secretions
- Further delayed up to 72 hr
-

Informed for lumbar puncture


A corrected pledget to plasma ratio > 2:1 indicates CSF leakage
Lumbar puncture

POTENTIAL PROBLEMS

Pledgets falling out


Nasal secretion radioactivity

CSF leak protocol

CSF Shunt Patency scan


RADIOPHARMACEUTICALS
HOW THE STUDY IS
PERFORMED

Tc-99m DTPA/ MAA


In-111 DTPA
- Tc-99m DTPA or MAA (0.5 1 mCi) 0r In-111 DTPA (0.25 0.5 mCi)
injected into the shunt
- Image taken at 30 min to 2 hr

PATIENT PREPARATION

Aware type of shunt

UNDERSTANDING THE
REPORT

Normal flow through the tubing should be seen with significant


reservoir clearance by 30 min
Ventriculoperitoneal shunt free distribution within abdominal cavity
If the tip is intra artrial MAA is trapped in the lungs
Hold up of tracer at shunt tip in peritoneum indicates a loculation
Ventricular chemotherapy shunt radiotracer appear at convexities by
24 hr if no obstruction
Unfamiliar with shunt

POTENTIAL PROBLEMS

Peri shunt injection injecting not into reservoir will cause false
positive

Shunt patency protocol

Pricks disease

Generalize cortical hypoperfusion


and hypometabolism favoring
prefrontal regions

Multiinfarct dementia

Scattered foci of decreased cortical


perfusion and metabolism

Alzheimers disease

Hypoperfusion and metabolism in


temporoparietal regions

Huntingtons disease

Decreased glucose metabolism and


perfusion in caudate nucleus and
putamen

Wilsons disease

Severe depression of lenticular


nuclei glucose metabolism

AIDS dementia

Diffuse hypometabolism affecting


subcortical more than cortical gray
matter

Tc-99m HMPAO SPECT for brain in


Alzheimer's
disease
showing
posterior temporoparietal defects

Tc-99m HMPAO SPECT of the brain


in dementia of Lewy bodies showing
posterior perfusion defect with
involvement of occipital cortex

Tc-99m HMPAO SPECT of the brain


in
frontotemporal
dementia
(Pricks disease) of showing
anterior perfusion defect

Tc-99m HMPAO SPECT of the brain


in vascular dementia showing
multiple perfusion defects, with
generally reduced perfusion in the
right internal carotid territory

Language activation H215O study in a


patient with a left hemisphere
glioma.
Activation
during
articulation
(red)
and
verb
generation (blue) is anterior to the
tumor

Coronal a and b 11C methionine PET in


a patient with recurrent glioma in the
left temporal lobe

Brain death

Stroke

Tumor

Trauma

TIA
Cortical cerebral imaging

Epilepsy

Dimentia

AIDS

PSY
Movement disorder

Alzheimer

Schizo
Attention deficit

Huntingtons
chorea

Lewi body

Obsessive compulsive
Pricks disease

Multi infarct

Parkinson

Brain

Blood brain barrier

Brain perfusion

Brain tumors

Metabolism

Cysternography

Tc-99m
glucoheptonate

IMP
I-123 iodoamphetamine

F-18 FDG

F-18 FDG

Tc-99m DTPA

Indium-111 pentreotide

Tc-99m DTPA

Tc-99m ECD
Ethyl cysteinate dimer

Tc-99m sestamibi

Tc-99m HMPAO

Thallium-201

Indium-111
DTPA

CNS

Blood Brain barrier

Diffusible lipophilic

Tc-99m HMPAO

Non diffusible

Tc-99m pertechnetate

Tc-99m ECD
Tc-99m DTPA

F-Flurodopa

FDG

Tc-99m HMPAO

Tc-99m ECD

Accumulation

Frontal
thalamus
cerebellum

parietal
occipital

Extraction

80%

60-70%

Dose remains in blood

1 hour < 12%

1 hour 5%
Good brain to background
ratio

Imaging time

Can image until 24


hours

Superior quality 15-30 min


Suboptimal quality if delayed

Excretion

Renal 40% and GI

Renal and GI

Dementia

Temporoparietal
hypoperfusion

Frontotemporal
hypoperfusion

Multiple
focal defects

Pricks
Alzheimer

AIDS
Pseudodepressive

Parkinson

Lewy body

Chronic alcoholism

Creutzfeldt-Jakob

Schizo
Multi infarct
Progressive
supranuclear palsy

Normo pressure
hydrocephalia

Frontotemporal
degeneration

Concerning Tc-99m HMPAO


A. Tc-99m HMPAO crosses BBB
B. Regional uptake of Tc-99m HMPAO is
proportional to regional cerebral blood flow to
the same area
C. Distribution of Tc-99m HMPAO in the brain si
independent of brain maturation
D. Diffuse lung uptake of Tc-99m HMPAO is seen
only in people with a history of smoking
E. Focal cerebral uptake is seen in herpes simplex
encephalitis

Concerning Tc-99m HMPAO


TTFTT

Brain SPECT
A. The distribution of Tc-99m HMPAO reflects regional
glucose metabolism
B. Tc-99m HMPAO has to be used within 30 min of being
prepared
C. It is helpful to create images in the axis of temporal
lobes when investigating temporal lobe epilepsy
D. Bright light or loud sounds at the time of injection
may alter the distribution of Tc-99m HMPAO
E. It is possible to accurately assess the size of lateral
ventricles with brain blood flow tracer

Brain SPECT
FTTTF

Concerning dementia
A. Increased accumulation of cerebral blood flow tracer
is seen in the frontal lobes in Pricks disease
B. Abnormalities in Alzheimers disease are often
bilateral and predominantly effect temporal and
parietal lobe
C. AIDS dementia complex usually shows no
abnormalities on brain SPECT imaging
D. Typical blood flow changes are seen in the basal
ganglia in asymptomatic patients with Huntingtons
chorea
E. A characteristic pattern of reduced rCBF is seen in
idiopathic Parkinsons disease

Concerning dementia
FTFTF

Concerning brain scintigraphy


A. In the initial stage of cerebral ischemia, regional
cerebral blood flow increases.
B. In the initial stage of cerebral ischemia, regional
oxygen extraction fraction increases.
C. With cerebral infarction, cerebral metabolic rate
for glucose increases.
D. Following cerebral infarction, local cerebral
blood flow increases.
E. In Parkinsons disease, PET perfusion studies
demonstrate decreased perfusion to the basal
ganglia contralateral to the affected limb.

Concerning brain scintigraphy


FFFTF

Concerning brain scintigraphy


A. In Parkinsons disease, there is increased F-18 flourodopa
uptake by the stratum (caudate and putamen nuclei).
B. Concerning brain tumors, an advantage of PET is that it can
image metabolically active cells versus gadolinium
enhanced MRI and contrast enhanced CT, which image
areas where there is blood brain barrier breakdown.
C. Gadolinium enhanced MRI and F-18 FDG PET imaging are
equally capable of distinguishing between recurrent brain
tumor and radiation necrosis.
D. All malignant brain tumors demonstrate hypermetabolism
over F-18 FDG.
E. In differentiating between infectious and neoplastic brain
lesions, the presence of hypermetabolism is not useful.

Concerning brain scintigraphy


FTFFF

Concerning brain scintigraphy


A. F-18 FDG freely crosses the BBB via the same carrier
mediated transport system as glucose
B. Local cerebral metabolic rates of glucose (specific
areas of the thalamus, basal ganglia, lobes) can be
determined through PET studies
C. Approximately 20 30 mCi of FDG PET should be
administered for a brain study
D. A routine FDG study can be performed 5 minutes
after radiopharmaceutical administration
E. External stimuli can increased regional glucose
metabolism within particular areas of the brain

Concerning brain scintigraphy


TTFFT

Concerning brain scintigraphy


A. Global glucose metabolism compared with normal
decreases to a greater degree in multiinfarct
dementia than in Alzheimers dementia
B. Small white matter lacunar infarcts are much more
easily identified with FDG PET than with CT
C. CT can reliably distinguish Alzheimers disease from
multiinfarct dementia
D. In Huntingtons disease, changes in the caudate nuclei
can be observed on PET earlier than the CT
E. Persistent ventricular filling on a radionuclide
cisternogram is diagnostic of communicating
hydrocephalus

Concerning brain scintigraphy


FFFTT

Concerning Tc-99m DTPA in brain


scintigraphy
A. Does not cross the BBB
B. Redistribute over several hours
C. In subacute stroke appears as an area of
increased uptake
D. Is the current agent of choice for
radionuclide cisternography
E. Requires preparation with oral perchlorate to
block the unwanted choroid plexus uptake

Concerning Tc-99m DTPA in brain


scintigraphy
TFTFF

Which radiopharmaceuticals have


been used for blood brain
scintigraphy?

Which radiopharmaceuticals have


been used for blood brain
scintigraphy?
Tc-99m pertechnetate, Tc-99m DTPA and Tc99m GH. The letter two were preferred
because of their faster background clearance,
lack of choroid plexus uptake and lower
radiation dose.

What is the flip-flop phenomenon


seen in the cerebrovascular
scintigraphy and what is the
significance?

What is the flip-flop phenomenon


seen in the cerebrovascular
scintigraphy and what is the
significance?
On the flow phase, parenchymal flow is
delayed on the abnormal side compared with
the contralateral normal side. Thus, as the
normal cortex clears, uptake in the abnormal
side peaks. This may be seen with the high
grade carotid artery stenosis with or without
cerebral infarction. Delayed carotid flow is
seen concomitantly.

Pearl
A hot nose may be seen on the flow phase
images and delayed images as a result of
shunting of the blood from the internal to the
external carotid system that supplies the face
and the nose in patients with severe carotid
stenosis, brain death, psychoactive drug use
and use of other drugs that cause nasal
congestion.

What is luxury perfusion?

What is luxury perfusion?


Increased perfusion may be seen in the region
of an infarct after a recent stroke (1 to 10
days) cause by an uncoupling of blood flow
from metabolism and oxygen demand.

How is brain death diagnosed?

How is brain death diagnosed?


The diagnosis is primarily by clinical. The patient
must be in deep coma with total absence of
brainstem reflexes and spontaneous respiration.
Reversible causes (drug or hypothermia) must be
excluded; the cause of the dysfunction must be
diagnosed (trauma or stroke); and the clinical
findings of brain death must be present for a
defined period of observation (6 to 24 hours).
Confirmatory EEG and radionuclide must be used
to increased diagnostic certainty, but the
diagnosis is primarily clinical. The radionuclide
study is more specific than EEG.

Which radiopharmaceuticals are used


to evaluate brain death and what are
the advantages of each?

Which radiopharmaceuticals are used


to evaluate brain death and what are
the advantages of each?
Tc-99m flow agents such as DTPA are inexpensive.
The 60 second flow study can be interpreted at
the bed side. Because Tc-99m HMPAO and Tc99m ECD fixes in the cortex, delayed static images
can be obtained and interpreted for diagnosis.
The clinician is not dependent on a flow study,
which demand a good bolus and good timing
with proper computer acquisition. However it is
more expensive.

What is the difference in mechanism


of uptake between F-18 FDG and Tc99m cerebral perfusion imaging

What is the difference in mechanism


of uptake between F-18 FDG and Tc99m cerebral perfusion imaging
F-18 FDG is a glucose analog and its uptake
represent regional glucose metabolism. Its
metabolically trapped intracellularly.
Tc-99m HMPAO and Tc-99m ECD are lipid
soluble cerebral perfusion agents taken up in
proportion to regional cerebral blood flow.
They fixed intracellularly.

How can SPECT brain perfusion or PET


FDG imaging can be useful in the
differential diagnosis of dementia?

How can SPECT brain perfusion or PET


FDG imaging can be useful in the
differential diagnosis of dementia?
Multiinfarct dementia is characterized by multiple
areas of past infarct, recognized as areas of decreased
uptake that correspond to the vascular distributions.
Alzheimers disease exhibits characteristic pattern of
bitemporal and parietal hypoperfusion and
hypometabolism.
Picks disease is associated with decreased frontal lobe
uptake.
AISS dementia complex is associated with a pattern of
multifocal or patchy cortical regions of decreased
uptake seen particularly in the frontal, temporal and
parietal lobes and the basal ganglia

What is the purpose of cerebral


perfusion imaging in patients with
seizure and what is the expected PET
or SPECT pattern?

What is the purpose of cerebral


perfusion imaging in patients with
seizure and what is the expected PET
or SPECT pattern?
PET F-18 FDG and SPECT cerebral perfusion
studies can often localize the seizure focus in
patients requiring surgery (typically temporal
lobectomy) for seizure control.
Interictally, a seizure focus shows decreased
metabolism (FDG) on PET and decreased
perfusion on SPECT.
Ictally there is increased activity is seen during a
seizure.

Which radiopharmaceuticals have


been found useful in imaging brain
tumors and what is their clinical
utility?

What is the purpose of cerebral


perfusion imaging in patients with
seizure and what is the expected PET
or SPECT pattern?
F-18 FDG imaging demonstrates increased uptake
in tumors owing to increased glycolysis. Uptake of
FDG is proportional to the malignant grade of
glioblastomas.
SPECT with Tl-201 and Tc-99m sestamibi can be
used in similar manner. Both Tl-201 and PET can
differentiate lymphoma from infection, most
often toxoplasmosis in AIDS patients.
Uptake of Tl-201 or FDG is indicative of
lymphoma.

Name the radiopharmaceutical used


for cysternography and the most
common clinical indication in this
study

Name the radiopharmaceutical used


for cysternography and the most
common clinical indication in this
study
In-111 DTPA. The most common use of this
radiopharmaceutical in modern practice is to
confirm the diagnosis of normal pressure
hydrocephalus (NPH). The next common use is
for (CSF) leaks.

What is the pattern of NPH on


cysternography?

What is the pattern of NPH on


cysternography?
Persistent ventricular filling and evidence of
convexity block.
The symptoms of NPH are incontinence,
dementia and gait disturbance.

Primary brain tumor


1. Low grade gliomas typically have poor or no
uptake
2. Intense uptake in the large temporoparietal
mass
3. Transformation of a low grade to a high grade
glioma
4. No. malignant tumors usually do not have
receptor for binding of RP, which is necessary
before intracellular incorporation

Brain death
1. Flat EEG :hypothermia/ barbiturates/ depressive
drugs
2. Deep coma/ no spontaneous breathing/ no
brain stem reflex
3. Tc-99m DTPA/ Tc-99m pertechnetate or Tc-99m
HMPAO/ Tc-99m ECD
4. Tc-99m DTPA no blood flow to cerebral cortex,
Tc-99m HMPAO shows salivary gland/ normal
brain perfusion cerebral cortical activity seen

Cerebral infarct
1. F18- FDG is dependent on glucose metabolism
2. SPECT Tc-99m HMPAO/ ECD are cerebral
perfusion agents that are lipid soluble, distribute
according to blood flow (gray to white matter
ratio, 3:1 to 4:1) and fix intracellularly
3. Wedge shape severe decreased metabolism in
the left posterior parietal region
4. Cerebral hemorrhage/ infarct/ neoplasm

Seizure disorder
1. Decreased metabolism in the left temporal lobe.
2. Temporal lobe infarct, benign mass or lower
grade tumor, post RT changes, interictal left
temporal lobe seizure focus.
3. Interictal left temporal lobe seizure focus.
4. Conformation of the location of the seizure
focus in a candidate for temporal lobe
lobectomy. Study is an alternative to surgical
depth electrode placement.

Alzheimers disease
1. Multiinfarct, Alzheimer's disease, AIDs related,
substance abuse, alcoholism, Parkinsons
disease, Pricks, Creutzfeldt-Jacob disease,
depression, metabolic.
2. Diagnostic pattern using Tc-99m HMPAO, ECD or
FDG PET: multiinfarct dementia or Alzheimers.
3. Hypometabolism (decreased FDG uptake) of the
posterior parietal and temporal lobes bilaterally
and to a lesser extent the frontal lobes.
4. Alzheimers disease > 80%

Pricks disease
1. Alzheimers disease, multiinfarct, late stage
Parkinsons disease, metabolic, drug related and
depression.
2. The lipophilic Tc-99m HMPAO or ECD, cross the
intact BBB and have rapid intracellular uptake in
proportion to cerebral blood flow. They are fixed
intracellularly. Subsequent imaging provide a
snapshot of the blood flow pattern at the time
of injection.
3. Alzheimers, multiinfarct or Pricks disease
4. Decreased blood flow in the frontal cortex
bilaterally as a result of frontal lobe dementia.

Brain lymphoma vs. infection in AIDS


1. Tumor, especially lymphoma, versus infection
e.g..: toxoplasmosis
2. Increased FDG uptake in the medulla
oblongata
3. Consistent with malignancy
4. Yes, with Tl-201 or Tc-00m sestaMIBI

Herpes Encephalitis
1. Increased uptake in the temporal lobe
2. Increased blood flow in this region
3. Seizure focus (ictal injection), infection or
tumor
4. Herpes encephalitis

Normal pressure hydrocephalus


1. In-111 DTPA administered intrathecally by
lumbar puncture
2. Ventricular reflux and convexity block with no
flow over the cerebral hemispheres above
the sylvian fissure
3. Communicating hydrocephalus, in this case is
normal pressure hydrocephalus
4. Triad of dementia, ataxia and incontinence

Huntingtons disease
1. Hypometabolism of the basal ganglion
2. Progressive
motor
abnormalities
of
involuntary choreiform movements and
akinetic rigidity with progressive cognitive
deteriotion
3. Neuronal degeneration in the striatum, with
the caudate more involved than the putamen
4. Huntingtons disease

CSF leak
1. In-111 or Tc-99m DTPA
2. Intrathecal injection
3. Activity in the region of the nose, indicating
CSF leak, probably at the cribriform plate
4. Anterior views. With the use of nasal
pledgets placed in the superior, middle and
inferior nasal turbinates

Thyroid

Pearls & Pitfalls

Thyroid

A normal 24-hour iodine


uptake in most lab ranges
between 10% and 30% to 35%.
Common
indications
for
radionuclide thyroid imaging
are to differentiate between
various types of hyperfunction
(Graves disease, toxic MNG or
autonomous adenoma) and to
assess nodularity (cold or hot)
and ectopic tissue.

Pearls & Pitfalls

Thyroid

Salivary glands are usually seen


on Tc-99m pertechnetate scan
unless the patient has Graves
disease. They are not seen on
an I-123 scan.
A lingual thyroid is usually
located in the midline at the
base of tongue, with no thyroid
seen in the normal location.
The ectopic gland is often
hypofunctional.

Pearls & Pitfalls

Thyroid

A thyroid gland with an


organification defect is usually
seen as a normal gland on a Tc99m pertechnetate scan but
manifests no activity on an
iodine scan in a child with a
high TSH level.
A large gland with intense
homogenous activity is usually
Graves disease.

Pearls & Pitfalls

Thyroid

A large gland with patchy


activity is usually a MNG but
could
also
be
chronic
thyroiditis or an infiltrative
process.
Most hot nodules are benign
hyperfunctioning adenomas.
They can be single or multiple
and can suppress the normal
portions of the glands.

Pearls & Pitfalls

Thyroid

Subacute thyroiditis classically


presents with a markedly
depressed radioiodine uptake
and nonvisualization of the
gland in a patient with
thyrotoxicosis and a tender,
swollen thyroid.
Chronic thyroiditis can mimic
numerous thyroid conditions
but on imaging is usually
patchy and decreased in
activity.

Pearls & Pitfalls

Thyroid

Thyroid cancer is usually a


single focal cold lesion and
only rarely is seen to be diffuse
or multifocal on thyroid scan.
Thyroid cancer can concentrate
Tc-99m sestamibi and persist
on delayed images. They are
typically cold on Tc-99m
pertechnetate scans.

Pearls & Pitfalls

Thyroid

Activity in the bladder,


stomach and bowel and diffuse
activity in the liver is usually
normal on a WBS of I-131.
After successful RAI ablation of
residual thyroid tissue, serum
Tg are sensitive to detect
recurrent thyroid cancer.

Thyroid
function
Thyrotoxicosis

Increased
thyroid
uptake
Graves disease

Normal
thyroid
uptake
Antithyroid drugs
PTU/CBZ

Hashitoxicosis

Decreased thyroid
uptake
Expended iodide pool
Thyrotoxic phase of
subacute thyroiditis
Thyrotoxicosis factitia
Antithyroid drugs
Struma ovarii

Euthyroid

Rebound after
antithyroid drug
withdrawal
Recovery from
subacute
thyroiditis

Hypothyroid

Hashimotos
disease

Hashimotos
disease after RAI
therapy
Subacute
thyroiditis

Hypothyroidism:
primary/secondary

The following may cause a solitary cold nodule


in thyroid sca

Thyroid adenoma

Solitary cold
nodule in
thyroid scan

Parathyroid adenoma
Lymphoma
TB
Colloid cyst
Metastatic carcinoma

Primary thyroid
carcinoma

Percentage to become thyroid carcinoma

Nodules

Cold

15 20%

Indeterminate

15 20%

MNG

5%

Hot

<1%

20% malignant = trapping but not organified

Discordant
nodule
(thyroid
adenoma)

Hot nodule of Tc-99m

Cold nodule of I-131

Small amount of functioning thyroid tissue


with intact trapping and organified

Reverse discordant
nodule
(thyroid adenoma/
Hashimotos
thyroiditis/
thyroglassal duct)

Cold nodule of Tc-99m

Hot nodule of I-131

Owls eye sign

Photopenic area in center of


functioning autonomous thyroid
nodule = cystic degeneration within
solitary functioning thyroid nodule

Fish eye sign

Central core of functioning tissue


surrounded by a rim of decreased
uptake = functioning adenoma with
cystic degeneration at periphery

Causes of
hyperthyroidism
Graves Disease
Toxic multinodular Goitre
Toxic adenoma

Thyroiditis
Hashimotos
Subacute (De Quervains)
Silent (post partum)

Antithyroid drugs
Surgery
Radioiodine

Self limiting
Symptomatic

Factitious

Iodine and iodinecontaining drugs agents


Drugs - amiodarone
Trophoblastic disease

Treatment of causes

Treatment Choices
Antithyroid drugs (thionamides)
inhibit thyroxine production
immunomodulation
Surgery
reduce thyroid bulk to decrease thyroxine
production

Radioiodine
same principle as surgery

Physical Characteristics of 131Iodine

reactor produced
T 8.02 days
eta and gamma emitter
eta - 606 keV max, 191 keV mean
Beta-particle emission tissue range
0.8mm (local therapeutic effect)
gamma - 364 & 637 keV
Clinical Form : Sodium Iodide

Physiological Basis of Radioiodine Treatment


Administration of oral
Sodium iodide

131I

concentrated by the cells

High local radiation (beta


particle emissions)
cytotoxic effect

Reduce thyroid mass/size

Decreased thyroxine
production

Contraindications
Pregnant women
a. cross the placenta accumulate in the
fetal thyroid gland severe neonatal
hypothyroidism
b. irradiation risk of thyroid cancer
c. retained activity in maternal bladder direct
exposure to the fetus

Lactating mother

Patients Preparation
No solid foods or drink dairy products for at least two
hours before and after treatment.
Low iodine diets for about 3 - 7 days
GD patients should be counseled regarding the risks of
ophthalmopathy.
Render patients euthyroid before treatment with 131I
or thyroidectomy.
- older patients (> 5060 years old)
- very thyrotoxic patients (symptomatic)
- patients with cardiac problems.

Patients Preparation
Follow drug interactions notes
Antithyroid drugs - resumed 3 to 10 days
following
treatment, or earlier, if clinically necessary
Beta blockers medications - to reduce symptoms
which may occur during treatment:
propranolol, 80160 mg/day, or
atenolol, 50150 mg/day, or
diltiazem 30 180mg/day (bronchospasm)

Doses
Two common approaches:
1. Fixed Dose
2. Calculated Dose - based on the size of the
thyroid & percentage uptake at 24 h.

Doses
Although treatments based on dose calculations
appear efficacious, they have not proven superior to
the use of empirically selected administered
activities.
Fixed dose advantages simple & successful
outcome in an acceptable number of patients.

Initial Therapy
Graves
Disease

-block
Antithyroid

Subacute
Thyroiditis

-block
Steroids until
resolution of
symptoms

Toxic adenoma
(Plummer Disease)

-block
Antithyroid

Hyperfunctioning
Multinodular
Goiter

-block
Antithyroid

Graves Disease

-blockers
Carbimazole or PTU

Modulate therapy
until normalization
of TSH, FT3, FT4
(18 24 months)

Withdrawal of -blocker
Progressively reduce
Carbimazole or PTU

Check TSH, FT4, FT3


Every 3 month

Graves Disease - 1 relapse

-blockers
Carbimazole or PTU

Inform the pt on alternative


advantages and limitations of
13I I therapy and surgery
Needs of substitutive Tx in
both cases

Pts age and gender

Size of goiter
Tracheal compression
Narcosis risks
Complications of thyroid surgery

Fixed dose
Vary but are commonly in the range of
185555 MBq (515 mCi)
Depending on the size of the gland

Calculation of Administered Activities for Treatment of Benign Thyroid


Diseases

Use the following equation to correct for 24-hour RAI


uptake in target thyroid tissue:

Administered activity = Thyroid tissue mass (g) x Activity per g


tissue/RAI uptake at 24 hours
with RAI uptake expressed as a fraction of 100% uptake.

Long Term Therapy


Toxic adenoma
(Plummer
Disease)

131I

Hyperfunctioning
Multinodular
Goiter

THERAPY vs. SURGERY

Pts age and gender


Size of goiter
Tracheal compression
Narcosis risks
Complications of thyroid surgery

Toxic Nodular Goitre


Definitive treatment is more commonly accomplished
with RAI or surgery
> RAI resistant than GD
Large doses
Standardized administered activities:
740 1110 Mbq (20 - 30 mCi)
Administered less than 370 MBq (10mCi)
common Tx failure
(Estour B, Millot L, Vergely N, et al. Efficacy of low doses of radioiodine in the treatment of autonomous
thyroid nodules: importance of dose/area ratio. Thyroid 1997; 7(3):357361.)

Large solitary nodules percutaneous ethanol injection.


(Del Prete S, Caraglia M, Russo D, et al. Percutaneous ethanol injection efficacy in the treatment
of large symptomatic thyroid cystic nodules: ten-year follow-up of a large series. Thyroid 2002;
12(9):815821.)

Total thyroidectomy for clinically benign disease of


the thyroid gland

Incidence of:
temporary recurrent laryngeal nerve palsy
2.3%
temporary hypoparathyroidism
14.4%
Permanent recurrent laryngeal nerve palsy
1.1%
permanent hypoparathyroidism
2.4%
Neither the initial clinical diagnosis nor a history of previous
treatment significantly influenced the rate of complications.
Bron LP, O'Brien CJ. Br J Surg. 2004 May;91(5):569-74.

Euthyroid Multinodular Goitre


Surgery first-line treatment esp. for enlarged goiter
Preferable for patients > 65 years old
> 90% of patients demonstrate a decrease in goiter size
-

Average reduction of 40% at 1 year .

(Freitas JE. Therapeutic options in the management of toxic and nontoxicnodular goiter.
Semin Nucl Med 2000; 30(2):8897. )

A small percentage may developed transient


hyperthyroidism
Approximately 4% developed autoimmune thyroid
disease.
(Huysmans AK, Hermus RM, Edelbroek MA, et al. Autoimmune hyperthyroidism occurring late after radioiodine
treatment for volume reduction of large multinodular goiters. Thyroid 1997; 7(4):535539.)

RAI > effective than L-thyroxine suppression for reducing


the size of nontoxic goitre

Goals and outcomes


80% response rate should be expected.
Primary goal of treatment is to resolve
hyperthyroidism
Hypothyroidism is preferable to persistent
hyperthyroidism - if not optimally treated, may
produce significant morbidity.
Hypothyroidism tends to occur more frequently
in patients with small thyroid glands & lower 24hour uptake measurements
Significantly reduce thyroid gland size in patients
with GD - 50% to 80% reduction in gland volume

Outcomes
Euthyroid
Hypothyroid
transient

permanent

early
late

Hyperthyroid

Side Effects
Generally it is mild, infrequent and self-limiting.
General
Thyroid tenderness
Salivary gland swelling
Nausea

NSAIDs

Vocal cord paresis extremely rare


Hormonal related
Transient hypothyroidism, permanent hypothyroidism
Transient hypoparathyroidism
Exacerbation of hyperthyroidism Beta blockers
Thyroid storm is uncommon

Side Effects
Severe side effects > likely in patient with large goitre risk of
tracheal compression.

Ophthalmopathy
Ophthalmopathy may be particularly severe in 3%
to 5% of patients with GD.
Progression of ophthalmopathy - approximately
15% of patients especially:
- who smoke
- have pre-existing eye disease
- high levels of TSH-receptor antibody
- severe manifestations of thyroid disease

Side Effects
Prednisone, 0.40.5 mg/kg per day,
beginning immediately after radioiodine
treatment, continued for one month,
and then tapered over three months,
has been shown to be effective in a
randomized controlled trial.
(Bartalena L, Marcocci C, Bogazzi F, et al. Relation between therapy for hyperthyroidism and
the course of Graves ophthalmopathy. N Engl J Med 1998; 338(2):7378. )

Referral
to
recommended.

Ophthalmologist

is

Side Effects
Cancer Risk
remains controversial
Ron et al, a study of >35,000 hyperthyroid
patients found that the incidence of
thyroid cancer in RAI-treated patients over
a 27-year period was not significantly
different from its incidence in the general
population.
Genetic Effects to Offspring
no evidence that exposure to radioiodine
affects the long-term outcomes of
subsequent pregnancies and offspring.

Patients Instruction & Precautions


First 72 hours after treatment:
Drink plenty of fluids.
Do not spend prolonged periods of time closer than 3 feet to any
adult, or within the same room as any child.
For the next 47 days:
Void as often as possible. Flush toilet twice after use.
Wash hands thoroughly and routinely.
Do not share eating utensils or towels. Wash utensils or clothing
separately.
Avoid close contact with children and pregnant women (not
closer than 2 feet) for long periods of time.
Sleep in a separate bed.
Avoid kissing and sexual intercourse.

Follow up
To assess the need for
- further 131Iodine treatment
- L-thyroxine replacement
Symptoms of uncontrolled hyperthyroidism
should be described, and patients should be
informed to seek medical attention if such
symptoms occur.
4 to 6 weeks and at regular intervals
thereafter.

Children with hyperthyroidism


Incidence of side effects from antithyroid
medications is higher & less consistent in taking the
medications
remission rates are lower.
Surgery - may have the highest cure rates but
thyroidectomy is technically more difficult in young
children.
Several
authorities
have
promoted
the
administration of 131I earlier in the management of
pediatric patients and even as the primary
treatment.
Children as young as 3 year old can be treated.

Chronic Renal Failure and Dialysis


Patients
Holst et al. reviewed the medical literature
and concluded that:
131I

dose does not need to be adjusted.


However, recommended 131I administration
as soon as possible after dialysis and a
delay in subsequent dialysis until the
maximum 131I uptake has occurred in the
thyroid.

Temperature

CNS

GI/liver dysfunction

Cardiovascular dysfunction

Heart failure
Precipitant history

37.2-37.6

37.7-38.2

10

38.3-38.7

15

38.8-39.3

20

39.4-39.9

25

>40

30

Mild agitation

10

Moderate delirium/psychosis

20

Seizure/coma

30

Diarrhea/vomiting/abd pain

10

Unexplained jaundice

20

99-109

110-119

10

120-129

15

130-139

20

>140

25

Pedal edema

Bibasilar rates

10

APO

15

AF

20

negative

positive

10

Score >45 suggestive of thyroid storm, 25 44 supports the diagnosis and < 25 unlikely thyroid storm

Molecular genetic of thyroid


carcinoma

Introduction
Thyroid cancer- most common malignant
tu of endocrine system & accounts for 1%
of all newly diagnosed cancer cases.
Most thyroid cancers originate from
thyroid follicular cells .
Type of thyroid malignancy :
papillary ca 80%,
follicular ca 15% may be of conventional
or oncocytic (Hurthle cell) type.[4]
Follicular ca develop either from preexisting benign follicular adenomas or
directly, bypassing the stage of adenoma.

2%

3%

Schematic representation of thyroid ca origin and its putative progression. Oncocytic adenoma
and ca are considered to be variants of follicular adenoma and ca. Papillary ca may be of the
classical type or manifests as one of its variants, including oncocytic variant of papillary ca

Papillary Ca
Activating mutations of BRAF, RET or RAS genes
seen in 70% papillary ca.
Genes mutations rarely overlap in the same
tumor, suggesting that activation MAPK signaling
is essential for tumor initiation
Alteration of a single effector of the pathway is
sufficient for cell transformation

BRAF

Mol p/way in thyroid papillary ca & typical microscopy & clin feat of tu assoc with specific
mutations

BRAF mutation
Involve nucleotide 1799 and result in a val-to-glut
substitution at residue 600 (V600E).
Other mech include K601E point mut, small inframe insertions or del surrounding codon 600,[14
16] & AKAP9-BRAF rear which is more common in
papillary ca a/w radiation exposure.[17]

With BRAF mutation.


Extrathyroidal extension , advanced tu stage
@ presentation, tu recurrences & lymph node
or distant mets.
BRAF V600E is independent predictor of tu
recurrence even in stage 1-2.
Decreased ability of tu to trap radioiodine &
dysregulation of function of Na iodide
symporter ( NIS) & other genes metabolizing
iodides in follicular cells

BRAF mut as therapeutic target..


BRAF inhibitors like BAY 43-9006, a multikinase
inhibitor with potent activity against RAF & other
protein kinases.
BAY 43-9006 effectively blocks the wild-type
BRAF and the mutant V600E BRAF kinase
activity.[13,31]
Sudies showed that BAY 43-9006 inhibit BRAF
signaling and growth of all thyroid cancer cell
lines carrying mutant BRAF and it impair the
growth of the cell line xenografts in nude mice.[32]

RET/PTC rear in papillary thyroid ca

RET/PTC
Fusion between the 3-portion of the RET
receptor tyrosine kinase gene and the 5-portion
of various genes.
Two most common rear types, RET/PTC1 and
RET/PTC3, are paracentric inversions, because
both RET and its respective fusion partner, H4 or
NCOA4 (ELE1), reside on the long arm of
chromosome 10.[3639]

RET/PTC 1
Two most common rear types RET/PTC 1 & RET/PTC 2 are
paracentric inversions bcoz both RET & its respective fusion
patner, H4 or NCOA4(ELE1), reside on the long arm of chr 10

RET/PTC 3

RET/PTC 3

RET/PTC
Most of these rare RET/PTC seen in papillary
ca fr pts with a h/o environmental or
therapeutic exposure to ionizing radiation,[40
46] & in children & young adult.
In papillary ca assoc/w exposure to ionizing
radiation (i.e. post Chernobyl), RET/PTC1 a/w
classic papillary histology, RET/PTC3 more
common in solid variant

RET/PTC 2 t(10;17)(q11.2;q23)

RET/PTC2 and nine more recently identified types of


RET/PTC are all interchromosomal translocations

RET/PTC 2 t(10;17)(q11.2;q23)

RET/PTC 5 t(10;14)(q11.2q32)

RET/PTC 5 t(10;14)(q11.2q32)

RET/PTC 6 t(7;10)(q32;q11.2)

RET/PTC 6 t(7;10)(q32;q11.2

RET/PTC 7 t(1;10)(p13;q11.2)

RET/PTC 7 t(1;10)(p13;q11.2)

RET/PTC 8 t(10;14)(q11.2;q22.1)

RET/PTC 8 t(10;14)(q11.2q22.1)

RET/PTC 9 t(10;18)(q11.2;q21)

RET/PTC 9 t(10;18)(q11.2;q21)

PCM 1/ RET t(8;10)(p21;q11.2)

PCM 1/ RET t(8;10)(p21;q11.2)

RFP/RET t(6;10)(p21;q11.2)

RFP/RET t(6;10)(p21;q11.2)

RET/PTC
ELKS-RET and HOOK3-RET fusions identified in
papillary ca with no apparent h/o of radiation
exposure.[47,48]

ELKS/RET - t(10;12)(q11.2;p13.3)

ELKS/RET - t(10;12)(q11.2;p13.3)

HOOK3/RET t(8;10)(p11.21;q11.2)

HOOK3/RET t(8;10)(p11.21;q11.2)

All fusions leave the tyrosine kinase domain of


the RET receptor intact and enable the
RET/PTC oncoprotein to bind SHC and activate
the RAS-RAF-MAPK cascade.[49]

RET/PTC mut as therapeutic target.


ZD6474, orally active low m.w receptor kinase
inhibits VEGVR-2 & blocks RET tyrosine
kinase.
ZD6474 block phosphorylation and signalling
from RET/PTC3 in vitro, induce growth arrest
of human papillary ca cell lines carrying
RET/PTC1 and prevent growth of RET/PTC3transformed fibroblasts in nude mice.[64,65]

RET/PTC mut as therapeutic target..


SU 12248 (multikinase inhibitor - sunitinib)
effectively inhibit signalling from RET/PTC
kinase in the experimental models and has
been tested in Phase II clinical trial in
radioiodine-refractory,
unresectable
differentiated thyroid cancer.[66]

RAS
RAS genes (HRAS, KRAS and NRAS) encode G-proteins
important in the intracellular transduction of signals arising
from cell membrane receptors.
In its inactive state, RAS protein is bound to GDP.
Upon activation, it releases GDP and binds GTP, activating
MAPK and PI3K/AKT.

Normally, the activated RAS-GTP protein becomes


quickly inactive due to its intrinsic GTPase activity and
the action of cytoplasmic GTPase-activating proteins.

Point mut in the discrete domains of the RAS


gene either increase its affinity for GTP (mut in
codons 12 and 13) or inactivate its
autocatalytic GTPase function (mut in codon
61).
As a result, the mutant protein becomes
permanently switched in the active position
and continuously activates its downstream
targets

Follicular Ca & Adenoma

RAS
PAX8-PPAR
PIK3CA
PTEN

Mol p/way in follicular thyroid ca

Foll Ca & Adenoma - RAS

Mutations found in 40 to 50% conventional


foll ca & in 20 to 40% adenomas.
Hotspots NRAS codon 61 & HRAS codon 61.
Mutations seen in 15 to 25% oncocytic cell ca.
Mut correlates with tu dedifferentiation &
metastasis

Foll Ca & Adenoma - PAX8-PPARy


Due to t(2,3)(q13;p25)
Leads to fusion of PAX8 & PPARy genes.
35% of conventional foll ca & lower in hurtle
cell ca.
A/w younger age, smaller size, solid/nested
growth pattern & vas invasion
If seen in adenoma ? Insitu (preinvasive foll
ca, ?invasion overlooked in histo)

Foll Ca & Adenoma other mut


P13K/AKT pathway mut seen in 6-13% foll ca
& 0-6% of foll adenoma
In exons 20 & 9 of PIK3CA genes.
LOH in regions harboring different tu supp
genes..
Del of 2p, 3p, 9q,10q,11p,15q & 17p.
LOH assoc/w aggressiveness & recurrences

Oncocytic Ca

LOH on chr 3q,18q,1p,2p,8q,14q


Mut of GRIM-19 in 15% oncocytic ca
GRIM-19 encodes a protein linked to retinoidinterferon-induced p/way of cell death &
involved in mitochondrial met.

Poorly diff & Anaplastic Ca

TP53
CTNNB1 ( Beta catenin)
RAS
BRAF
PIK3Ca
PTEN

Poorly diff Ca & Anaplastic Ca-TP53


Point mut of TP53 are late events
Most involve exons 5-8 of gene & alter DNA
binding properties
Assoc/w accelerated cell growth & progressive

This is the neck after removal


of the cancerous lymph nodes

Medullary Thyroid Ca
RET is activated by point mutation vs chr rear in
papillary ca.
Germline mutations in specific functional regions of
RET found in almost all pts with familial medullary ca.
MEN 2A & familial medullary ca, mutations are
typically located in the extracellular domain, within the
cysteine-rich region.[124]
90% of MEN 2A mutations affect a single codon 634,
whereas in familial medullary ca they are more evenly
distributed along the cysteine rich region.[125]

In MEN 2B, the majority of germline


mutations occur in codon 918 in the
intracellular tyrosine kinase domain of RET.
This alter the substrate specificity of RET
kinase, resulting in phosphorylation of
unusual intracellular proteins.[126]

This CT scan shows a thyroid cancer


tu in the throat, encircling,narrowing,
& displacing the windpipe(trachea)

Sporadic medullary ca, somatic mutations of


the RET gene in 2080% of cases.[127,128]
Mut affect codon 918 mostly, some have
heterogeneous distribution within the tu &
detected only in a subset of metastatic
nodules.[128]

RET kinase inhibitors ZD6474 and SU12248


(sunitinib), now in Phase II clinical trials in pts
with familial & sporadic medullary thyroid
cancer.[63,66]

Summary
FNA dx of thyroid nodules can be improved by
testing for BRAF, RET/PTC and other mutations.
BRAF, RET/PTC and RAS mutations correlate with
specific phenotypical features of papillary ca.
BRAF is a reliable diagnostic marker for
malignancy and an independent prognostic
marker for tumor recurrence and more
aggressive behaviour of papillary carcinomas.
These are lymph nodes that
received cancer cells fr the
primary ca in the thyroid gld

Summary
PAX8-PPAR & RAS mut correlate with specific
phenotypical feat of follicular ca.
Limitations..Papillary
ca
(~30%),
conventional follicular cas (~20%) & oncocytic
follicular ca (>50%) do not harbour any of the
known mutations.

Black discoloration of cancerous


lymph nodes.Papillary ca is
sometimes black in color especially
afer having spread to lymph nodes

In the investigation of hyperthyroidism


A. In Graves disease, uptake of Tc-99m is usually
uniform
B. If, after 2 years following I-131 treatment for Graves
disease the patient is still euthyroid, the incidence of
subsequent development of hypothyroidism is
negligible
C. A normal isotope scan exclude Graves disease
D. Hypothyroidism following I-131 treatment of a
solitary hyperfunctioning nodule is more common
than in Graves disease
E. Low tracer uptake into the thyroid gland may occur in
untreated hyperthyroidism

In the investigation of hyperthyroidism


TFFFT

The following are true


A. About 25% of toxic nodules are palpable
B. The scan appearances of Graves disease may be
confused with toxic MNG
C. Thallium-201 may be used to demonstrate
thyroid tissue in which uptake but not metabolic
activity has been diminished
D. Almost 10% of patients with Graves disease will
relapse following a prolonged course of
antithyroid drugs
E. RAI therapy for thyrotoxicosis is contraindicated
in pregnancy

The following are true


FTTFT

Concerning thyroid imaging in


thyroiditis
A. Thyroid uptake usually diffusely decreased in
acute(supurative) thyroiditis.
B. Uptake in the early phase of subacute (de
Quervians)thyroiditis is usually increased
C. Scan appearances in Hashimotos thyroiditis may
be difficult to distinguish from Graves disease
D. Radiation induced thyroiditis is commonly seen
following external radiation therapy
E. RAI uptake in Reidels thyroiditis is increased

Concerning thyroid imaging in


thyroiditis
FFTFF

In the investigation of solitary thyroid


nodules
A. A single warm nodule is likely to be an adenoma
B. The risk of carcinoma in a hot nodule is 10%
C. The risk of carcinoma in a cold nodule is at least
10%
D. The risk of carcinoma in the dominant nodule
within MNG is approximately 1%
E. A history of childhood of head and neck
irradiation substantially increases the risk of
malignant change in solitary thyroid nodule

In the investigation of solitary thyroid


nodules
TFTFT

The following are true


A. The commonest site for ectopic thyroid tissue is
the base of tongue
B. I-123 is superior than Tc-99m in confirming the
presence of a lingual thyroid gland
C. Isotopes of Iodine are superior than Tc-99m for
localizing retrosternal thyroid tissue
D. Absence of activity below the sternal notch
excludes an intrathoracic goitre
E. Ectopic thyroid tissue may be found in up to 50%
of cases of neonatal hypothyroidism

The following are true


TTTFT

Concerning malignancy of the thyroid


gland
A. The majority of tumors are follicular type
B. Most tumors are well differentiated
C. Most thyroid tumors are detected following
demonstration of an area of increased uptake
relative to normal thyroid on Tc-99m scan
D. Anaplastic carcinoma never takes up I-131
E. Screening of relatives is mandatory in
anaplastic carcinoma

Concerning malignancy of the thyroid


gland
FTFTF

The following may cause a solitary cold


nodule in thyroid scan
A.
B.
C.
D.
E.

Lymphoma
Parathyroid adenoma
TB
Wegners granulomatous
Metastatic adenocarcinoma

The following may cause a solitary cold


nodule in thyroid scan
TTTFT

The following are true


A. The patient with toxic MNG presenting with pressure
symptoms should be treated with RAI first instance
B. Patients receiving I-131 for thyrotoxicosis should take
approximately 1 week off work following treatment
C. The absolute of thyroid hormones and TSH before
treatment with RAI is a useful predictor of the
development of hypothyroidism in Graves
D. A rise of thyroglobulin level following complete
ablation for thyroid carcinoma is indicative of
recurrent malignant disease
E. Uptake of RAI into thyroid carcinoma metastases has
prognostic significance

The following are true


FTFTT

What is the origin of lingual and


sublingual thyroid tissue?

What is the origin of lingual and


sublingual thyroid tissue?
The main thyroid anlage begins as a
downgrowth from the foramen cecum.
Thyroid tissue may be seen anywhere along
the tract of the thyroglassal duct from the
foramen cecum to the usual location of the
gland.
However with lingual thyroid tissue, there is
usually a failure of normal development and
no tissue in the normal location of the thyroid

What is meant by the organification of


iodine?

What is meant by the organification of


iodine?
In thyroid metabolism, iodide is oxidized to
iodine and incorporated into tyrosine to form
either monoiodotyrosine or diiodotyrosine.
A deficiency in peroxidase which catalyzes the
reaction,
is
cause
of
congenital
hypothyroidism.

What is the difference in mechanism


of thyroid uptake between Tc-99m
pertechnetate and radioiodine?

What is the difference in mechanism


of thyroid uptake between Tc-99m
pertechnetate and radioiodine?
Radioiodine is taken up or extracted (trapped)
by the thyroid follicular cell and organified,
binding to tyrosine residues on thyroglobulin
and stored in colloid in the follicle.
Tc-99m pertechnetate is trapped but not
organified.

Which are common causes of falsely


low thyroid uptake?

Which are common causes of falsely


low thyroid uptake?
Thyroid hormone
Iodine containing drugs
Recent administration of IV iodine contrast.

What is the difference between


thyroid scan and thyroid uptake?

What is the difference between


thyroid scan and thyroid uptake?
A thyroid uptake is usually a nonimaging study
using gamma detector probe.
Thyroid scan results from gamma camera
imaging.

How can a thyroid uptake test


differentiate two most common causes
of thyrotoxicosis, Graves and subacute
thyroiditis?

How can a thyroid uptake test


differentiate two most common causes
of thyrotoxicosis, Graves and subacute
thyroiditis?

In the initial phase of subacute thyroiditis, thyroid


hormone are released from the inflamed gland
causing thyrotoxicosis.
Due to pituitary feedback, TSH is suppressed.
Radioiodine or Tc-99m uptake requires TSH
stimulation.
Thus, the uptake of radioiodine or Tc-99m
pertechnetate is low or suppressed.
With Graves
disease, TSH is suppressed,
however the gland is autonomous and the uptake
is high.

What is the mechanism of action of


antithyroid drugs PTU and CBZ?

What is the mechanism of action of


antithyroid drugs PTU and CBZ?
Both PTU and CBZ are thiourea antithyroid
drugs that block the organification of iodine

Cold thyroid nodule


1. IV Tc-99m pertechnetate, 140 keV, 6 hours;
oral sodium I-123, 159 keV and 8 hours.
2. A single cold nodule has a 15% to 20%
chance of malignancy.
3. FNA
4. Pinhole collimator 4 to 6 mm

Hyperthyroidism/ thyroiditis
1. Graves disease, toxic nodules, thyroiditis,
iatrogenic thyroid hormone ingestion, iodine
induced, trophoblastic tumors, Hashitoxicosis
and struma ovarii.
2. Aid in the DD of hyperthyroidism
3. A nonimaging gamma probe obtains
counts/time from neck and a phantom counting
activity equal to the orally administered dose to
convert to gamma probe counts Ci % RAIU =
neck (Ci) divided by the total administered
dose (Ci) after background correction.
4. Subacute thyroiditis based on the history of
neck tenderness, laboratory finding and RAIU.

Graves disease
1. Scan appearance may be similar. With the large goiter the
scan often has a plumper appearance with convex
borders. The pyramidal lobe may be seen as in this case.
2. Surgery is seldom performed because of the high risk. PTU
and CBZ sometimes are used initially who require cooling
down most of the patients are treated with RAI after 6 to
12 months of antithyroid medication. Many patients are
treated initially with I-131.
3. I-131 uptake (10 Ci), I-123 scan and uptake (300 Ci) and
Graves disease therapy; I-131 (5-15 mCi)
4. Short term; occasional exacerbation of hyperthyroidism
and thyroid storm. Long term; hypothyroidism. There is no
increased incidence of secondary cancers or reduction in
fertility.

I-131 star artifact


1. A. Post therapy I-131 WBS shows intense uptake
in the neck with a star effect, diffuse liver
activity and bladder clearance. B, Pinhole image
of the neck with three foci of uptake.
2. Septal penetration of high energy of I-131
gamma rays through the collimator septa.
3. Pinhole collimator centered on the thyroid.
4. Radiolabeled thyroid hormone is metabolized in
the liver. This usually seen only on the post
therapy scan.

Toxic autonomous thyroid nodule


1. Hot nodule in the mid right of the thyroid
with increasing suppression of the remaining
gland at each successive year
2. Toxic autonomous thyroid nodule
3. Surgery and RAI I-131
4. The iodine uptake may be moderately
elevated, but it often is in the normal range.
Normal 24 hour uptake is 10 to 30%

Lingual thyroid
1. Tc-99m pertechnetate is taken up by thyroid
follicular cells like iodine but not organified. I123 taken up and organified.
2. Lower radiation exposure to the pediatric
patient.
3. Focal uptake at the base of the tongue.
Normal is submandibular glans and mouth.
No thyroid in the neck.
4. Lingual thyroid.

Left thyroid agenesis


1. Uniform uptake in the right lobe. No activity
seen at the left lobe or elsewhere in the neck or
upper chest.
2. The uniform activity in the right lobe is more
intense than the salivary glands indirect
evidence of an elevated uptake in the absence
of intrinsic salivary gland disease.
3. Surgical
excision,
replacement
by
hypofunctioning adenoma or carcinoma or
agenesis of the left lobe with Graves disease of
the solitary right lobe.
4. Graves disease with agenesis of the left lobe.

Gastrointestinal & Hepatobiliary

Pearls & Pitfalls

Gastric emptying
studies

The study usually are performed


the rate of emptying of solids
from stomach, they are most
commonly performed by Tc-99m
colloid mixed with scramble eggs
before they are cooked.
Half of the activity of a solid meal
should be out of the stomach in
about 90 minutes.
Solids leave the stomach in a
linear
fashion,
liquids
exponentially.

Pearls & Pitfalls

Gastrointestinal
bleeding studies

Usually these are performed by


Tc-99m labeled RBC, but Tc-99m
sulfur colloid can also be used.
A focus of bleeding should
change shape and location on
sequential images. If the activity
does not move, it may represent
a vascular abnormality such as
aneurysm or intussusception.

Pearls & Pitfalls

Gastrointestinal
bleeding studies

The best way to pinpoint the


bleeding site is to find an
image in which there is a
definite abnormality and then
to look at the earlier images
and find the first image in
which the activity can be seen.
This is necessary because
activity seen as a result of
bleeding can go anterograde or
retrograde in the bowel.

Pearls & Pitfalls

Gastrointestinal
bleeding studies

Bladder activity from free Tc99m pertechnetate can be


confusing and sometimes it is
necessary for the patients to
void or catheterized.
If bleeding is intermittent, use
Tc-99m RBC. If the bleeding is
active, either Tc-99m RBC or
Tc-99m sulfur colloid can be
used.

Pearls & Pitfalls

Gastrointestinal
bleeding studies

False positives:
Free pertechnetate seen at
stomach
Renal excretion of breakdown
products
Penile
blood
pool,
transplanted
kidney
and
varices.

Pearls & Pitfalls

Gastrointestinal
bleeding studies

False negatives:
Small bowel bleeding
Rectal bleeding obscured by
bladder activity
A stationary abnormal focus
could represent clotted blood

Gastroesophageal reflux : Milk study


Patient preparation

Overnight fasting

Radionuclide

Tc-99m SC (0.2 1 mCi)

Feeding meal

The radionuclide is mixed with half of the


meal and fed the child. The second cold
half of the meal is then fed to the child.
Orange juice for adults and children

Imaging

After blurping infant, place supine with


gamma camera and radioactive marker at
the mouth
Acquire delayed image of the chest

Cimetidine

Histamine H receptor antagonis


Increases uptake of Tc-99m pertechnitate
Inhibit releasing of gastric mucosa
Dose 20 mg/kg orally 2 days prior of the study

Pentagastrin
Increases rapidity, duration and intensity of Tc99m pertechnitate uptake
Increased mucin producing cells
Increases intestinal motility

Glucagon
Antiperistalsis
Decreased bowel peristalsis
Prevent tracer washout from diverticulum

Tc-99m RBC

Focal activity appears

Criteria for
diagnosis of
bleeding site with
Tc-99m RBC

Activity increases over time


activity movement confirm
to intestinal anatomy
Movement may be
antegrade or retrograde

Tc-99m RBC in vitro

Tc-99m RBC

Tc-99m SC

Tc-99m SC

Meckels Diverticulum

Pearls & Pitfalls


These scan are performed with
Tc-99m pertechnetate which
concentrates in normal and
ectopic gastric mucosa
Meckels diverticulum
imaging
Most Meckels diverticula do
not contain ectopic gastric
mucosa but the ones that
bleed almost always do.
A Meckels diverticula should
be seen anteriorly, lateral or
oblique.

Pearls & Pitfalls


Cimetidine can be used to fix
the
radiotracer
and
pentagastrin can increase
uptake in the gastric mucosa,
Meckels diverticulum
thus increasing the sensitivity
imaging
of the study.
Bladder activity from the Tc99m pertechnetate is normal.
The patient may need to void
or be catheterized if there is a
suspicious lesion nearby.

Pearls & Pitfalls


False positives:
Meckels diverticulum Confounding urinary system
imaging
activity, GI obstruction, tumor
and inflammation.

Pearls & Pitfalls

False negatives:
Meckels diverticulum
Lack of sufficient gastric
imaging
mucosa.
Recent barium study or
bladder.

Meckels

Tc-99m pertechnitate (Meckels)

Hepatobiliary

Pearls & Pitfalls

Hepatobiliary

The most common indications for


hepatobiliary study are to
differentiate between acute or
chronic cholecystitis, to look for
suspected bile leaks or biliary
obstruction and in the setting of
neonatal
jaundice,
to
differentiate neonatal jaundice or
biliary atresia.
Tc-99m (IDA) agents are cleared
and excreted by hepatocytes but
not conjugated.

Pearls & Pitfalls

Hepatobiliary

Cardiac blood pool activity


should clear by 5 to 10 minutes.
Lack of clearance indicates poorly
functioning hepatocytes.
If there is persistent cardiac
blood pool activity and no biliary
excretion, the DD includes
hepatitis and biliary obstruction.
Renal and bladder activity may
be seen if the liver cannot
efficiently
excrete
the
radiotracer.

Pearls & Pitfalls

Hepatobiliary

Bowel activity should be seen by


1 hour. Delayed biliary to bowel
transit can be result of a number
of entities, including common
duct calculous, tumor, stricture,
morphine, sphincter dyskinesia
and chronic cholecystitis.
For
hepatobiliary
scans
performed to differentiate biliary
atresia from neonatal hepatitis,
delayed 24 hours images to look
for evidence of excretion into
bowel are often necessary.

Pearls & Pitfalls

Hepatobiliary

The normal GB with patent


cystic duct is usually seen by
30 minutes and should always
be seen by 1 hour.
Nonvisualization of the GB is
most likely the result of acute
or chronic cholecystitis.
Nonvisualization of the GB at 4
hours or after given morphine
at 1 hour is most likely due to
acute cholecystitis.

Pearls & Pitfalls

Hepatobiliary

Nonvisualization of the GB at 1
hour but visualization at 4
hours after morphine is most
likely
due
to
chronic
cholecystitis.
Look for either RIM sign or
cystic duct sign of acute
cholecystitis if the GB is not
seen by 1 hour. A RIM sign
increases the likelihood of
complicated
cholecystitis
(gangrene, abscess or rupture)

Pearls & Pitfalls

Hepatobiliary

Bile leaks often pool in the region


of the porta hepatis, along right
lateral aspect of the liver. If the
GB was recently removed, the
bile may pool in the GB fossa and
mimic a GB.
An unequivocally GBEF after CCK
is more than 50%. Borderline 35
50% and abnormal < 35%.
A recent meal may cause GB
contraction and result in
nonvisualisation
of
GB,
simulating acute cholecystitis.

Pearls & Pitfalls

Hepatobiliary

False positive
Fasting longer than 24 hours
may cause GB distend with
viscous bile. CCK can contract
GB,
Severe illness, pancreatitis,
chronic cholecystitis and rapid
biliary to bowel transit.

Pearls & Pitfalls

Hepatobiliary

False negatives:
Mistake with duodenum, renal
pelvis and cystic or common
bile duct

Right hepatic duct

Left hepatic duct

Common hepatic duct

Cystic duct

Common bile duct

Gallbladder

Spincter of oddi

Patient preparation

NBM for 4 hours before study


If fasting longer than 24 hours, infuse sincalide

Radiopharmaceutical

Tc-99m HIDA/ mebrofenin/ disofenin


IV
Adults bilirubin < 2mg/dl : 5 mCi
2-10 mg/dl : 7.5 mCi
>10 mg/dl : 10 mCi
Children 200 Ci/kg (no less than 1 mCi)

Patient positioning

Supine

Instrumentation

Collimator: LEGP
Window : 15% 140 keV

Imaging protocol

Inject Tc-99m IDA IV


At 60 min, acquire right lateral and left anterior oblique
If GB not filled and acute cholecystitis suspected, inject IV
morphine sulfate (0.04 mg/kg over 1 min)
Perform delayed imaging 2 to 4 hour if:
1. Morphine sulfate is not administered and GB has not
filled
2. Hepatic insufficient/ suspected biliary leak/ partial
common duct obstruction

Radiopharmaceutical

Mechanism of uptake

Indication

Tc-99m sulfur colloid

Kupffer cell uptake

Focal nodular hyperplasia

Tc-99m HIDA

Hepatocyte uptake

Cholescintigraphy

Tc-99m RBC

Blood pool distribution

Cavernous hemangioma

Tc-99m MAA

Blood flow & capillary


blockage

Hepatic arterial perfusion

Xe-133

Lipid soluble

Focal fatty tumor uptake

Gallium-67 citrate

Lactoferrin transport and


iron binding

Tumor/abscess

F-18 FDG

Glucose metabolism

tumor

CCK

33- amino acid polypeptide


Gallbladder contraction
Relaxation of sphincter of oddi
Sincalide = synthetic form of CCK

Contracts gallbladder
Relaxes sphincter of Oddi

Gastrointestinal
actions of CCK

Stimulates intestinal
motility
Inhibits gastric emptying
Reduces gastrointestinal
sphincter tone
Stimulates hepatic bile
secretion
Stimulates pancreatic
enzyme secretion

Morphine sulphate
Constricting sphincter of Oddi
Increases intraluminal biliary pressure
IV 0.04 mg/kg when GB not filled by 60
minutes
Should not be given to :
1. Poor clearance to the bowel
2. Significant retention of radiotracer in biliary
duct

Fasting < 4 hour


Fasting > 4 hour

False positive of
Acute
Cholecystitis

concurrent severe illness


Chronic cholescystitis
hepatic insufficient
hyperalimentation
Alcoholism/pancreatitis (?)

Drugs a/w poor GB


contraction

Diseases a/w poor GB


contraction

Morphine

DM

Atropine

Sickle cells disease

Nifedipine (CCB)

IBS

Indomethacin

Pancreatic insufficiency

Progesterone

Crohns disease

Oral contraception

Celiac disease

Octreotide

Achalasia

Theophylline

Obesity

Benzodiazepine

Cirrhosis

Phentolamine

Pregnancy
Dyspeptic syndrome
Truncal vagotomy

Differential diagnosis of Primary


Hepatic tumor with Tc-99m HIDA
Lesion

Flow

Uptake

Clearance

Focal
nodular
hyperplasia
Hepatic
adenoma
Hepatocellul
ar carcinoma

Increased

Immediate

Delayed

Normal

none

Increased

Delayed

Delayed

Causes of Focal
Liver Defects

Cyst
Benign and malignant
tumor
Abscess
Hematoma
Laceration
Radiation therapy
Infarction
Cirrhosis
Fatty infiltration
Dilated bile duct

Arm injection

Increased focal
uptake in Tc-99m SC

Leg injection
focal nodular hyperplasia
Budd-Chiari syndrome
Cirrhosis

Metastases
Lymphoma/leukemia

Liver heterogeneity
of Tc-99m SC

Hepatitis
Chronic passive congestion
Cirrhosis
Parenchymal liver disease
Fatty metamorphosis

Normal HIDA

Normal HIDA

HIDA scan, GB visualized after


morphine

Acute cholecystitis

What are two FDA approved IDA in use


and how are they different?

What are two FDA approved IDA in use


and how are they different?
Tc-99m DISIDA and Tc-99m mebrofenin.
Mebrofenin has better hepatic extraction 98%
and renal excretion is 1 %. The higher
extraction of mebrofenin is preferable in
patients with hepatic insufficiency.

What is the most important Q to ask


the patient before starting
cholescintigraphy for suspected acute
cholecystitis?

What is the most important Q to ask


the patient before starting
cholescintigraphy for suspected acute
cholecystitis?
When did you last eat. If the patient has eaten
in the last 4 hours, the GB may be contracted
secondary to endogenous stimulation of CCK
and therefore tracer cannot enter into GB.
If the patient has not eaten > 24 hours, the GB
may not have had the stimulus to contract and
will be full of thick, concentrated bile which
may prevent tracer entry.

What are 5 indication for CCK


infusion?

What are 5 indication for CCK


infusion?
1. Empty GB in patient fasting longer than 24
hours
2. Differentiate common duct obstruction from
normal hypertonic sphincter of Oddi
3. Exclude GB acalculous cholecystitis if GB fills.
4. Diagnose chronic acalculous cholecystitis.
Confirm or exclude chronic calculous
cholecystitis.
5. Assist in the diagnosis of sphincter of Oddi
dysfunction.

In what clinical settings are false


positive HIDA studies likely to occur
when performed to rule out acute
cholecystitis.

In what clinical settings are false


positive HIDA studies likely to occur
when performed to rule out acute
cholecystitis.
In patients who have fasted less than 4 hours
or more than 24 hours, patient receiving
hyperalimentation and those who have
chronic cholecystitis, hepatic insufficiency or
severe illness.

What is the rim sign sometimes seen


with cholescintigraphy and what is its
significance?

In what clinical settings are false


positive HIDA studies likely to occur
when performed to rule out acute
cholecystitis.
The rim sign is increased uptake and delayed
clearance of activity in the hepatic
parenchyma adjacent to the GB fossa. It has
been associated with an increased incidence
of complications e.g.. perforation and
gangrene.

At what time after HIDA injection is


nonfilling of the GB diagnostic of acute
cholecystitis

At what time after HIDA injection is


nonfilling of the GB diagnostic of acute
cholecystitis
One hour is defined as abnormal. However,
nonfilling of the GB is diagnostic of acute
cholecystitis if delayed images show no filling
by 2 to 4 hours or 30 minutes after morphine
administration.

What is the mechanism of morphine


augmented cholescintigraphy?

What is the mechanism of morphine


augmented cholescintigraphy?
Morphine increases tone at the sphincter of
Oddi, resulting in increased intraductal
pressure. This result in bile flow through the
cystic duct, if it is patent.

What is the most common scan finding


in chronic cholecystitis?

What is the most common scan finding


in chronic cholecystitis?
A normal study. < 5% of patients with chronic
cholecystitis have delayed filling. Other
associated findings include delayed biliary to
bowel transit time and rarely nonvisualization
of GB or intraluminal filling defects.

What is acute acalculous cholecystitis?

What is acute acalculous cholecystitis?


Cholecystitis without a stone occluding the
cystic duct. The obstruction may caused by
debris or inflammatory changes or the
cholecystitis may be limited to the GB wall
because of infection, ischemia or toxins. It
occurs in hospitalized patients who have
sustained trauma, burns, sepsis or other
serious illness.

What are the cholescintigraphic


findings of high grade common duct
obstruction?

What are the cholescintigraphic


findings of high grade common duct
obstruction?
Persistent hepatogram with no clearance into
biliary ducts.

What are the cholescintigraphic


findings of partial common duct
obstruction?

What are the cholescintigraphic


findings of partial common duct
obstruction?
Prominent retention of activity in the common
duct, delayed biliary to bowel clearance and
most important, poor ductal clearance on
delayed imaging or with CCK.

What is the difference in clinical


presentation and clinical course of
patients with FNH and hepatic
adenoma

What is the difference in clinical


presentation and clinical course of
patients with FNH and hepatic
adenoma
FNH is asymptomatic and found incidentally,
whereas hepatic adenomas often present with
hemorrhage and can be life threatening.
Adenomas are closely associated with the use
of OCP which must be discontinued.

What are the Tc-99m SC scan findings


in FNH and hepatic adenoma?

What are the Tc-99m SC scan findings


in FNH and hepatic adenoma?
Hepatic adenomas do not show Tc-99m SC
uptake because they do not usually have
Kupffer cells. FNH is associated with increased
blood flow. Uptake may be increased, normal
or nonexistent. Two thirds of cases of FNH
show some Tc-99m SC uptake.

What are the characteristic


scintigraphy findings in liver
hemangioma?

What are the characteristic


scintigraphy findings in liver
hemangioma?
Blood flow is normal. Immediate images show
a cold defect, whereas delayed images
acquired 1 to 2 hours after tracer
administration show increased uptake within
the lesion compared with the normal liver,
often equal to uptake in the spleen and heart.

Beside FNH, what are other causes of


increased focal uptake on Tc-99m SC
imaging?

Beside FNH, what are other causes of


increased focal uptake on Tc-99m SC
imaging?
Superior vena cava syndrome (with arm
injection), inferior vena cava syndrome (with
leg injection), Budd-Chiari syndrome and
cirrhosis with a regenerating nodule.

What is functional asplenia?

What is functional asplenia?


Nonvisualization of the spleen on a Tc-99m SC
when the spleen is anatomically present and
when functions other than reticuloendothelial
extraction are intact. Functional asplenia is
caused by an acquired dysfunction of the
reticuloendothelial system e.g.. sickle cell
anemia or by a disruption of the blood supply
(splenic artery occlusion).

What is achalasia and how can


radionuclide studies help in making
the diagnosis and following the
patients course?

What is achalasia and how can


radionuclide studies help in making
the diagnosis and following the
patients course?
Achalasia is characterized by absence of
peristalsis in the distal two thirds of the
esophagus, increased lower esophageal sphincter
pressure and incomplete sphincter relaxation
after swallowing. The diagnosis can be confirmed
by esophageal manometry. Radionuclide
esophageal transit studies have a high sensitivity
for making the diagnosis and evaluate the
effectiveness of esophageal dilatation.

In regard to reflux and aspiration


studies?
A. The milk study is a sensitive method
diagnosing gastroesophageal reflux.
B. The milk study is a sensitive method
diagnosing aspiration.
C. Frequent image acquisition improves
sensitivity of milk study.
D. The salivagram is a sensitive method
diagnosing aspiration.

for
for

the
for

In regard to reflux and aspiration


studies?
TFTT

Which anatomical portion of the


stomach are responsible for solid
emptying and which for liquid
emptying

Which anatomical portion of the


stomach are responsible for solid
emptying and which for liquid
emptying
Liquid emptying is largely caused by the slow
contractions of the proximal fundus whereas
the distal stomach or antrum is responsible for
the grinding and sieving solid food.

Which of these factors will affect the


rate of gastric emptying?
A.
B.
C.
D.
E.
F.

Meal content
Time of day
Gender
Position
Stress
Exercise

Which of these factors will affect the


rate of gastric emptying?
TTTTTT

Regarding gastric emptying studies?


A. Attenuation results in an underestimation of
gastric emptying when performed in the
anterior view
B. A solid gastric emptying time activity curve
shows a rise in activity after ingestion in the
anterior view
C. The geometric mean method of attenuation
correction is considered the reference standard
D. The left anterior oblique method of attenuation
correction is superior to the geometric mean.

Regarding gastric emptying studies?


TTTF

When might the use of Tc-99m SC


offer advantages over Tc-99m RBCs for
the diagnosis of acute GI bleeding?

When might the use of Tc-99m SC


offer advantages over Tc-99m RBCs for
the diagnosis of acute GI bleeding?
With very rapid GI bleeding and vascular
instability, the radiotracer can be injected and
the study completed in 15 to 20 minutes. It is
likely to be positive with the rapid
hemorrhage when transfusion cannot keep up
with the bleeding rate. The patient can then
go directly for angiography.

List in increasing order the labeling


efficiency for method to label Tc-99m
RBCs; in vivo, in vitro or in vivtro

List in increasing order the labeling


efficiency for method to label Tc-99m
RBCs; in vivo, in vitro or in vivtro
In vivo 75%, in vivtro 85% and in vitro 98%

Why is the Tc-99m RBC for detecting GI


bleeding more sensitive than the Tc99m SC?

Why is the Tc-99m RBC for detecting GI


bleeding more sensitive than the Tc99m SC?
Delayed images can be performed up to 24
hours with RBC labeling

What are the criteria needed to


diagnose confidently the site of
bleeding on a radionuclide study?

What are the criteria needed to


diagnose confidently the site of
bleeding on a radionuclide study?
a. Hot spot appears where there was none and
conforms to bowel activity
b. Activity increases over time
c. Activity moves antegrade and retrograde

Ectopic mucosa is most often seen


clinically in Meckels diverticulum.
What other gastric abnormalities may
contain gastric mucosa?

Ectopic mucosa is most often seen


clinically in Meckels diverticulum.
What other gastric abnormalities may
contain gastric mucosa?
Duplication of GI tract
Barretts esophagus

What is the origin of Meckels


diverticulum?

What is the origin of Meckels


diverticulum?
Failure of closure of the omphalomesentric
duct of the embryo, which connects the yolk
sac to the primitive foregut via the umbilical
cord

Concerning Meckels diverticulum


A. Ectopic gastric mucosa in a Meckels
diverticulum can be reliably distinguished from
ectopic mucosa elsewhere in the small intestine
B. Cimetidine given before the scan to blocks the
secretion of pertechnetate from gastric mucosa
C. Children are more likely to have a positive test
than adults
D. Diverticulitis is a common cause of false positive
result
E. A false negative result may be obtained if there
has been a recent barium study

Concerning Meckels diverticulum


FTTFT

Concerning GI bleeding studies


A. Hepatic hemangioma is a recognised cause of
false positive Tc-99m RBS study
B. A rejecting transplanted kidney is a recognised
cause of a false positive Tc-99m tin colloid study
C. Anemia is a cause of visualization of the GB in
labeled RBC study
D. Diffuse colonic activity in a labeled RBC study
indicates colonic bleeding
E. When reporting a study. It is not necessary to
view the dynamic images that provide all
information

Concerning GI bleeding studies


TTTFF

GI bleeding studies
A. Tin colloid studies are better at detecting bleeding
sites proximal to the ligament of Treitz than distal to it
B. Delayed images on a tin colloid scan may be of value
in separating liver and spleen activity from bowel
activity
C. In vitro labeling of RBC gives fewer false positive tests
than in vivo
D. Bleeding must be continuous to be detected on a
labeled RBC study
E. 51 Cr-chromate labeled RBC can detect bleeding
down to 1-2 mls per day

GI bleeding studies
FFTFT

Concerning cholescintigraphy
A. Failure to visualize the GB by 4 hr with normal activity
appearing in the bowel is highly suggestive of GB
disease
B. Chronic Cholecystitis can be distinguished from
acalculous cholecystitis
C. Giving CCK before isotope is more likely to
demonstrate the GB than giving morphine
D. Common duct obstruction should be excluded if
visualization of bowel activity is delayed beyond 1
hour
E. GB emphysema has a typical rim sign appearance

Concerning cholescintigraphy
TFTTT

Concerning cholescintigraphy
A. Cholescintigraphy has greater specificity than
ultrasound at detecting bile duct obstruction
B. Following cholecystectomy, an abnormal
accumulation of HIDA outside biliary tree likely
due to bile leak
C. US is more sensitive than cholescintigraphy at
detecting bile leak
D. Cholescintigraphy can diagnose afferent loop
obstruction after gastroenterostomy
E. Ascites can be detected

Concerning cholescintigraphy
TTFTT

In pediatric cholescintigraphy
A. Phenobarbitone is used to maximise hepatic of
tracer
B. Neonate hepatitis can be distinguished from
Rotor syndrome
C. Choledocal cyst can be distinguished from
pancreatic cyst
D. Cholescintigraphy is more accurate in diagnosing
biliary atresia in children > 3 months than in
neonates
E. Dilated intrahepatic ducts are often seen in
patients with biliary atresia

In pediatric cholescintigraphy
TTTFF

Concerning GI transit studies


A. It is easier to detect abnormalities of gastric
emptying for solids than for liquids
B. DM is a common cause of delayed gastric
emptying
C. Hyperthyroidism is the commonest of rapid
gastric emptying
D. Scleroderma affects esophageal transit through
the whole esophagus
E. Achalasia has a specific appearance on a
radionuclide esophageal transit time

Concerning GI transit studies


TTFFF

Concerning Milk Studies


A. Reflux is more likely to be detected if the infant
is placed prone
B. The longer the test the more sensitive it
becomes
C. Labeled milk studies are better at detecting
post-prandial reflux than 24 hour pH monitoring
D. A normal study excludes significant lung
aspiration
E. Reflux detected by scintigraphy is more likely to
be significant

Concerning Milk Studies


FTTFF

3. False positive for Tc-99m RBC


TTTFF

6. Tc-99m HIDA
TTTTF

8. Sulphur colloid scan


FFFFT

9. Colloid shift is seen in


TTTTF

12. Focal defect in sulphur colloid scan


is seen in
TTTTT

What physiologic molecules does Tc99m IDA most closely mimic?

What physiologic molecules does Tc99m IDA most closely mimic?


Bilirubin

What is the main difference between


Tc-99m IDA and bilirubin?

What is the main difference between


Tc-99m IDA and bilirubin?
Tc-99m IDA is not conjugated

What level of bilirubin is required to


interfere with the quality of HIDA
scan?

What level of bilirubin is required to


interfere with the quality of HIDA
scan?
> than 20 to 30 mg/dl

How does prior CCK administration


affect the biliary to bowel transit time?

How does prior CCK administration


affect the biliary to bowel transit time?
CCK delays biliary to bowel transit time

How can one distinguish acute from


chronic cholecystitis as a cause of non
visualization of the GB at 1 hour ?

How can one distinguish acute from


chronic cholecystitis as a cause of non
visualization of the GB at 1 hour ?
Obtain delayed image and perform morphine
augmentation

What 2 findings on scan increased


likelihood that nonvisualisation of GB
is caused by acute cholecystitis ?

What 2 findings on scan increased


likelihood that nonvisualisation of GB
is caused by acute cholecystitis ?
1. Hyperemia in GB fossa on early images
2. Rim sign on later images

What is the prognostic implication of


rim sign in scan?

What is the prognostic implication of


rim sign in scan?
It is associated with GB gangrene and
perforation

How does morphine augmented


cholescintigraphy work?

How does morphine augmented


cholescintigraphy work?
Morphine increases the tone of sphincter of
oddi
Raising common bile duct pressure
Facilitating GB filling if cystic duct is patent

How does one perform morphine


augmented cholescintigraphy?

How does one perform morphine


augmented cholescintigraphy?
If GB does not fill by 60 minutes
Give 0.04 mg/kg IV morphine
If GB does not fill by 30 minutes means the
patient has cystic duct obstruction

What is the major cause of a false


negative scan for acute cholecystitis in
scan ?

What is the major cause of a false


negative scan for acute cholecystitis in
scan ?
Acute acalculous cholecystitis

How does one perform GB ejection


fraction?

How does one perform GB ejection


fraction?
Administer IV sincilide 0.02 g/kg
Normal GB ejection fraction = 35%

What patient preparation is requires


before scan for biliary atresia?

What patient preparation is requires


before scan for biliary atresia?
Phenobarbital 5 mg/kg for 5 days

Tc-99m colonic bleeding


1. Tc-99m labeled RBCs.
2. A. left colon, rectosigmoid region
3. B. Right colon, hepatic flexure. It moves
rapidly to the left colon.
4. RBC scintigraphy, 0.1 mil/min; contrast
angiography 1 ml/min.

Meckels Diverticulum
1. Meckels can. The radiopharmaceutical is taken up and
secreted by gastric mucosa.
2. Pentagastrin increases rapidity, intensity and duration of
uptake. It is used with glucagon, which is antiparistaltic
that inhibits rapid dispersion effect of pentagastrin.
Cimetidine, a histamine antagonist, increases and
prolonged uptake because of inhibition of Tc-99m
pertechnetate secretion from gastric mucosal cells.
3. Increasing focal uptake in the mid abdomen suspicious for
Meckels diverticulum, however, atypical timing of uptake
lessens the certainty. The uptake should be coincident
with gastric uptake in cases of Meckels diverticulum. This
may be false positive scan.
4. Acid and pepsin secretion by the gastric mucosa produces
inflammation and ulceration of adjacent bowel mucosa.

Diabetic gastroparesis
1. Consistent with severe diabetic gastroparesis. Obstruction
cannot be ruled out.
2. Normal values are meal specific and depend on its
volume/composition, the method of acquisition,
attenuation correction, processing and quantification.
Normal values must be determines in each clinic or results
of a published method should be followed closely.
3. Solid or semisolid gastric emptying meals are more
sensitive for detection of mild to moderate delay in
emptying than studies conducted after a liquid meal.
4. Activity is detected with greatest efficiency close to the
camera. The anterior view alone underestimates emptying
and posterior view overestimates it because of variable
attenuation as the meal moves through the stomach from
the posterior gastric fundus to the more anterior gastric
antrum.

Gastroesophageal reflux
1. Vomiting, pulmonary symptoms, asthma,
pneumonia, sudden death, failure to thrive
and anemia.
2. 24 hours pH monitoring.
3. Tc-99m sulfur colloid 1 mCi in the childs
usual feeding, formula or milk
4. 5 to 10 second/frame.

Gastrointestinal bleeding
1. Tc-99m labeled RBC study. Abnormal focal
uptake appearing simultaneously at two sites
in the right abdomen, increasing in intensity
and changing in pattern with time.
2. Caecum and ascending colon.
3. Review images an a computer in cinematic
mode.
4. Acute bleeding due to angiodisplasia,
diverticula, neoplasm, IBD and ischemia.

Axillobifemoral bypass graft


1.
2.
3.
4.

Tc-99m labeled RBC or sulfur colloid


Evidence of current or recent bleeding
Tc-99m RBCs
The vascular flow and delayed static images
show no active bleeding seen. Labeled RBCs
are shown in a tubular shape in the right
abdomen that connects after a Y bifurcation
to the two iliac vessels as a result of an
axillobifemoral vascular bypass graft. The
patient has prominent splenomegaly.

Free pertechnetate vs gastric bleeding


1. Tc-99m RBC
2. In both studies the stomach visualize
promptly. The chest and neck image shows
thyroid and salivary gland uptake in study A
but not in study B
3. A. Free pertechnetate. Negative for
gastrointestinal bleeding. B. Active bleeding
originating from the stomach
4. In vitro

Rectal bleeding
1. Abnormal activity accumulate early in the lower
midline pelvis. The appearance is changing over
time and seems to decrease and then increase
again.
2. To differentiate activity in the rectum from
bladder and penis, in this patient the activity is
seen in the rectum.
3. Positive for gastrointestinal bleeding is not the
answer. Localization is critical. The 90 minute
lateral view confirms that this is rectal bleeding.
4. New activity, increases in amount over time and
moves intraluminally.

Small bowel bleeding


1. Tc-99m RBCs. The in vitro method of labeling
was used.
2. Evidence of active bleeding, first seen at 5
minutes, starting in the left upper abdomen
and moving across the mid to lower
abdomen to the right lower and then right
mid abdomen.
3. Small bowel bleeding originating from the
region of the jejunum.
4. Arteriovenous malformation or tumors

Pulmonary aspiration
1. Recurrent pneumonia, cough, asthma, apnea or
sudden infant death.
2. Tc-99m sulfur colloid in a small volume of fluid
placed on the tongue and allowed to mix with
oral secretion and swallowed.
3. Poor bolus progression noted in the dynamic
esophageal swallow study with entrance into
the main bronchi bilaterally and then into the
right lower lobe.
4. The milk study is very sensitive for reflux,
however it is insensitive for aspiration. The
salivagram is sensitive for small amounts of
pulmonary aspiration.

Choledocal cyst
1. A. Good hepatic uptake and clearance into the
gallbladder, common hepatic and proximal common
bile duct at 60 minutes. No biliary to bowel transit. B.
the gallbladder contracts with sincalide infusion,
however focal increasing accumulation of radiotracer
occurs just medial to the proximal portion of the
common bile. The proximal common duct activity
empties into the duodenum.
2. Likely choledocal cyst. Partial biliary obstruction also
would cause radiotracer retention within the more
proximal biliary duct.
3. Cholangitis, sepsis, pancreatitis or obstruction.
4. Congenital anomaly. Localize dilatation of the biliary
tract, either fusiform or diverticular outpouching.
Surgery is the appropriate therapy.

Chronic acalculous cholecystitis


1. CAC is clinically and pathologically identical to chronic
calculous cholecystitis, except for absence of
gallstone.
2. Anatomical imaging diagnosis depends heavily on
visualization of gallstones. CCK cholescintigraphy
allows quantification of gallbladder contraction.
Diseased gallbladders do not contract.
3. Findings consistent with chronic acalculous
cholecystitis in patient B but not A.
4. A low GBEF has a positive predictive value of more
than 90% that cholecystectomy will cure the patients
symptoms and that diagnosis will be confirmed by
pathological gallbladder examination.

Post cholecystectomy syndrome


1. Prompt hepatic uptake, clearance into biliary ducts by 15
to 30 minutes, retention of activity in ducts, apparently
dilated proximal hepatic and common ducts, biliary to
bowel clearance but prominent retention in common duct
with apparent cutoff distally.
2. 60 minutes: partial common duct obstruction versus post
biliary duct obstruction surgery with nonobstructed
persistent bile duct dilatation. 120 minutes: suspected
partial obstruction.
3. Common duct stone, inflammatory stricture of the
common duct, sphincter of Oddi dysfunction, inflamed
cystic duct remnant.
4. Sphincter of Oddi dysfunction is a partial biliary
obstruction at the level of the sphincter of Oddi not
caused by atones or stricture.

Focal nodular hyperplasia


1. Increased uptake that corresponds to the lesion
seen on CT. the uptake is retained after liver
washout.
2. Benign and malignant tumors that have
hepatocytes e.g.., hepatic adenoma, hepatoma
and FNH. The letter is the correct diagnosis.
3. FNH usually requires no specific therapy. Hepatic
adenoma requires discontinuation of oral
contraceptives and surgical removal and
hepatoma requires resection.
4. Tc-99m sulfur colloid.

Hot spot Tc-99m sulfur colloid imaging


1. Uptake by reticuloendothelial system.
2. 85% by Kupffer cells, 10% by the spleen and
5% by the bone marrow.
3. A. the central large cold region may be
caused by an intrahepatic mass but the
pattern has been stable and CT shows
cirrhosis. B. increased uptake in the region of
the quadrate lobe, consistent with FNH.
4. Superior vena cava syndrome, inferior vena
cava obstruction, Budd-Chiari syndrome.

Cholescintigraphy normal study


1. No oral intake for 3 to 4 hours before
injection.
2. Tc-99m disofenin or Tc-99m brida.
3. Both are iminodiacetic acid analogues,
extracted and excreted by hepatocytes into
biliary system. Mebrofenin has higher
hepatocyte extraction (98%)
4. Yes. Acute cholecystitis is ruled out with a
high degree of certainty.

Acute cholecystitis
1. A. Abnormal. B. No. C, yes
2. A. Sensitivity 98%, specificity 95%, B greater
than 90%.
3. The few direct comparisons published have
shown cholescintigraphy superior to US.
4. Prolonged fasting, hyperalimentation, chronic
cholecystitis and hepatic insufficiency.

Morphine augmented
cholescintigraphy
1. Morphine produces contraction of the sphincter
of Oddi, which increases intraluminal common
bile duct pressure. Bile and excreted radiotracer
then preferentially flow through the cystic duct
into the GB if the cystic duct is pattern.
2. Exclude drug allergy. Do not give if evidence of
common duct obstruction.
3. The accuracy is at least as good, if not better
than the delayed imaging method.
4. IV 0.04 mg/kg morphine. Images are acquired
for an additional 30 minutes.

Biliary leak
1. Rapid bile leakage probably originating from
the region of the ligated cystic duct and
extending toward the right colonic gutter
with time, over the dome of the liver.
2. Rapid biliary leak.
3. Cystic duct ligature after cholecystectomy,
surgical
anastomosis,
trauma
and
inflammatory processes.
4. Confirm that fluid collection seen by
anatomical imaging modalities are biliary in
nature.

Cavernous hemangioma of the liver


1. Tc-99m RBCs
2. Immediate images show no definite
abnormality. Delayed images show increased
focal uptake in the left lobe consistent with
cavernous hemangioma.
3. At least 2 cm
4. Very specific (>99%) for hemangioma. Poor
sensitivity for small lesions.

Biliary atresia
1. Phenobarbital, 5mg/kg/day for 3 to 5 days
before study.
2. Inflammatory, infections and metabolic causes
for neonatal hepatitis and biliary atresia.
3. A. delayed blood pool clearance as the result of
hepatic insufficiency. Biliary clearance at 50
minutes and increasing through 120 minutes. B.
good liver function. No secretion into biliary
duct during the initial 120 minutes or at 5 and
24 hours. Case B consistent with biliary atresia.
Case A with neonatal hepatitis.
4. Sensitivity 97% and specificity 82%.

RIM sign
1. Increased incidence of false positive in patients who
have been fasting more than 24 hours
2. Nonvisualization of GB after 60 minutes. After
morphine, no filling of the GB occurs. Increased
uptake is seen in the region of the GB fossa, which
persists after most of the liver has washout (RIM sign)
3. Nonvisualization of the GB after morphine is
consistent with acute cholecystitis. The RIM sigh is
very specific for acute cholecystitis and confirms
diagnosis.
4. The RIM sign indicates severe acute cholecystitis
which is associated with an increased incidence of GB
gangrene and perforation.

Genitourinary scan

Pearls & Pitfalls

General

The common indications for


radionuclide scan for kidneys
include allergy to iodinated
contrast material, assessment
of possible renal artery
stenosis and differentiation of
obstruction from collecting
system. In children cortical
scanning agents are used for
evaluation of pyelonephritis.

Pearls & Pitfalls

General

Tc-99m DTPA reflects glomerular


filtration, Tc-99m MAG3 is
essential a tubular agent.
Tc-99m DMSA is renal cortical
agent.
Tc-99m glucoheptonate is used
to evaluate both renal cortical
and collecting system.
Maximum parenchymal activity is
seen at about 3 to 5 minutes and
bladder activity is seen by 4 to 8
minutes.

Pearls & Pitfalls

General

Half of the renal activity should


be cleared in about 8 to 12
minutes.
On a postcaptopril renogram
using Tc-99m MAG3 , the kidney
that has delayed clearance with
significant renal cortical retention
should be the one with the renal
artery stenosis.
Bilateral delayed clearance can
be caused by bilateral stenosis,
obstruction,
renal
disease,
dehydration or hypotension.

Pearls & Pitfalls

General

ATN perfusion is usually


normal
with
bilaterally
increased renal cortical activity
and rising renogram curves
when Tc-99m MAG3 is used.
When faced with abnormal
time activity curves, look at the
images to see if the problem is
parenchymal or related to the
collecting system.

Pearls & Pitfalls


Common indication are to
differentiate rejection from
ATN.
Renal transplant ATN is usually seen within the
first
week
after
transplantation. It usually has
preserved renal perfusion with
progressive accumulation of
tubular agents (Tc-99m MAG3)
in the renal parenchyma. It
should improved with time.

Pearls & Pitfalls


Severe ATN can present with
enough edema immediately
after surgery to have reduced
perfusion and therefore look
Renal transplant
similar to acute rejection.
Cyclosporin toxicity can look
like ATN but is usually not seen
in
the
immediate
postoperative period.
Rejection usually has poor
perfusion and poor tubular
excretion.

Pearls & Pitfalls


Look for photopenic defects
around the transplant on the
blood pool image that may be
caused
by
urinomas,
Renal transplant
hematomas and lymphoceles.

Pearls & Pitfalls

Testicular
imaging

Common indications are to


differentiate
between
epididymitis and acute or
delayed torsion.
The initial problem is to
determine whether blood flow
is increased or decreased.
The best way to determine
which side is abnormal is to
obtain a patient history.

Pearls & Pitfalls

Testicular
imaging

Epididymitis
should
be
diffusely hot on all images and
frequently focal hot in the
region of the epididymitis.
Increased
flow
to
one
hemiscrotum and a rim of
testicular activity with a cold
center (halo sign) can be a
delayed torsion but also can be
testicular abscess, hematoma
or tumor

Glomerular filtration 100 %

DTPA

Tubular secretion 100%

MAG 3

Tubular secretion 80%


Glomerular filtration 20%

HIPPURAN

Cortical binding 40%

DMSA

Glomerular filtration 80%


Cortical binding 15%

GLUCOHEPTONATE

Tc-99m DTPA

Tc-99m MAG 3

Glomerular filtration

100%

Tubular secretion

100%

Extraction fraction

20%

40 50%

Target to back ground


ratio

Poor

Good

Protein bound

No

Yes

Time for imaging

Immediate

Immediate

Heating for labeling

No

Yes

30 60 SECONDS
1-3 MINUTES

COLLECTING
SYSTEM

CORTICAL

LIVER
KIDNEY

lungs
ADRENAL

RVH with DTPA

RVH with MAG 3

Renogram pattern with ACE

Renogram pattern with ACE


A = normal TAC curve (grade 0)
B = peak mildly delayed but greater than 5 min
and with delayed excretion (grade 1)
C = very delayed uptake but some wash out
(grade 2A)
C = extremely delayed uptake with no wash out
(grade 2B)
D = complete renal failure where blood pool
moves through the kidney without an extraction
phase

Pre surgical
insult

Autoimmune
rejection

Complication of
renal transplant

Vasomotor nephropathy
resolution ATN

Minutes to hours

Hyperacute rejection

Minutes to hours

Accelerated rejection

1 5 hours

Acute rejection

After 5 days first 3/12

Chronic rejection

Months to years

Cyclosporine rejection

Months

Urine leak

Days or weeks

Hematoma

1st few days

Infection

1st few days

Lymphocyle

2 4 months

Renal artery stenosis

After 1st months

Surgical

Vascular occlusion

Vascular

Infarct
Renal obstruction
Vesicoureteroreflux

Days/months/years

1. Static radionuclide imaging of


kidney
A. Tc 99m DMSA is the radionuclide of choice
B. Tc 99m DMSA is the best for imaging
pseudotumor of kidney
C. DMSA can measure relative tubular mass
D. DMSA can detect of areas of renal ischemia
E. Optimum time for DMSA scanning is one
week after onset of symptoms in infection

1. Static radionuclide imaging of


kidney
TTFFF

2. MAG 3 scintigraphy
A. Best method for assessment of scarring in
children
B. Prolonged transit time in dilated system
C. Reliable for diagnosis of obstruction in renal
failure
D. Prolonged parenchymal transit time in renal
artery stenosis
E. Patients are dehydrated before study

2. MAG 3 scintigraphy
FTFTF

3. Renal scintigraphy
A. For captopril test, ACE inhibitors should on
the day of the test
B. MAG 3 is used for assessing renal scarring
C. DMSA is superior to MAG 3 for both dynamic
and structural assessment
D. Radiation is lower for MAG 3
E. 100% of MAG 3 is excreted by tubular
secretion

3. Renal scintigraphy
FTFTF

4. Renal scintigraphy
A. DMSA images are acquired within one hour
of injection to avoid artifacts
B. DTPA is isotope of choice for dynamic renal
scans
C. Kidney/background ratio is better for MAG 3
than DTPA
D. I 123 can be produced only by cyclotron
E. Hippuran is completely cleared by glomerular
filtration

4. Renal scintigraphy
FFTTF

5. Renal scintigraphy
A. The glomerular filtration rate of DTPA is 500
ml/min
B. DTPA is completely cleared by tubular
filtration
C. The maximum diuresis occurs within 2
minutes of diuretic administration
D. Images are acquired between 5-10 minutes
for perfusion in transplants
E. Post void film is indicated in cases of stasis

5. Renal scintigraphy
FFFFT

6. Static renal scans are indicated in


the following instances
A.
B.
C.
D.
E.

Horseshoe kidney
Renal agenesis
Column of bertin
Individual renal function
UTIs

6. Static renal scans are indicated in


the following instances
TTTTT

7. Recognised indication of dynamic


nuclear scanning
A.
B.
C.
D.
E.

Hypertension
Renal transplantation
Renal trauma
Reflux
Tumor

7. Recognised indication of dynamic


nuclear scanning
TTTTF

9. Abnormal bone uptake in kidney


A.
B.
C.
D.
E.

Radiation
Chemotherapy
Papillary necrosis
Multiple myeloma
Amyloidosis

9. Abnormal bone uptake in kidney


TTFTF

10. In urinary tract obstruction


A. A DMSA scan always estimates renal function more
accurately than DTPA scan
B. Evan if a kidney contributes less than 10% to the total
GFR, its function will usually improve if the
obstruction is relieved
C. Failure of the collecting systems to empty on standing
confirms the presence of obstruction
D. Lower urinary tract obstruction may be missed on a
DTPA study
E. Giving furosemide during a DTPA study results in a
greater specificity for diagnosing PUJ obstruction than
giving it at the start of the study

10. In urinary tract obstruction


FFFFTF

11. Concerning radionuclide studies


A. DMSA gives an accurate assessment of overall
renal function in renal tubular acidosis
B. DTPA studies can reliably exclude obstruction in
dehydrated patient
C. A tissue injection can give a false positive result
for obstruction
D. An aortic aneurysm can be excluded if a dilated
aorta is not seen on the blood flow renogram
images
E. Visualization of a ureter on a renogram is
abnormal

11. Concerning radionuclide studies


FFTFF

12. Concerning radionuclide studies in


acute renal failure
A. ATN typically can be distinguished from other
causes of acute renal failure
B. On a DTPA study in pre renal failure, there is
impaired renal blood flow, poor uptake, delayed
intrarenal transit and little or no excretion
C. A rising T/A curve on DTPA study confirms
obstruction
D. In ATN , a horizontal third phase to the T/A curve
in a DTPA study indicates that recovery is
occurring
E. MAG 3 is agent of choice

12. Concerning radionuclide studies in


acute renal failure
TFFTT

13. Concerning renogram and renal


transplants
A. On a DTPA scan, rejection can be distinguished from
cyclosporine A toxicity
B. Arterial thrombosis can be distinguished from venous
occlusion
C. A high perfusion index means that there is a good blood
supply to the transplant
D. Arterial occlusion can be detected on a DTPA study even if
an earlier one has shown ATN from which the patient has
not yet recovered
E. Vesicoureteric reflux is the commonest
cause of
pelviureteric dilatation
F. A urinary leak can be excluded if it is not shown in early
images

13. Concerning renogram and renal


transplants
FFFTTF

14. Concerning MAG 3


A. A kit reconstituted first thing in the morning
remains usable until the 99m-Tc decays
B. MAG 3 has a smaller volume of distribution
than DTPA
C. Liver and biliary activity may be seen
D. MAG 3 measures GFR more accurately than
DTPA
E. The radiation dose from a MAG 3 study is
less than from DTPA

14. Concerning MAG 3


FTTFF

15. In renal tract infections


A. In acute pyelonephritis, there may be area of
decreased perfusion on the flow study
B. In a patient with UTI, a small smooth kidney is unlikely
to be due to chronic pyelonephritis alone
C. The overall sensitivity of DMSA is increased by SPECT
D. A recognised feature of established scars is that they
become more prominent with time
E. Renal parenchymal involvement can be predicted
from clinical and lab parameters
F. Most acute parenchymal defects develop into cortical
scars

15. In renal tract infections


TFFTFF

16. Concerning vesicoureteric reflux


A. In patients who reflux, 20% reflux only during micturition
B. Reflux seen on direct scintigraphy cystography correlates
closely with the results of an MCUG
C. Any reflux is a risk factor for pyelonephritis
D. Renal scarring occurring after an episode of acute
pyelonephritis is independent of continued reflux
E. A recognised appearance on a DMSA scan of a kidney
damaged by infection is a normal size kidney with reduced
function
F. Reflux occurring after the age of 3 yrs in patients with
normal DMSA scans is probably irrelevant
G. A 1 yr old with their first proven UTI and hydronephrosis
on US should have a DMSA and an MCUG

16. Concerning vesicoureteric reflux


TFFFTTF

17. In ACE I renography


A. With longstanding total occlusion of the renal
artery, there can be no uptake in the affected
kidney
B. Branch arterial stenosis can be detected
C. In a patient with solitary kidney, ACE I
renography may result in acute renal failure
D. Furosemide is recommended at the start of the
test to improve specificity
E. In a MAG 3 study, a continually rising curve
indicates a haemodynamically significant
stenosis

17. In ACE I renography


FTTTT

18. Renal radionuclide imaging


A. Can distinguish between renal contusion and
infarction
B. Can detect radiation nephritis 3 wks after
treatment
C. Can detect central cysts more easily than
peripheral ones
D. A phantom kidney may be seen after
nephrectomy
E. A phantom kidney, if due to mesenteric vessels
more commonly occurs on the right

18. Renal radionuclide imaging


TTFTF

19. The following are causes of a


flattened DTPA curve
A.
B.
C.
D.
E.

Hypovolaemia
Hypoplastic kidney
Recent contrast angiogram
After extracorporeal shock wave lithotripsy
Severe cyclosporine toxicity

19. The following are causes of a


flattened DTPA curve
TTTTT

20. In DTPA renal scan


A. The examination is contraindicated in renal
failure
B. Fluid restriction is mandatory
C. Vascular phase is acquired immediately after
injection
D. The acquisition is performed with patient
facing the camera
E. Calculation of GFR takes the background into
consideration

20. In DTPA renal scan


TFTFT

What percentage of renal plasma flow


is filtered through the glomerulus and
what percentage is secreted by the
tubules

What percentage of renal plasma flow


is filtered through the glomerulus and
what percentage is secreted by the
tubules
20% of renal plasma flow is cleared by
glomerular filtration.
80% by tubular secretion.

Which radiopharmaceuticals are most


often used clinically for measurement
of GFR and ERPF?

Which radiopharmaceuticals are most


often used clinically for measurement
of GFR and ERPF?
GFR : Tc-99m DTPA
ERPF : I-131 OIH (iodohippurate) or Tc-99m
MAG3
However Tc-99m MAG3 does not actually
measure ERPF and a correction factor based
on proportional clearance compared with I131 OIH must be applied.

What is the percentage of cortical


binding of Tc-99m DMSA and Tc99mGH?

What is the percentage of cortical


binding of Tc-99m DMSA and Tc99mGH?
Tc-99m DMSA :40-50%
Tc-99m GH : 10-20%

What is Websters rule?

What is Websters rule?


Pediatric radiopharmaceutical doses can be
estimated using the formula (age + 1)/(age +
7) x adult dose.

The time to peak activity of a renal


time activity curve (TAC) represents:
A. The end of extraction
B. The beginning of renal clearance
C. The time point at which the amount of
cortical uptake of the radiopharmaceutical is
equal to clearance

The time to peak activity of a renal


time activity curve (TAC) represents:
FFT

What are the general method for


calculating absolute GFR?

What are the general method for


calculating absolute GFR?
Blood sampling, blood sampling and urine
collection and camera based methods.

At what step in the renin-angiotensinaldosterone cascade does captopril


work?

At what step in the renin-angiotensinaldosterone cascade does captopril


work?
Captopril blocks the conversion of angiotensin
I to angiotensin II in the lung.

Which of these factors effects the


accuracy of diuresis renography?
A.
B.
C.
D.
E.

State of hydration
Renal function
Dose of diuresis
Radiopharmaceutical
Bladder capacity

Which of these factors effects the


accuracy of diuresis renography?
TTTTT

What is the most sensitive technique


for diagnosing scarring secondary to
reflux?

What is the most sensitive technique


for diagnosing scarring secondary to
reflux?
Tc-99m DMSA

How can radionuclide imaging


differentiate upper from lower UTI?

How can radionuclide imaging


differentiate upper from lower UTI?
Upper UTI or pyelonephritis , Tc-99m DMSA
shows tubular dysfunction, manifested by
decreased uptake.
This is a reversible process.
With appropriate therapy, tubular will return
in 3 6 months.
Upper UTI has prognostic implication because
it may lead to subsequent renal scarring, HPT
and renal failure.

What is meant by direct vs indirect


cystography and which is the preferred
method in detecting VUR?

What is meant by direct vs indirect


cystography and which is the preferred
method in detecting VUR??
Direct cystography is sensitive, requiring
urinary tract catheterization and infusion of
radiotracer into the bladder. Reflux can be
detected during bladder filling and voiding.
Indirect method, where a routine renogram is
initially performed, cannot be used to detect
reflux during the bladder filling stage because
radiotracer is flowing through the collecting
system antegrade.

What is the common development


abnormality leading testicular torsion?

What is the common development


abnormality leading testicular torsion?
The bell-clapper testis

What is the difference in blood supply


to the testis and scrotum?

What is the difference in blood supply


to the testis and scrotum?
The testes receive the blood from testicular
artery.
Scrotum receives supply from the pudendal
vessels.

Urinoma
1. A photopenic region, best seen on early images, involves
most of the left renal fossa. Only the upper pole is
functioning. Urinary clearance into the left renal pelvis
appears displaced medially by the photopenic defect. The
right pelvic and uppe2/3 of the ureter fill. Poor clearance
bilaterally. Delayed images show increased uptake in the
region of the initial cold defect and inferior to it.
2. The cold defect is a urinoma with an attenuating mass
effect. Over time the radioactive urine enters this space
and mixed with the nonradioactive urinoma. Activity in
the region of the urinoma increases over time while the
earlier seen kidney and background activity have cleared.
3. Activity urinary leak and urinoma.
4. Urinary tract obstruction.

Bilateral obstruction
1. Left kidney; delayed and decreased cortical uptake, no
clearance into the calyces or pelvis. Right kidney;
prompt uptake and clearance into the collecting
symptom, faint persistent visualization of the right
ureter and, poor response to furosemide.
2. Before furosemide; high grade obstruction on the left
and HN on the right, suspicious of obstruction. A
furosemide high grade obstruction on the left and
obstruction on the right.
3. Filling of the renal collecting system.
4. Percent radiopharmaceutical uptake by each kidney
divided by total renal uptake between 1 and 3
minutes (before clearance into the pelvicalycial
system)

Torsion of testicular appendage


1. A, Mildly increased flow and mild focal left upper
scrotal uptake on the static image. B, Increased flow
and increased distribution on the left hemiscrotum. C,
Increased flow and delayed halo sign of the right
hemiscrotum.
2. A, Torsion of a testicular appendage. B, Acute
epididymitis/orchitis. D, Delayed testicular torsion.
3. A, Loss of blood supply to the appendage of the
testicle. B, Infection, viral or bacterial. C, Infarction of
the testicle caused by torsion more than 24 hours
duration.
4. Testicular torsion, 35%; torsion of the appendix testis,
35%; acute epididymitis, 25%.

Radionuclide cystography
1. Tc-99m DTPA and Tc-99m SC
2. Radionuclide cystography is more sensitive in detection of
VUR and results in 50 to 200 times less radiation exposure
to gonads compared with the contrast study.
3. The direct method is commonly used and requires urinary
catheterization and installation of radiotracer into the
bladder through catheter. The indirect method is
performed after routine DTPA/MAG3. when the bladder is
full, a prevoiding image is obtained, followed by dynamic
images during and after voiding.
4. Grading criteria are similar to those used with contrast
cystography, however the radionuclide studys limited
resolution does not permit assessment of calyceal
morphology. Mild reflux; confined to the ureter.
Moderate; reaches the pelvicalyceal system. Severe;
distorted collecting system and dilated tortuous ureter.

Unilateral obstruction
1. Good symmetrical cortical uptake and prompt
excretion into collecting systems bilaterally.
Retention of activity in left renal collecting
system, apparent cutoff in the upper ureter and
very poor response of furosemide. The right side
shows a prominent collecting system but washes
out spontaneously before furosemide.
2. HN of the left kidney. TRO obstruction.
3. Consistent with significant obstruction of the left
kidney.
4. Dehydration, renal insufficiency, inadequate
diuretic dose, full bladder and large collecting
system.

Renal scan
1. Tc-99m DTPA, Tc-99m MAG3 and I-131 hippuran.
2. DTPA; glomerular filtration, MAG3; tubular secretion
and hippuran; 20% glomerular and 80% tubular.
3. DTPA is inexpensive, provides a good image quality
but has low extraction efficiency (10-20%) and poor
poor quality images with renal insufficiency. MAG 3
has a high extraction rate (60%), good target to
background and good quality images with renal
insufficiency. Hippuran has good extraction efficiency,
high target to background, poor image quality, poor
cortical/collecting system differentiation and delivers
a high radiation dose in renal insufficiency.
4. No . A dose of 5 mCi is needed for good blood flow
images.

Diuretic renography
1. Bilateral cortical uptake and excretion into collecting
systems. Retention in the right collecting system at 30
minutes with good post furosemide washout.
2. Good response to surgical correction with no obstruction.
3. Not with certainty. Ureteral nonvisualization is not
diagnostic of ureteropelvic junction obstruction because a
standing column of ureteral urine can prevent radiotracer
entry.
4. It measures pressure flow relationships and requires
fluoroscopically guided trocar or spinal needle insertion
into the renal pelvis. Basal and pressure measurements
during infusion of a contrast solution at a rate are
recorded. Obstruction pressure is defined as greater than
15cm water, no obstruction as less than 10 to 12 cm
water.

Transplanted kidney
1. Rapid leakage of urine just inferior to the
transplanted kidney and extravasating into the
scrotum.
2. Urinary leak caused by disruption of surgical
anastomosis.
3. Hematomas and abscess occur in the early
postoperative course, whereas lymphoceles
generally are noted 4 to 8 weeks after surgery.
4. ATN, acute rejection or obstruction. Cyclosporin
toxicity usually occurs months after transplant.

Non viable kidney after


transplantation
1. No blood flow to the transplanted kidney. No
renal uptake. A photopenic region in the
shape of the transplanted kidney.
2. Nonviable kidney
3. Arterial or venous thrombosis, severe
irreversible rejection and acute cortical
necrosis.
4. Removal of nonviable transplanted kidney

ATN
1. ATN, accelerated acute rejection, urinary leak
and urinary obstruction.
2. The second postoperative week. Accelerated
rejection may occur during the first week in
patients who have had previous transplants or
received multiple transfusion.
3. Normal blood flow, very poor function, no
excretion. Base of penis seen inferiorly.
4. The pattern of normal blood flow but poor
function during the first week after
transplantation is typical of ATN

Testicular torsion
1. Tc-99m pertechnetate, initial blood flow and
then tracer distributes in the extracellular fluid
space.
2. Acute epididymitis, testicular torsion or torsion
of the testicular appendage.
3. Developmental abnormality of testicular
descent and attachment predisposes to
spermatic cord torsion. The most common
anatomical abnormality is bell clapper testis.
4. Decreased blood flow to the right testicle and a
photopenic right testicle consistent with acute
testicular torsion.

Acute renal transplant rejection


1. Very decreased and delayed blood flow and
poor transplant function.
2. Acute rejection.
3. Fever,
transplant
tenderness
and
enlargement, decreased urinary output and
rising serum creatinine level.
4. Biopsy.

Captopril renography
1. With renal artery stenosis, glomerular filtration decreases
and GFR drops. Renin released from juxtaglomerular
convert angiotensin I to angiotensin II. Angiotensin II
causes vasoconstriction of the glomerular efferent
arterioles, rising filtration and maintaining GFR. ACE
inhibitor blocks conversion of angiotensin I to II resulting
in decreased in a decrease GFR.
2. The right kidney is small but with good function. With
captopril cortical retention persists, consistent with renin
dependent renovascular HPT of the right kidney. This is
confirmed by the renal cortical time activity curves.
3. Yes. The accuracy of I-131 hippuran, Tc-99m DTPA and Tc99m MAG 3 are similar.
4. Sensitivity 90% and specificity 95%. Sensitivity is less for
detection of renin dependent HPT if the patient has been
taking an ACE inhibitor chronically or renal insufficiency.

Pyelonephritis
1. 40% of DMSA binds and fixes to functioning
proximal cortical renal tubules.
2. Diagnosis of pyelonephritis or renal scarring.
3. Decreased uptake in the lower half of the
right kidney on initial imaging (A). Repeat
SPECT show normalization of uptake. A,
Pyelonephritis. B, Renal scarring
4. DMSA in the early stages of infection is the
best
predictor
of
renal
sequalae.
Identification of pyelonephritis will increase
the duration of antibiotic therapy.

Lung scintigraphy

Pearls & Pitfalls


Unless it is completely and
absolutely
normal,
never
interpret a V/P scan without a
recent chest radiograph.
Lungs perfusion
A normal perfusion lung scan
essentially excludes clinically
significant PE.
An abnormal ventilation scan
or chest radiograph will not
change this assessment.

Pearls & Pitfalls

Lungs perfusion

Other causes of perfusion


defects are COPD, asthma,
tumor, mediastinitis, mucous
plug, fat emboli and vasculitis.
Most perfusion defects caused
by PE are wedge shaped and
extend to the periphery,
usually bilateral and multiple.
An unmatched defects refers to
one that is seen on the
perfusion
scan
without
ventilation abnormality.

Pearls & Pitfalls


Even a low probability scan has
a 15% to 20% probability of PE.
A very low probability scan has
a positive predictive value of
Lungs perfusion
less than 10%.
Asthma, mucous plugs and
COPD can cause segmental
defects, but they should not
have normal ventilation scans.

Pearls & Pitfalls

Lungs perfusion

A stripe sign of peripheral activity


around a perfusion defect is
frequently seen with COPD and
indicates a very low probability
defect.
When there is an infiltrate on the
CXR and a small perfusion defect,
pneumonia is a common cause.
When there is a small infiltrate
and a relatively larger perfusion
defect, PE should be considered.

Pearls & Pitfalls

Lungs perfusion

If there is ventilation of one


whole lung but no perfusion, the
differential diagnosis includes
congenital absence of the
pulmonary
artery,
massive
central PE, mediastinal fibrosis or
hilar neoplasm.
A focal hot spot in the lung is due
to an injected labeled clot that
was either formed in the syringe
or dislodged from the end of the
central line through which Tc99m MAA was injected.

Pearls & Pitfalls

Lungs perfusion

On an aerosol ventilation scan,


collection of activity in the
central bronchi is an indication of
COPD.
Poor perfusion to the lung apices
may be normal after lung
transplantation.
Large segmental V/Q mismatches
are more likely to represent PE
than the small ones.
Multiple irregular tiny perfusion
defects can be due to COPD, fat
emboli and vasculitis.

Pearls & Pitfalls

Lung perfusion

Parenchymal renal activity on a


Tc-99m MAA scan indicates a
possible right to left shunt.
This can be confirmed by
noting activity in the brain.
V/Q scans remain useful for
renal failure patients and
contrast allergies.

Lungs

Perfusion

Tc-99m human
Albumin microspheres

ventilation

Tc-99m MAA
Macroaggregated
albumin

Xenon 133

Radioactive
gases

Xenon 127

Krypton 81m

Radioaeresols

Tc-99m DTPA

Tc-99m
technigas

Xenon 133

Tc-99m DTPA

Mode of decay

Beta minus

Isometric

Half life

5.5 days

6 hours

Biological half life

30 seconds

45 minutes

Photo energy

81 keV

140 keV

Multiple view imaging

No

Yes

Useful for severe


COAD

Yes

-/+

Used after perfusion


scan

No

No

Tc-99m MAA
Size

10 90 m

Minimum number of particles to 100000 unless


pulmonary
be used in adults
hypertension and right to left
shunt
Ideal number of particles

200000 - 500000

Biological half life

48h

Injection

Care should be taken not to


cause particle aggregates that
can produce hot spot

Safety

Particles block <1/1000 of the


capillaries
and
precapillary
arterioles

Terminology
V/Q match defect
V/Q mismatch
Triple match
Segmental defect
Reversed mismatch

Both scans abnormal in same area


and equal size
Abnormal perfusion
Normal ventilation
V/Q matching with CXR
Wedge shape and pleural base
Large > 75%
Moderate 25 -75%
Small < 25%
Abnormal ventilation
Normal perfusion

Xenon-133 ventilation scintigraphy


Patient preparation

None

Dosage

10-20 mCi inhale

Collimator

LEGP

Photopeak

20% window centered at 81 keV

Positioning

Patient is seated
Place nose clamps on patient and connect
mouthpiece for several minutes
Center camera over chest posteriorly
Patient breathes continuously through
mouthpiece for several minutes

Acquisition

First breath- patients exhales fully and is ask to


minimally inspire and hold it long enough to
obtain 100000 count or 10-15 seconds
Equilibrium-obtain 2 sequential 90 seconds
images while patient breath normally
Washout-turn system to exhaust
Obtain 3 sequential 45 seconds posterior
images then right and left posterior oblique
and final posterior image

Tc-99m DTPA ventilation scintigraphy


Patient preparation

None

Dosage

30 mCi Tc-99m DTPA nebulizer

Collimator

LEGP

Photopeak

20% window centered at 140 keV

Positioning

Place nose clamps on patient and connect


mouthpiece for several minutes
Center camera over chest posteriorly
Patient breathes continuously through
mouthpiece for several minutes

Acquisition

Acquire posterior image for 250k and


mark time
Views: posterior/anterior/right and left
lateral/right and left posterior oblique

Tc-99m MAA perfusion scintigraphy


Patient preparation

None

Patient precaution

Pulmonary hypertension : reduces


particle to 100000
Right to left cardiac shunt : reduced
particle number

Dosage

2-5 mCi IV over several respiratory cycle


with patient supine

Collimator

LEGP

Photopeak

20% window centered at 140 keV

Acquisition

Acquire posterior image for 500k


counts/image
Views: posterior/anterior/right and left
lateral/right and left posterior oblique

Understanding the report

Mismatched or matched V/Q scan

Understanding probability

Qualitative lung scan

Mismatched decreased perfusion with


normal ventilation
Matched abnormal perfusion and
abnormal ventilation
Ventilation abnormal image shows
slow to wash in/ slow to wash out (wash
in images are obtained in all ventilation
agent but wash out only with xenon gas)
Normal scan- low probability of PE (0-5%)
Low probability scan -PE (10-15%)
Intermediate probability -PE (25-30%)
High probability PE (80-100%)

Describe the percent contribution to


global perfusion of each lung
Combined with pulmonary function test

Potential problems
False positive

Conditions that can mimic high probabilityvasculitis / pulmonary arterial anomalies i.e.
hypoplasia and stenosis

Clinical pretest probability of PE

Useful for deriving post test probability

Interpretations of defect size

High probability 2 moderate / large


mismatched perfusion defects
If patient with cardiopulmonary disease- 4
mismatched defects
Moderate 25-27% area lung segment
Large 75% segment
2 defects each 50% of segment can be counted
as 1 large defect

Widespread ventilatory abnormalities

Widespread ventilatory abnormalities =


intermediate probability of PE
i.e. pulmonary infarction or pneumonia showed
matched V/Q and matched CXR infiltrate

Right to left shunt

Number of particle can be reduced for


patients with right to left shunt i.e.
children.

Clinical pre scan probability

V/Q scan
probability for
PE

High

Intermediate

Low

High

97%

88%

56%

Intermediate

66%

28%

16%

Low

40%

16%

4%

Technegas (ventilation)

DTPA (perfusion)

Patient with COPD

Primary vascular lesion


PE
Vasculitis
Septic/fat/air emboli
Pulmonary artery hypoplasia or atresia

Primary ventilation
abnormality

Causes of perfusion
defect

Pneumonia
Atelectasis
Pulmonary edema
Asthma
COPD
Bullae

Mass effect
Tumor
Adenopathy
Pleural effusion

Iatrogenic
Surgery
Radiation fibrosis

Acute PE
Chronic PE
Vasculitis

V/Q mismatched
defect

Mediastinal or hilar
adenopathy
hypoplasia or aplasia of
pulmonary artery
Post radiation therapy
Bronchogenic carcinoma
Septic/drug abuse

COPD

V/Q matched defect

Air space disease


Tumor
Pleural effusion
Asthma
Lung infarction

Pneumonia

V/Q reversed
mismatched defect

Mucus plugging

Alveolar pulmonary edema

Pneumonectomy
Mediastinal fibrosis

Unilateral lung
perfusion
abnormality

Pneumothorax
Tumors
Mucous plug
Pulmonary embolus
Pulmonary artery stenosis
Swyer-James syndrome
Massive pleural effusion

Pacemaker
Bullae
Trauma

Non segmental
defect

Tumors
Hemorrhage
Cardiomegaly

Hilar adenopathy
Atelectasis
Pleural effusion
Pneumonia

Perfusions defect

ventilation

CXR

Probability category

2 segments

Mismatch

clear

high

<2

Mismatch

clear

Intermediate

Match

LL zone

Intermediate

Match

UL zone

Low

Match

Clear

Low

>3

N/A

Clear

Low

1-3

N/A

Clear

Very low

N/A

Shows anatomical
reason for perfusion
abnormality

Very low

N/A

N/A

Normal

Moderate to large

Multiple
Small (<25% of a segments)

Non segmental

None

Wells clinical prediction rule for the diagnosis of pulmonary embolism (PE)

Variable

Points

Clinical signs and symptoms of DVT

Alternative diagnosis is less likely than PE

Heart rate > 100 bpm

1.5

Immobilization or surgery in the previous 4 weeks

1.5

Previous DVT or PE

1.5

Hemoptysis

Malignancy

Risk categories based on Wells score


(version used in PIOPED II)

Risk category

Cumulative score

Low

<2

Intermediate

2-6

High

>6

Risk categories based on Wells score


(version used in Christopher study)

Risk category

Cumulative score

PE unlikely

PE likely

>4

Dyspnea (73%)

Most common symptoms of


PE (PIOPED study)

Pleuritic chest pain (66%)


Coughing (37%)
Hemoptysis (13%)

Abdominal/ pelvic pain


Decreasing level of
consciousness

Atypical
presentations of PE

Fever
Productive cough
Seizures
Syncope
Wheezing

AMI
Acute stroke
DIC
Advanced age

Conditions
associated with DDimer production

Heart failure
Connective tissue disease
Infection
Malignancy

Post operative
Pregnancy
Renal failure
Trauma
Sickle cells
DVT

The following associated with matched


ventilation/perfusion defects of lung
scans
A.
B.
C.
D.
E.

IV drug abuse
Congenital rubella syndrome
Histoplasmosis
Thymolipoma
Haematogenous metastases

The following associated with matched


ventilation/perfusion defects of lung
scans
TFTFF

The following may cause perfusion


defects on lung scan
A.
B.
C.
D.
E.
F.
G.
H.
I.

SLE
TB
Sarcoidosis
Retrosternal goitre
Radiation pneumonitis
Anomalous pulmonary artery
Bronchial carcinoma
Congenital lobar emphysema
Haemangioendoliomatosis

The following may cause perfusion


defects on lung scan
TTTTTTTTT

Concerning PE
A. A single perfusion defect separated from the
pleural surface by a rim of normal perfusion is
rarely due to embolus
B. Normal perfusion exclude clinically significant PE
C. The combination of clinical impression and lung
scan results is a better predictor of PE than
either taken alone
D. Patients with a low probability scan and no
evidence of venous thrombosis may be safely
left untreated
E. He particle count of Tc-99 MAA may need to be
increased in patients with pulmonary
hypertension to achieved diagnostic images

Concerning PE
TTTTF

Total loss of perfusion to one lung


occurs in
A.
B.
C.
D.
E.

Bullous emphysema
Beckwith-Wiedmann syndrome
Following a Blalock-Taussig shunt
Pleural effusion
Swyer-James syndrome

Total loss of perfusion to one lung


occurs in
TFTTT

Regarding V/Q scanning in the


following up of confirmed PE
A. Perfusion defects may clear at 24 hours
B. A perfusion defect still present at three
months is likely in persist indefinitely
C. A persistent defect implies lung infarction
D. Underlying heart failure may enhance the
resolution f perfusion defects
E. A repeat V/Q scan should be performed prior
to stopping anticoagulation

Regarding V/Q scanning in the


following up of confirmed PE
TTFFT

The fissure sign occurs in


A.
B.
C.
D.
E.

Sub pulmonic effusion


Cystic fibrosis
Sarcoidosis
PE
Emphysema

The fissure sign occurs in


TTFTT

Reverse V/Q mismatched occurs in


A.
B.
C.
D.
E.

Pulmonary atelectasis
PE
Alveolar proteinosis
Bronchogenic carcinoma
Pleural effusion

Reverse V/Q mismatched occurs in


TFFTT

The visualization of the kidneys on Tc99m MAA perfusion scan can be


caused by:
A.
B.
C.
D.
E.

Right to left shunt


Free Tc-99m
MAA particle less than 10 m in size
Kidney obstruction
Less dehydration

The visualization of the kidneys on Tc99m MAA perfusion scan can be


caused by:
TTTFF

A. Low probability
B. Intermediate probability
C. High probability
1.
2.
3.
4.
5.

Clear CXR with multiple matched V/Q defects


Single large segmental V/Q mismatch
Perfusion defect < radiological infiltrate
Multiple small perfusion defect
Perfusion defects equal to radiographic
infiltrate
6. Lobar V/Q mismatch

A. Low probability
B. Intermediate probability
C. High probability
1. Clear CXR with multiple matched V/Q
defects. A
2. Single large segmental V/Q mismatch. B
3. Perfusion defect < radiological infiltrate. A
4. Multiple small perfusion defect. A
5. Perfusion defects equal to radiographic
infiltrate. B
6. Lobar V/Q mismatch. C

Stripe sign generally is reflective of


A.
B.
C.
D.
E.

PE
Non embolic disease such as COPD
Pulmonary hypertension
Pleural fluid
Asthma

Stripe sign generally is reflective of


FTFFF

Matched absence or near absence of


perfusion and ventilation in the entire
lung may be associated with?
A. Lung carcinoma
B. Aspirated foreign body
C. Swyer James syndrome

Matched absence or near absence of


perfusion and ventilation in the entire
lung may be associated with?
TTT

Regarding V/Q
A. Kr-81m is generator produced
B. Rb-81 is cyclotron produced
C. Albumin macroaggregates remain in the
pulmonary capillary bed for weeks, limiting the
number of follow up perfusion scans that can be
safely performed.
D. The biologic half life of MAA is longer than
albumin microspheres
E. A patient with high pretest probability of PE, the
negative predictive value of a normal scan
exceeds 95%

Regarding V/Q
TTFFT

Regarding V/Q
A. In a patient with low pretest probability of PE,
the positive predictive value of positive scan
exceeds 95%
B. The presence of the triad has a positive
predictive value of PE exceeding 95%
C. V/Q scan is nondiagnostic in most patients with
pulmonary edema
D. Most patients with PE show infiltrate or effusion
on properly exposed CXR
E. Absence of perfusion to an entire lung is
common with PE

Regarding V/Q
FFFFF

What are two commonly used


radiopharmaceuticals for ventilation
imaging? What are their advantages
and disadvantages

What are two commonly used


radiopharmaceuticals for ventilation
imaging? What are their advantages
and disadvantages
Xenon-133 and Tc-99m DTPA aerosol. Xenon-133
demonstrates more clearly the physiology of
respiration and is very sensitive to detection of
obstructive airways disease manifested by slow
washout. The disadvantages is the rapid washout, its
suboptimal view due to low photopeak (81 keV) and
poor count rate image.
Tc-99m DTPA aerosol allows high count images in all
projections, however the images are comparable only
to the inspiratory phase of xenon-133.

What is minimum number of particles


recommended for pulmonary
perfusion imaging?

What is minimum number of particles


recommended for pulmonary
perfusion imaging?
This requires at least 100,000 particles in
normal adults and many authorities
recommend a minimum of 200000 500000
particles.

What is the size range of MAA


particles?

What is the size range of MAA


particles?
In commercial preparations the majority of
particles are 20 to 40 m with a range of 10
90 m.

What is the biological fate of MAA


particles?

What is the biological fate of MAA


particles?
MAA particles are easily broken down in the
lung. Delayed imaging performed several
hours after pharmaceutical administration
demonstrates
activity
in
the
reticuloendothelial system because of
phagocytosis of the breakdown particles.

Pearl
One way to determine whether radioactivity
outside of the lungs is caused by free Tc-99m
or shunted Tc-99m MAA is to image the brain.
Free pertechnetate should localize in the
brain, whereas Tc-99m MAA particles that gain
access to the systemic circulation will lodge in
the first capillary bed that they encounter,
including the capillary bed in the brain.

What is the preferred patient position


during administration of Tc-99m MAA?

What is the preferred patient position


during administration of Tc-99m MAA?
Administering Tc-99m MAA with the patient
supine results in a more homogenous
distribution of particles in the lung than when
the patient is sitting or standing.

What is the stripe sign?

What is the stripe sign?


Stripe sign refers to a stripe or zone of activity
seen between a perfusion defect and the
closest pleural surface. Because PE are
typically pleura based, the stripe sign suggests
another diagnosis, often emphysema.

What is physiological basis for


perfusion defects in areas of poor
ventilation?

What is physiological basis for


perfusion defects in areas of poor
ventilation?
The classic response to hypoxia at the alveolar
level is vasoconstriction. Shunting a blood
away from the hypoxic lung zone maintains
oxygen saturation.

What is the shrunken lung sign?

What is the shrunken lung sign?


The lung may appear smaller than usual in
patient sustaining multiple small emboli, such
as fat emboli, that distribute uniformly around
the lung periphery

What is the classical appearance of


multiple PE on lung perfusion
scintigraphy?

What is the classical appearance of


multiple PE on lung perfusion
scintigraphy?
Multiple pleura-based, wedged shaped areas
of significantly diminished or absent
perfusion. The size of the defect may vary
from subsegmental to segmental or may
involve an entire lung or lobe.

What is the sensitivity of the high


probability scan category for detecting
pulmonary embolism?

What is the sensitivity of the high


probability scan category for detecting
pulmonary embolism?
In the PIOPED study, 41 % of patients with PE
had a high probability scintigraphy pattern.

What are the most common clinical


signs and symptoms in patients with
confirmed PE?

What are the most common clinical


signs and symptoms in patients with
confirmed PE?
In PIOPED, the three most common presenting
symptoms, dyspnea 80%, pleuritic chest pain
60% and cough 40%.
On physical examination, lung crackles 60 %,
leg swelling 30% or pleural friction rub 5%.

High probability of PE
1. Perfusion is decreased in the right lower lobe
except for the superior segment. Ventilation is
truncated in the right lower lobe consistent with
subpulmonic effusion
2. High probability for PE. Mismatch between
perfusion and ventilation is evident in the basal
segment. The perfusion defect is considerably
larger than the effusion on the X-ray
3. > 90%
4. Most common : Normal
5. Next most common : atelectasis

Hot spots on lung scan


1. Multiple hot spot are present in the upper and lower
lobes, predominately in the right lung field
2. Radioactive emboli as a result of poor technique
caused by drawing back blood into the syringe
containing the Tc-99m MAA before injection of the
radiotracer, causing clumping
3. The lungs, the target organ, received 5 mCi from Tc99m MAA. Approximately 0.2 rads to the lung from a
20 mCi xenon study
4. The biological half life is brief. The patient breathes
Xe-133 gas to equilibrium, then expels it into a trap.
Only Xe-133 absorbed as a result of fat solubility has
any appreciable biological half life

Ventilation perfusion stripe sign,


emphysema
1. Decreased upper lobe ventilation is seen on the
single breath with air trapping in both upper
lobes and the right lower lobe on washout
images
2. Decreased perfusion to the majority of both
lungs, with preserved perfusion in the
subpleural lung, most evident at the lung bases
and medial aspect of both upper lobes
3. Low probability
4. Stripe sign

Hamptons Hump intermediate


probability
1. Posteroanterior and lateral chest radiographs
demonstrate a pleural based opacity in the lateral
right lung base
2. A single wedge shape, pleural based defect in the
same location as the radiographic abnormality,
probably the anterobasal segment of the right lower
lobe. Normal Xe-133 ventilation study
3. Intermediate probability for PE
4. CXR findings in PE without infarction are uncommon.
When present, they are usually associated with a
large, central embolus. Discoid atelectasis is the next
most common finding

Emphysema caused by alfa 1


antitrypsin deficiency
1. Single breath or wash in, equilibrium and wash out phase
2. Single breath : patient breathes in and holds a single
maximum deep inspiration while a 100000 count image is
acquired. Equilibrium : patient breathes a mixture of air
and xenon while serial images are obtained every 60 90
seconds for 3 minutes. Washout : patient breathes room
air and exhales xenon while serial images are obtained
3. Ventilation : nonuniform in the upper lung zones
bilaterally, initial near absent at the bases. As upper lobes
wash out, xenon fills and is retained in both bases
indicating severe air trapping. Perfusion : heterogeneous
to both upper lung zones that match the early ventilation
images. The extensive perfusion abnormalities in both
lower lung zones are matched with areas of washout air
trapping
4. Low probability for PE, alfa 1 antitrypsin deficiency

Tc-99m DTPA ventilation study with


aerosol clumping
1.

2.
3.

4.

Multiple perfusion defects in upper and lower lung fields. Many appear
segmental e.g.., the lateral of the right lower lobe, superior segment of
the left lower lobe. Ventilation shows extensive diffused clumping within
the air ways throughout both lung fields making determination of
matching or mismatching difficult. The study was interpreted as
intermediate probability since ventilation study could not be interpreted,
however the segmental perfusion defect pattern is suspicious for
embolus
Tc-99m DTPA aerosol provide (0.1 0.5 m) normally distribute on firs
impact within alveoli. With airway turbulence e.g.., asthma or COPD,
particles impact proximally within bronchi and appear as focal hot spot
Tc-99m DTPA aerosol ventilation is performed first. The patient breathes
in less than 1 mCi at tidal volume until an adequate count rate is
obtained the sixfold larger Tc-99m MAA perfusion dose (5 mCi)
overwhelms the retained ventilation dose, allowing the two consecutive
Tc-99m studies
Xe-133 an inert gas, is advantageous for patients with COPD and asthma.
Delayed filling and clearance in regions of obstructive disease can be
seen. Disadvantage : only posterior views are possible because of rapid
exhalation

Low probability V/Q scan


1. Bilateral inhomogeneous distribution. Matched
perfusion and ventilation abnormalities throughout
the upper and lower lobes, especially in the right
lower lobe basal segments. Minimal atelectasis in the
lower lobes in chest x-ray. Low probability for PE
2. < 20%
3. < 1 %
4. Large (segmental): > 75% of a segment
Moderate (subsegmental): 25% to 75% of a segment
Small (small subsegmental):< 25% of a segment

Intermediate probability and


pregnancy
1. Bronchodilator therapy before V/Q scan as an asthmatic
therapeutic trial
2. PE are life threatening to the patient and fetus. The radiation
dose to the patient and fetus is low (< 2 rads). The benefit/risk
ratio is high. No change in procedure is required. In young,
nonsmoking patients without cardiopulmonary disease, one
might reduce the dose, conduct only a perfusion study or both
3. Hypoperfusion of the left lower lobe, except the superior
segment. Better ventilation than perfusion with mismatching in
the anterior basal and part of the lateral basal segments. No
ventilation in the posterior basal and part of the lateral basal.
Costrophenic angle loss seen in the left lateral and LPO views.
CXR: atelectasis/ infiltrate with elevation of the left diaphragm
4. Intermediate probability for PE. One large/ one moderate
segmental mismatch and one large and one moderate segmental
match corresponding to the x-ray atelectasis/infiltrate

High probability of PE
1. Perfusion: decreased right lower lobe except for
the superior segment. Ventilation: normal
except for mildly truncated right lower lobe
consistent with subpulmonic effusion
2. High probability of PE. Mismatch between
perfusion and ventilation in the basal segments.
The perfusion defect is considerably larger than
the pleural effusion on the radiograph
3. > 80%
4. Most common: normal, next most common:
atelectasis

Stripe sign
1. Decreased perfusion in the right upper lobe
(apical and anterior segment). Stripe sign of the
posterior segment of the right upper lobe. Normal
ventilation
2. Two segmental mismatched. High probability of PE
3. Its presence signifies perfused lung tissue
between a perfusion defect and the adjacent
pleural surface. Its presence can be use to classify
a segment as not related to PE.
4. Usually a manifestation of airway obstruction. The
sign has been correlated with CT and PET showing
spared perfusion of the cortex of the lung in
asthma and emphysema

Unilateral matched V/Q abnormality


1. Perfusion: global decreased perfusion to the
entire left lung, more striking than the
ventilation.
Xe-133
ventilation
images:
decreased and delayed ventilation of the entire
left lung. Washout images show no significant
air trapping
2. Low probability
3. Hilar mass, severe unilateral parenchymal lung
disease, prior shunt for congenital heart disease
4. Lung cancer

PET and oncology

A : without
attenuation
corrected
B : with attenuation
corrected

HG = high grade

If spleen > liver = post chemo

Ur Metser, MD, Einat Even-Sapir, MD, PhD, Semin Nucl Med 37:206-222 2007

Ur Metser, MD, Einat Even-Sapir, MD, PhD, Semin Nucl Med 37:206-222 2007

Ur Metser, MD, Einat Even-Sapir, MD, PhD, Semin Nucl Med 37:206-222 2007

Ur Metser, MD, Einat Even-Sapir, MD, PhD, Semin Nucl Med 37:206-222 2007

Ur Metser, MD, Einat Even-Sapir, MD, PhD, Semin Nucl Med 37:206-222 2007

Ur Metser, MD, Einat Even-Sapir, MD, PhD, Semin Nucl Med 37:206-222 2007

Ur Metser, MD, Einat Even-Sapir, MD, PhD, Semin Nucl Med 37:206-222 2007

Ur Metser, MD, Einat Even-Sapir, MD, PhD, Semin Nucl Med 37:206-222 2007

Ur Metser, MD, Einat Even-Sapir, MD, PhD, Semin Nucl Med 37:206-222 2007

Brown Fat
Sedation
Adrenergic blocking agents

Higher tumor grade


Large number of variable
cells
Increased tumor blood flow

Inflammation

High FDG uptake

Tumor hypoxia
Acute radiation

Recent chemotherapy
Recent surgery

Benign lesion
Necrosis

Low FDG uptake

Mucinous , bronchoalveolar
carcinoma
Hyperglycemia

High insulin
Chronic radiation

Scar
Prior chemotherapy

History

Course of action

Prior surgery

Delay scan 4-6 weeks

Chemotherapy

Delay scan several weeks or schedule


scan just before next cycle

Radiation therapy

Delay scan at least 3 months

Colony stimulating factor

Delay scan for 1 week for short acting


agents or several weeks for long
acting agents

Serum glucose > 200

Reschedule scan

Insulin administration

Wait at least 2 hours

Breastfeeding

Discontinue scan at least 6 hours

Which of these statements is true


regarding F-18 FDG PET?
A. F-18 FDG uptake is normally high in brain and
heart
B. The mechanism of F-18 FDG uptake and
metabolism is identical to that glucose
C. Oncology patients should fast for at least 4
hours prior to injection
D. Unlike glucose, F-18 FDG is excreted through
the genitourinary system

Which of these statements is true


regarding F-18 FDG PET?
TFTT

The sensitivity of FDG PET is high for


detection of many malignancies. For
which of the following tumors is the
sensitivity of FDG PET not high?
A.
B.
C.
D.
E.

Colorectal cancer
Melanoma
Hepatocellular carcinoma
Renal cell carcinoma
Lymphoma

The sensitivity of FDG PET is high for


detection of many malignancies. For
which of the following tumors is the
sensitivity of FDG PET not high?
FDG PET has poor sensitivity for primary
hepatocellular carcinoma, renal cell carcinoma
and prostate cancer.
This is less true of metastatic disease than
primary tumors.

Concerning F-18 FDG PET in lung


cancer staging
A. The sensitivity for detection is high for
tumors of 5mm and greater size.
B. Sensitivity and specificity of FDG PET is
higher than CT for mediastinal staging for
lung cancer.
C. False negatives may be seen with
hyperglycemia.
D. False
negatives
may
occur
with
bronchoalveolar carcinoma

Concerning F-18 FDG PET in lung


cancer staging
FTTT

What are the advantages CT PET over


PET alone?
A. Automated hardware and software fusion for
anatomical fusion
B. Diagnostic CT at the same time as the PET
scan.
C. CT is used for more rapid attenuation
correction.
D. Eliminates false positive interpretation.

What are the advantages CT PET over


PET alone?
TFTF

What are some limitations of FDG PET


in tumor staging?

What are some limitations of FDG PET


in tumor staging?
PET imaging does not detect microscopic
metastases, tumor involvement in local LNs
may be obscured by activity in an adjacent
tumor, concurrent infection/inflammatory
process may cause false positive and
sensitivity for intracranial metastases is low.

What are limitations of tumor


restaging by PET?

What are limitations of tumor


restaging by PET?
Posttherapy effect of surgery, chemotherapy
and radiation therapy may cause increased F18 FDG uptake, which can be confused activity
from active tumor.
Even patients scheduled for imaging after an
appropriate delay after therapy may require
follow up imaging.

What are some differences between


nonattenuation corrected and
attenuation corrected PET images?

What are some differences between


nonattenuation corrected and
attenuation corrected PET images?
Nonattenuation
corrected

Structures near surface more intense


The skins more intense
The lungs more intense

Lung carcinoma
PET sensitivity in 1 node is 75%
Pet sensitivity for mediastinal involvement is
91%
Adrenal is most common metastasis site

Lung carcinoma
Epidemiology

UK = 38000 cases
22000 men and 16000 women
Death = 34000 per annum
Peak incidence in Europe 60-70 years
3 x more common in men

Lung carcinoma

Small cell carcinoma


25%

Oat cell

Intermediate cell

Non small cell carcinoma


75%

Squamous cell
50%

Bronchoalveolar

Adenocarcinoma
15%

Acinar
Papillary
Solid

Large
Cell
Anaplastic
10%

Bronchial derived

Lung carcinoma
Symptoms

Cough
Hemoptysis
Dyspnea
Chest pain
Recurrent chest infection

Lung carcinoma
Signs
Finger clubbing
Nicotine staining
Cervical sympathetic (Horners syndrome)=
partial ptosis/ miosis (pupil constriction)/
enopthalmos/ anhydrosis
Wasting of hand

Lung carcinoma
Differential diagnosis
Benign
tumors-papilloma/
carcinoid/
leiomyoma
Metastasis
Primary malignant-mesothelioma/ soft tissue
sarcoma
Chronic lung abscess
Radiographic artifact-nipple shadow

Lung carcinoma
Investigation

CXR
Sputum cytology
Bronchoscopy
CT
MRI
Bone scan
Indirect laryngoscopy
Lung function test

Staging lung carcinoma based on TNM classification scheme


Stage

TNM

Ia

T1 N0 M0

Ib

T2 N0 M0

IIa

T1 N1 M0

IIb

T2 N1 M0
T3 N0 M0

IIIa

T3 N1 M0
T1-3 N2 M0

IIIb

T4 N0-2 M0

T1-4 N3 M0
IV

Any T Any N M1

Based on AJCC staging system 1997

Single pulmonary nodule


1. Only 20% to 30% are malignant. Higher for
smoker 50%.
2. CXR/CT criteria, 30% - 40% are
indeterminate, 50% are benign.
3. High.
4. False negative: lesion < 1cm, bronchoalveolar
carcinoma and carcinoid tumors. False
positive: benign tumor, inflammatory,
infection (TB).

Lung carcinoma staging


1. Focal increased uptake corresponding to the
nodule on CT. abnormal uptake is seen in the
right and left paratracheal regions, the right
and left hilum and mediastinum.
2. FDG PET has preoperatively upstaged the
patient, who is no longer a surgical
candidate.
3. 65%
4. 85%

Breast carcinoma
Epidemiology

UK = 45000 cases
325 arising in men
12000 death per annum
Peak age incidence 50-70 years

Breast carcinoma
Etiology

Family history
Menstruation > 12 or stop >55 years
On HRT
Women with no borne children
Women have had children late > 30 years
Overweight
Alcohol

Breast carcinoma
Classification

Ductal carcinoma in situ


Lobular carcinoma in situ
Inflammatory breast carcinoma
Pagets disease of the breast
Basal cancers
Male breast cancer

Breast carcinoma
Differential diagnosis

Fibroadenoma
Breast abscess
Fat necrosis
Galactocoele

Breast carcinoma
Investigations

Mammography
FNA
Ultrasound
MRI
Excision biopsy

(ipsilateral mobile axillary nodes)


(ipsilateral fixed axillary nodes)

(ipsilateral internal mammary nodes metastasis)

GIST

Mesenchymal neoplasm
70% arise from stomach
25% arise from small intestine
Express growth factor receptor with tyrosine
kinase activity (c-kit) detect by CD117
Highly malignant
Resistant to conventional treatment

Staging TNM
(Gastrointestinal)

Tis carcinoma in situ


T0 no evidence of primary
TX primary cannot be assessed
T1 involving lamina propria/submucosa
T2 involving muscularis propria/subserosa
T3 penetrates serosa
T4 involving adjacent structures
N0 1 6 nodes involved
N2 7 -15 nodes involved
N3 - > 15 nodes
M0 no distant metastasis
M1 distant metastasis

Colon carcinoma
Epidemiology
UK = 36000 cases, 22000 men and 16000
women
3rd commonest
16000 deaths per annum
Occur 50 years of age

Colon carcinoma
Etiology

Diet high meat/fat/calorie/alcohol


First degree
Polyps
Chronic Crohns colitis

Colon carcinoma
Symptoms

Change in bowel habit


Blood per rectum
Mucus per rectum
Tenesmus
Obstructive symptoms
Iron deficiency anemia

Colon carcinoma
Investigations

Digital PR examination
Double contrast barium enema
Colonoscopy
CT
MRI

Colon carcinoma
DUKES Staging
Dukes A/ stage A invasion into but not
through bowel wall
Dukes B/ stage B involving bowel wall but
not lymph nodes
Dukes C/ stage C involvement of nodes
Dukes D/ stage D - metastasis

TNM colorectal carcinoma

TNM colorectal carcinoma

Colorectal carcinoma
1. Increased uptake consistent with tumor in the
liver corresponding to CT mass. No other liver
lesions or metastases are seen elsewhere.
2. Surgical resection is planned. The preoperative
FDG PET scan is used to determine the presence
of any other metastases in the liver or
elsewhere.
3. a. increased serum CEA
with normal
conventional imaging. b. equivocal lesion with
conventional imaging. c. preoperative staging
before curative resection.
4. Negative scan. No evidence of tumor.

Colorectal carcinoma
1. Large abnormal region of increased uptake in
the pelvis consistent with recurrent tumor. A
large tumor is present in the left pelvis adjacent
to the bladder in addition to multiple small sites
of tumor uptake.
2. Tc-99m CEA is superior to CT in extrahepatic
abdomen and pelvis but equal to CT in detecting
tumor in the liver.
3. Rising serum CEA level with negative CT findings.
Potentially resectable recurrent disease, usually
in the liver, done to exclude other metastases
that would preclude surgery.
4. In-111 Oncoscint and FDG PET

Prostate carcinoma
Epidemiology
UK = 35000 cases = 12% cancer cases and
10000 deaths per annum
Age over 70 years
2nd most common after lung carcinoma

Prostate carcinoma
Symptoms
Prostatic outflow obstructive symptoms: poor
stream/ frequency/ terminal dribble/ nocturia
Erectile dysfunction
Haematospermia
Bone pain
Hypercalcemia

Prostate carcinoma
Investigations

PSA = normal 4 ng/ml


Blood test = hypercalcaemia
Biopsy
IVU
CT scan
Bone scan
MRI

Prostate carcinoma
1. In-111 labeled monoclonal antibody directed
against the prostate specific membrane antigen,
a glycoprotein expressed by normal and prostate
cancer cells.
2. CT and MRI sensitivity ranges from 5% - 20%.
Accuracy of In-111 ProstaScint is 70%.
3. Paraaortic upper abdominal uptake consistent
with tumor adenopathy. Focal uptake in the left
upper chest consistent with tumor.
4. Recurrent tumor limited to the prostate bed or
metastatic pelvic nodes require radiation ports.
Extrapelvic metastases require systemic therapy.

Cervical carcinoma
Epidemiology
UK = 2700 cases, 900 deaths per annum
Age 40 50 years
24000 carcinoma in citu (CIN III)

Cervical carcinoma
Etiology

HPV
Intercourse at early age
Lower socioeconomic group
Oral contraceptive
Miscarriage
Smoking

Cervical carcinoma
Symptoms

Vaginal bleeding
Vaginal discharge
Hematuria
Low back pain
Renal failure due
obstruction

to

bilateral

ureteral

Cervical carcinoma
Investigations

Cervical cytology
Biopsy
CT scan
MRI

Cervical carcinoma

Cervical carcinoma

Ovarian carcinoma
Epidemiology
UK = 6600 cases
2.3 % of all cancer cases
4400 cases deaths per annum

Ovarian carcinoma
Pathology
Common appearance
Psedomucinous cyst
Serous cyst
Microscopic appearance
Epithelial adenocarcinoma
Germ cell tumors

Ovarian carcinoma
Investigation

Serum CA 125
CXR
Ultrasound
CT scan

Ovarian carcinoma staging

Hodgkins Lymphoma
Epidemiology
UK = 1500 cases, 850 men and 650 women
with 350 deaths per annum
Two peak incidence in young 20 30 years
and over 70 year

Hodgkins Lymphoma
Etiology
Epstein Barr virus

Hodgkins Lymphoma
Symptoms
B symptoms:
1. Fever > 38C
2. Weight loss > 10%
3. Night sweat

Hodgkins Lymphoma
Sign

Enlarge nodes at neck


Hepatospleenomegaly
Abdominal mass
Inguinal nodes

Hodgkins Lymphoma
Differential diagnosis
Infection: pyogenic/ TB or viral
Leukemia
carcinoma

Hodgkins Lymphoma
Investigations

Blood count
ESR
Serum LDH
Alkaline phosphatase

Marrow
CXR
CT/ MRI
PET scan
Biopsy

Non Hodgkins lymphoma


Epidemiology
UK = 10000 cases, 5300 for men and 4800 for
women with 4500 deaths per annum
Occurs equally in men and women
More 50 years of age

Non Hodgkins lymphoma


Epidemiology
UK = 10000 cases, 5300 for men and 4800 for
women with 4500 deaths per annum
Occurs equally in men and women
More 50 years of age

Non Hodgkins lymphoma


Etiology
Infective agents : EBV/ helicobacter/ human T
cell lymphotropic virus/ HIV
Altered immune status
Irradiation

Non Hodgkins Lymphoma


Sign

Enlarge nodes at neck


Hepatospleenomegaly
Abdominal mass
Inguinal nodes

Non Hodgkins Lymphoma


Investigations

Blood count
ESR
Serum LDH
Alkaline phosphatase
CSF - DLBCL

Marrow
CXR
CT/ MRI
PET scan
Biopsy

Hodgkins Lymphoma

Classic HD
Nodular
lymphocyte
predominant

Lymphocyte rich
Lymphocyte depleted

Mixed cellularity
Nodular sclerosis

Lymphoblastic

High grade
Curable
Aggressive and urgent
treatment

Follicular
Lympho
plasmacytoid

Low grade
Indolent
Respond well to chemo but
difficult to cure

Burkitts

DLBCL

Sezary syndrome

Non Hodgkins lymphoma

Peripheral T cell
MALT

Mantle cell

Lymphocytic

Adult T cell
leukemia

Anaplastic
large cell

T cell

Angiocentric

Angioimunoblastic
Mycosis fungoides

Hodgkins disease
1. FDG is glucose analog. Increased FDG uptake occurs
with increased glucose metabolism. Is taken up
intracellularly and phosphorylated similar to glucose,
however, unlike glucose, it can not be metabolized
further and is intracellularly trapped.
2. a. intense multifocal uptake bilaterally in the neck,
upper chest and parasternal regions. b. uptake limited
to the left neck. Study a. abnormalities are caused by
muscle tension. Diazepam 5 mg was take just before
study b.
3. 110 minutes. 2. 10 and 20 minutes. It is important to
remember that the half life of positron are very short.
4. 3.2 rads/ 5 mCi to urinary bladder. Brain and heart.

Intracranial lymphoma
1. Tl-201 was used. Tc-99m sestamibi also can
be used, however it is taken up by choroid
plexus and could pose diagnostic problems in
some cases.
2. Tumor, particularly lymphoma, versus
infection, usually toxoplasmosis or infection
e.g.. CMV or herpes simplex.
3. Malignant lymphoma.
4. 90% sensitivity and false negative < 10%.

Lymphoma
1. CT scan is in indeterminate, but FDG PET
demonstrates a complete response.
2. CT assessment of tumor response is based on a
decreased in the size of the mass or complete
resolution. Post therapy residual masses are common
and CT cannot differentiate residual tumor from post
therapy fibrosis and necrosis.
3. Multiple high energy photopeak: 185, 300, 394
resulting poor image resolution, need delayed images
from 2 to 3 days.
4. Study completed 2 hours after injection. FDG PET
target to background ratio higher and image quality
superior to Ga-67.

Hodgkins disease
1. The bone scan shows mild increased uptake
at L3. The Ga-67 scan shows abnormal
uptake in the L3 vertebral body, bilateral
neck, mediastinum, right paratracheal
regions, posterior thorax, right lung base and
liver.
2. Bone scan.
3. Hodgkins
disease,
TB
or
atypical
mycobacteria; Hodgkins disease likely.
4. Stage IV.

Non Hodgkins lymphoma


1. Adult dose is 10 mCi. Imaging performed 48 to
72 hours after injection.
2. CT provides better sensitivity for initial staging.
Ga-67 is superior to CT after therapy and
residual masses after therapy.
3. a, large mass in the anterior mediastinum
extends to supraclavicular regions. b. complete
response to therapy. The residual chest mass on
CT caused by necrosis and fibrosis, not tumor.
4. Laxative may be given, and imaging may be
delayed as needed at 4 to 7 days after injection.

Axillary LNs

Sentinel node
Is the first lymph node bed which a tumor cell
would come if it penetrated into lymphatic
fluid.
If axillary sentinel node is tumor free < 3%
chance of any tumor metastasis
Radiopharmaceutical= Tc-99m SC injected
subdermally/ periareolar/ intradermal or
peritumoral

Scintimammography
1. Tc-99m sestamibi lipophilicity allows it to enter the
cell where it is concentrated in the mitochondrial
region related to charge.
2. Patient A has prominent focal uptake in a right breast
mass. Patient B has definite focal uptake at the
periphery of the breast prosthesis.
3. Accuracy of conventional mammography: sensitivity,
70% to 95%; positive predictive value for cancer, 20%
- 30%. Scintimammography trial: sensitivity/
specificity, 75%/83%
4. Most false negative findings are in lesions less than 1
cm. False positive findings occur in fibroadenomas
and benign and malignant tumors other than breast
cancer

Scintimammography
1. The 5 year survival rate for breast cancer
decreases with axillary node involvement.
Adjuvant chemotherapy is indicated.
2. A sentinel node biopsy drained by the
lymphatics in a nodal basin.
3. If the sentinel node biopsy is tumor negative, no
axillary dissection is needed. If positive, axillary
dissection is performed.
4. Tc-99m sulfur colloid is often used. It is injected
around the lesion or biopsy site. Imaging usually
is performed. At surgery a gamma probe is used
to help locate the sentinel node.

Head and neck carcinoma


PET sensitivity for recurrence is 76-96%
Negative predictive value > 90%

Thyroid carcinoma
PET sensitivity for poorly differentiated
carcinoma > 90%
Benefits for: anaplastic and Hurtle cell variant
of follicular

Medullary thyroid carcinoma


Arises from parafollicular cells
Does not accumulate I-131
PET sensitivity 78% and specificity 79%

Can evaluate with :


1. Somatostatin
2. DMSA pentavalent
3. Thallium 201
4. Tc-99m sestaMIBI

Esophageal carcinoma

PET sensitivity 95%


LNs involvement sensitivity >70%
LNs involvement specificity >90%
Distant metastasis accuracy 83%

Colorectal carcinoma

Primary sensitivity >90% : CT 60%


LNs involvement sensitivity 85 99%
LNs involvement specificity 71 87%
Alter management 29 36%

Melanoma
PET sensitivity > 90% and specificity 87%
PET alter treatment 20 26%

feto protein for hepatocellular carcinoma


Ca 19-9 for pancreatic carcinoma
PET less sensitive for hepatocellular carcinoma

Breast carcinoma

PET sensitivity 88% and specificity 79%


> 2-5 cm PET sensitivity 92%
< 2 cm PET sensitivity 68%
LNs involvement sensitivity 79-100% and
specificity 66-100%

False negative:
1. Well differentiated
2. Slow growing : lobular/tubular/DCIS

Ovarian carcinoma
PET sensitivity for staging and restaging (5090%)
PET specificity 60-80%
PET alters management 15% cases

Mucinous
cystadenocarcinoma

PET false ve in
ovarian carcinoma

Well differentiated
cystadenocarcinoma
Disseminated peritoneal
carcinomatosis
Borderline tumors
Stage I tumors confined to
the ovary

Testicular carcinoma
PET sensitivity 84%
PET negative predictive value is 94%

Neuroendocrine tumor

Radionuclide therapy
Hepatocellular ca

Intra vascular therapy


Phosphorus 32 phosphate

I-131 lipiodol

Bone metastasis

Rhenium 188

Rhenium 188 lipiodol


Phosphorus 32 phosphate

Intra articular therapy

Yittrium 90 microsphere
Rhenium 188 HEDP

Phosphorus 32 phosphate
Samarium 153 EDTMP

Strontium 89 chloride

Yittrium 90 citrate

Hyperthyroid / thyroid ca
I-131 NaI

Intra cavity therapy

Non Hodgkins Lymphoma

Polycythemia vera
Phosphorus 32 phosphate

Phosphorus 32 phosphate colloid

I-131 anti CD 20 antibody

HD
NHD
Hepatocellular
Melanoma
Lungs
Head & neck
Soft tissue sarcoma
Abd & pelvic tumors

Tumor imaging
Gallium - 67
DMSA(V)
MTC

Tc-99m

FDG

I-131

Thyroid ca

Tumor specific

MIBI/tetrofosmin
MTC
Breast

Non specific

NP-59
Adrenal cortex

MIBG
Adrenal medulla

MTC
Breast
Brain
Bone
MTC
Karposi sarcoma

Thallium-201

Peptide
I-111 pentreotide
Somatostatin
MTC

Tc-99m CEA
Colorectal

Monoclonal
Tc-99m Neo Tect
Somatostatin
Lungs ca

Organ specific
In-111 satumomab (onco scint)
Ovarian
colorectal

Cold

Thyroid
I-123
Tc-99m pertechnetate

Hot

Liver
Tc-99m SC

Bone
Tc-99m MDP
Tc-99m HDP

Brain
Tc-99m DTPA
Tc-99m glucoheptonate

Hepatocyte

Tc-99m HIDA

I-111 zevalin
NHD

In-111 capromab
Prostate

What is the mechanism of G-67 uptake


in tumors?

What is the mechanism of G-67 uptake


in tumors?
Its binds to serum iron transport molecules such
as transferrin, which transport Ga-67 to the
tumors.
Ga-67 enters the extracellular fluid space via the
tumors leaky capillary endothelium.
Its bound to the tumor cell surface by transferrin
receptors and then transported into the cell,
where it binds to proteins such as ferritin and
lactoferrin, which are increased concentration in
tumors.

Ga-67 uptake is normally seen in which


of the following organs: salivary
glands, lacrimal glands, thymus,
spleen, breast and heart?

Ga-67 uptake is normally seen in which


of the following organs: salivary
glands, lacrimal glands, thymus,
spleen, breast and heart?
Normal uptake : salivary glands and lacrimal
glands. Spleen has uptake but is low level.
Breast uptake is variable and most prominent
in post partum.
Children after chemo : thymus
Not normal : heart due to myocarditis and
pericarditis.

Which malignant tumors useful for Ga67 for diagnosis, staging and re
staging?

Which malignant tumors useful for Ga67 for diagnosis, staging and re
staging?

Hodgkins disease
Lymphoma
Hepatoma
Melanoma

Which of the following statement


associated with Hodgkins and non
Hodgkins lymphoma?
A. Contagious spread of LN involvement in young
patients.
B. Multicentric disease with a highly variable clinical
course and high incidence of extranodal tumor
involvement.
C. Mediastinal masses are common.
D. Abdominal involvement of mesenteric and
retroperitoneal nodes is common.
E. High cure rate.
F. Variable clinical course that can be indolent or rapid
lethal

Which of the following statement


associated with Hodgkins and non
Hodgkins lymphoma?
HD
Contagious spread of LN
involvement in young
patients.
Mediastinal masses are
common.
High cure rate.

Non HD
Multicentric disease with a
highly variable clinical course
and high incidence of
extranodal tumor
involvement.
Abdominal involvement of
mesenteric and
retroperitoneal nodes is
common.
Variable clinical course that
can be indolent or rapid lethal

Tc-99m sestamibi has been used for


determination of malignancy of breast
masses.
A. Its accuracy is higher for palpable than for
non palpable masses.
B. Its sensitivity is poor for lesions less than 1
cm in size.
C. Fibroadenomas are always negative.
D. Useful for dense breast patients, previous
surgery, radiation therapy and breast
implants.

Tc-99m sestamibi has been used for


determination of malignancy of breast
masses
TTFT

Regarding In-111 capromab pendetide


(ProstaScint)
A. Murine monoclonal antibody against a
prostate specific membrane antigen express
by
more
than
95%
of
prostate
adenocarcinomas.
B. Its main indication is for localization of soft
tissue metastases after prostactomy in
patients with a rising PSA and negative bone
scan.
C. Elevated human murine antibody (HAMA)
titers are observed in 50% of patients.
D. SPECT is mandatory for the pelvis.

Regarding In-111 capromab pendetide


(ProstaScint)
TTFT

Regarding In-111 Octreoscan


A. It is somatostatin receptor imaging agent.
B. The sensitivity for all NET is very high.
C. Highest uptake is seen in the spleen and
kidneys.
D. Only NET have somatostatin receptors.

Regarding In-111 OctreoScan


TFTF

Lacrimal drainage study

Inner canthus

Using 2 - 4 MBq Tc-99m pertechnetate


Face in front collimator
Imaging time 5,10, 15, 20 minutes

Dacryoscintigram : Normal drainage on the left eye with obstruction on


the right eye at the level of common canaliculus

Radiosynovectomy

Reference
R, U, Yun. Bekerman, C. 1985, Atlas of nuclear medicine artifacts
and variants .
Biersack, H,Jurgen. Freeman, L,M. 2007, Clinical nuclear medicine
.
Krishnamurthy, G,T. Krishnamurthy, S. 2009, 2nd edition, Nuclear
hepatology .
Thrall, J, M. Ziessman, H,A. 2001, 2nd edition, The Requisites .
Ziessman, H,A. Case review nuclear medicine.
Elgazzar A,H. 2006, The pathophysiologic basis of nuclear
medicine .
Sharp, P,F. Gemmell, H, G. 2005, 3rd edition, Practical nuclear
medicine .
Mettler F, A. 2006, 5th edition, Essential of nuclear medical
imaging .
Journal from Seminar of Nuclear Medicine.

Vous aimerez peut-être aussi